You are on page 1of 35

B. Nyatakan domain, kodomain, objek, imej dan julat bagi setiap hubungan yang berikut.

BAB State the domain, codomain, object, image and range of each of the following relations. 1.2

1
FUNGSI
FUNCTIONS CONTOH 1. Diberi pasangan bertertib:
Set Q Given the ordered pairs:
7 {(9, 12), (10, 13), (11, 14), (12, 15)}
1.1 Hubungan SPM K1 ’13, ‘14 6
5 (a) Domain / Domain = {9, 10, 11, 12}
A. Wakilkan setiap hubungan yang berikut dalam bentuk (a) gambar rajah anak panah, (b) pasangan bertertib,
dan (c) graf Cartes. 3 (b) Kodomain / Codomain = {12, 13, 14, 15}
Represent each of the following relations in the forms of (a) an arrow diagram, (b) ordered pairs, and (c) a Cartesian graph. Set P
2 4 6 8 (c) Objek / Object = 9, 10, 11, 12
1.1 (a), (b), (c)
CONTOH (a) Domain / Domain = {2, 4, 6, 8} (d) Imej / Image = 12, 13, 14, 15
Diberi set A = {5, 6, 7, 8} dan set B = {10, 12, 14, 16}. Hubungan daripada set A kepada set B ialah (b) Kodomain / Codomain = {3, 5, 6, 7} (e) Imej bagi 10 / Image of 10 = 13
‘dua kali bagi’.
Given set A = {5, 6, 7, 8} and set B = {10, 12, 14, 16}. The relation from set A to set B is ‘twice of’. (c) Objek / Object = 2, 4, 6, 8 (f) Objek bagi 15 / Object of 15 = 12

(d) Imej / Image = 3, 5, 6, 7 (g) Julat / Range = {12, 13, 14, 15}
(a) A
Dua kali bagi
B
(b) {(5, 10), (6, 12), (c)
Twice of Set B
(7, 14), (8, 16)} (e) Imej bagi 6 / Image of 6 = 5
5 10
16 (f) Julat / Range = {3, 5, 6, 7}
6 12 14
7 14 12 2. Tiga kali bagi 3. Diberi hubungan antara set A = {13, 15, 26, 34}
Three times of
8 16 10 P Q dengan set B = {4, 6, 7, 8} ditakrifkan sebagai
Set A 3 9 ‘hasil tambah digit’.
5 6 7 8 Given the relation between set A = {13, 15, 26, 34} and
6 18
set B = {4, 6, 7, 8} is defined by ‘sum of digits’.
9 27
1. Diberi set A = {2, 3, 4, 5} dan set B = {6, 9, 12, 15}. Hubungan daripada set A kepada set B ialah
12 36
‘tiga kali bagi’. (a) Domain / Domain = {13, 15, 26, 34}
39
Given set A = {2, 3, 4, 5} and set B = {6, 9, 12, 15}. The relation from set A to set B is ‘three times of’.
(b) Kodomain / Codomain = {4, 6, 7, 8}
(a) (b) {(2, 6), (3, 9), (c) (a) Domain / Domain = {3, 6, 9, 12} (c) Objek / Object = 13, 15, 26, 34
Tiga kali bagi
A Three times of B (4, 12), (5, 15)} Set B
(b) Kodomain / Codomain = {9, 18, 27, 36, 39} (d) Imej / Image = 4, 6, 7, 8
15
2 6 (c) Objek / Object = 3, 6, 9, 12 (e) Imej bagi 26 / Image of 26 = 8
12
3 9 (d) Imej / Image = 9, 18, 27, 36
9
(f) Objek bagi 7 / Object of 7 = 34
4 12 (e) Objek bagi 27 / Object of 27 = 9 (g) Julat / Range = {4, 6, 7, 8}
6
5 15
Set A
(f) Julat / Range = {9, 18, 27, 36}
2 3 4 5
C. Nyatakan jenis hubungan antara set P dan set Q bagi setiap yang berikut.
State the type of relation between set P and set Q in each of the following. 1.3
2. Diberi set A = {2, 4, 6, 8} dan set B = {4, 16, 36, 64}. Hubungan daripada set A kepada set B ialah
‘kuasa dua bagi’. 1. 2. 3. Diberi pasangan bertertib:
Given set A = {2, 4, 6, 8} and set B = {4, 16, 36, 64}. The relation from set A to set B is ‘a square of’.
P Q Set Q Given the ordered pairs:

(a) (b) {(2, 4), (4, 16), (c) x 5 5


{(5, 3), (7, 3), (7, 6),
Kuasa dua bagi
Square of (6, 36), (8, 64)} Set B
A B y 8 4 (9, 3), (9, 6)}
64 z 9 3
2 4
36 10 Set P
4 16 1 2 3
16
6 36
4
8 64 Hubungan satu kepada Hubungan satu kepada satu Hubungan banyak kepada
Set A
2 4 6 8 banyak banyak

1 2

D. Selesaikan setiap yang berikut. 1.2 Fungsi SPM K1 ‘13, ‘14, ‘16 K2 ’14

Solve each of the following. 1.2 ; 1.3

CONTOH A. Tulis semua jenis hubungan dan tunjukkan setiap hubungan itu dengan gambar rajah anak panah.
1. Write down all types of relations and illustrate each relation with an arrow diagram. 2.1
Set B Set Q
Set A Set P Jenis hubungan Gambar rajah anak panah FAKTA UTAMA
–2
10 Type of relation Arrow diagram
1 7 –1 • Fungsi ialah hubungan yang setiap objeknya
11 8 0 mempunyai hanya satu imej.
2 a x
9 2 A function is a relation where each of its objects
12 Satu kepada satu b y
has only one image.
3 3 c z
13 • Hanya hubungan satu kepada satu dan
hubungan banyak kepada satu ialah fungsi.
Nyatakan Nyatakan Only one-to-one relations and many-to-one
a x relations are functions.
State State
Satu kepada banyak y (a) Hubungan satu kepada satu
(a) jenis hubungan ini. (a) jenis hubungan ini. b z One-to-one relation
the type of this relation. the type of this relation.
(b) julat hubungan ini. (b) julat hubungan ini. 1 a
2 b
the range of this relation. the range of this relation. a x 3 c
(c) imej bagi 2. (c) imej bagi 9. Banyak kepada satu b y
the image of 2. the images of 9. c z
(b) Hubungan banyak kepada satu
(d) objek bagi 12. (d) objek bagi –1. Many-to-one relation
the objects of 12. the objects of –1.
a x 1 x
b 2
(a) Hubungan banyak kepada satu (a) Hubungan banyak kepada banyak Banyak kepada banyak y
3 y
c z
(b) {11, 12} (b) {–1, 0, 2}
(c) 12 (c) 0 dan 2
(d) 2 dan 3 (d) 7 dan 8 B. Tulis setiap fungsi dengan menggunakan tatatanda fungsi. Seterusnya, jawab soalan yang berikut.
Write each function by using function notation. Hence, answer the following questions. 2.2 ; 2.3

2. Set N 3. Diberi hubungan daripada set A kepada set B ialah 1. 2. Diberi fungsi:
Set P Set Q
‘dua kali bagi’ dan set B = {2, 6, 10, 14}. Given a function:
A Given the relation from set A to set B is ‘twice of’ and 6 3
B
C
set B = {2, 6, 10, 14}.
(a) Nyatakan
12
14
6
7
 13 , 3,  15 , 5,  18 , 8,  101 , 10
State 18 9
D (a) Nyatakan objek fungsi ini.
Set M
(i) set A. State the objects of this function.
1 2 3 4 (ii) hubungan ini dalam pasangan bertertib. (a) Nyatakan julat fungsi ini. (b) Bolehkah 0 menjadi objek dalam fungsi ini?
the relation in ordered pairs. State the range of this function.
Nyatakan Berikan sebabnya.
(b) Wakilkan hubungan ini dalam bentuk gambar (b) Jika 10 ialah suatu objek dalam set P, apakah Can 0 be an object in this function? Give the reason.
State rajah anak panah. imej yang sepadan dalam set Q?
(a) hubungan ini dalam bentuk pasangan bertertib. (c) Apakah imej bagi 4 jika 4 ialah suatu objek
Represent this relation in the form of an arrow If 10 is an object in set P, what is the corresponding
the relation in the form of ordered pairs. diagram. dalam fungsi ini?
image in set Q? What is the image of 4 if 4 is an object in this function?
(b) jenis hubungan ini. (c) Jika 10 ialah suatu imej dalam fungsi ini,
the type of this relation. (a) (i) Set A = {1, 3, 5, 7} nyatakan objeknya.
(c) imej bagi 3. If 10 is an object in this function, state its image. Tatatanda fungsi: f : x → 1
the image of 3.
(ii) {(1, 2), (3, 6), (5, 10), (7, 14)} x
(d) domain hubungan ini. (b)
the domain of this relation. Set A
Dua kali bagi
Twice of Set B Tatatanda fungsi: f : x → 1 x (a) 1 , 1 , 1 , 1
2 3 5 8 10
1 2
(a) {3, 6, 7, 9} (b) 0 tidak boleh menjadi objek dalam fungsi ini
(a) {(1, B), (2, A), (3, D), (4, C)} 3 6 kerana imejnya tidak wujud.
(b) Hubungan satu kepada satu (b) 5
(c) 1
5 10
(c) D (c) 20 4
7 14
(d) {1, 2, 3, 4}

3 4

1
C. Bagi setiap fungsi berikut, cari imej bagi objek x yang diberikan.
For each of the following functions, find the images for the given objects x. 2.4 2. Satu fungsi ditakrifkan sebagai f (x) = 4x + m. 3. Diberi dua fungsi, f (x) = x + 3 dan g(x) = 2x – 6.
A function is defined by f(x) = 4x + m. Cari nilai x jika
CONTOH 1. f (x) = 2x + 5 ; x = 3, 4 (a) Cari nilai m jika f (2) = 16. Given two functions, f(x) = x + 3 and g(x) = 2x – 6. Find
Find the value of m if f(2) = 16. the value of x if
f (x) = x2 – 1 ; x = 3, –5
f(3) = 2(3) + 5 = 11 (b) Seterusnya, cari nilai x jika f (x) = 0. (a) f (x) = g(2). (b) f (x) = g(x).
f (3) = 32 – 1 = 8 Gantikan x dengan 3. Hence, find the value of x if f(x) = 0.
f(4) = 2(4) + 5 = 13 (a) f(x) = g(2) (b) f(x) = g(x)
f (–5) = (–5)2 – 1 = 24 Gantikan x dengan –5. (a) f(2) = 16 (b) f(x) = 0 x + 3 = 2(2) – 6 x + 3 = 2x – 6
4(2) + m = 16 4x + 8 = 0 = –2 3 + 6 = 2x – x
3. f (x) = 10 – 4x ; x = 3, –2 8 + m = 16 4x = –8 x = –5 x=9
2. f (x) = x + 6 ; x = 9, –12 m=8 x = –2
3
f(3) = 10 – 4(3) = –2
9
f (9) = + 6 = 9
3 f(–2) = 10 – 4(–2) = 18
f (–12) = –12 + 6 = 2 F. Selesaikan masalah yang berikut.
3
Solve the following problems. 2.3 ; 2.4

D. Bagi setiap fungsi berikut, cari objek bagi imej yang diberikan. CONTOH 1. Fungsi f ditakrifkan sebagai f : x → ax + b. Jika
For each of the following functions, find the objects for the given images. 2.4 Fungsi f ditakrifkan sebagai f : x → mx + n. f (2) = 0 dan f (4) = 6, cari nilai a dan nilai b.
Jika f (2) = 10 dan f (4) = 16, cari nilai m dan nilai n. A function f is defined as f : x → ax + b. If f(2) = 0 and
CONTOH 1. f (x) = 3x + 2 ; Imej / Images = 2, –4 A function f is defined as f : x → mx + n. If f(2) = 10 and f(4) = 6, find the values of a and b.
f (x) = 3x + 5 ; Imej / Images = 4, x f(4) = 16, find the values of m and n.
f(x) = 2 f(x) = –4 f(2) = 0
f (x) = 4 f (x) = x Jika imej sesuatu
fungsi ialah x,
3x + 2 = 2 3x + 2 = –4 f (2) = 10 2a + b = 0 ...... ➀
3x + 5 = 4 3x + 5 = x maka f (x) = x. 3x = 0 3x = –6 2m + n = 10 …… ➀ f(4) = 6
3x = –1 2x = –5 x=0 x = –2 f (4) = 16 4a + b = 6 ...... ➁
x=–1 x=–
5 4m + n = 16 …… ➁
➁ – ➀: 2a = 6
3 2
➁ – ➀: 2m = 6 a=3
m=3
2. f (x) = 4 – x ; Imej / Images = 6, –5 3. f (x) = x2 – 3 ; Imej / Images = 4, 2x Gantikan a = 3 ke dalam ➀.
Gantikan m = 3 ke dalam ➀. 2(3) + b = 0
f(x) = 6 f(x) = –5 f(x) = 4 f(x) = 2x 2(3) + n = 10 6+b=0
4–x=6 4 – x = –5 6 + n = 10 b = –6
x2 – 3 = 4 x2 – 3 = 2x
x = –2 x=9 n=4
x2 = 7 x2 – 2x – 3 = 0
x=± 7 (x – 3)(x + 1) = 0
x = 3, –1 2. f m+x
Rajah di sebelah menunjukkan 3. f Rajah di sebelah menunjukkan
x
x satu fungsi: x ax + b
fungsi f : x → ax + b. Cari
The diagram shows a function: 1 nilai a dan nilai b.
E. Selesaikan masalah yang berikut. –3
m+x The diagram shows a function
Solve the following problems. 2.3 ; 2.4
4 f:x→ ,x≠0 f : x → ax + b. Find the values
2 x
2 of a and b.
CONTOH dengan m ialah pemalar. Cari 0
1. Diberi fungsi f (x) = 3x + 5, cari nilai m.
Diberi fungsi f (x) = 2x – 4, cari Given the function f(x) = 3x + 5, find where m is a constant. Find the
Given the function f(x) = 2x – 4, find (a) objek jika imejnya ialah 8. value of m. f (1) = –3
(a) f (3). the object if the image is 8. a(1) + b = –3
(b) nilai x jika imejnya ialah 3x. (b) nilai p jika f (p) = 11. f(4) = 2 a + b = –3 ...... ➀
the value of x if the image is 3x. the value of p if f(p) = 11. m+4 =2 f(2) = 0
4 a(2) + b = 0
(a) f (3) = 2(3) – 4 (a) f(x) = 8 (b) f(p) = 11 m+4=8
=2 2a + b = 0 ...... ➁
3x + 5 = 8 3p + 5 = 11 m=4
➁ – ➀: a = 3
(b) f (x) = 3x 3x = 3 3p = 6
2x – 4 = 3x x=1 p=2 Gantikan a = 3 ke dalam ➀.
–4 = 3x – 2x 3 + b = –3
x = –4 b = –6

5 6

1.3 Fungsi Gubahan SPM K1 ’13, ‘15, ‘16 K2 ’14


B. Cari nilai x bagi setiap fungsi gubahan yang berikut.
Find the value of x for each of the following composite functions. 3.2
A. Cari fungsi gubahan berdasarkan fungsi f dan fungsi g yang diberikan.
Find the composite functions based on the given functions f and g. 3.1 ; 3.2
CONTOH 1. Diberi: f (x) = x + 6 dan g(x) = 3x + 4
CONTOH Diberi: f (x) = 2x + 5 dan g(x) = 3x – 2 Given: and

f : x → 2x + 3 fg(x) = f (x2 + 1) gf (x) = g(2x + 3) fg(3) = f (32 + 1) Given: and Jika gf (x) = 1, cari nilai x.
Cara Lain Cari nilai x jika gf (x) = 1. If gf(x) = 1, find the value of x.
g : x → x2 + 1 = 2(x2 + 1) + 3 = (2x + 3)2 + 1 = f (10) fg(3) = 2(32 ) + 5 Find the value of x if gf(x) = 1.
= 2x2 + 2 + 3 = 4x2 + 12x + 9 + 1 = 2(10) + 3 = 2(9) + 5 gf(x) = 1
= 2x2 + 5 = 4x2 + 12x + 10 = 23 = 23
gf (x) = 1 g(x + 6) = 1
g(2x + 5) = 1 3(x + 6) + 4 = 1
1. f : x → x + 4 fg(x) = f(2x + 5) gf (x) = g(x + 4) gf (1) = g(1 + 4) 3(2x + 5) – 2 = 1 3x + 18 + 4 = 1
g : x → 2x + 5 = 2x + 5 + 4 = 2(x + 4) + 5 = g(5) 6x + 15 – 2 = 1 3x = –21
= 2x + 9 = 2x + 13 = 2(5) + 5 6x = –12 x = –7
= 15
x = –2

2. f : x → 2x – 4 fg(x) = f(3x) gf (x) = g(2x – 4) fg(2) = 6(2) – 4


g : x → 3x = 2(3x) – 4 = 3(2x – 4) = 12 – 4
= 6x – 4 = 6x – 12 =8
2. Diberi: f (x) = 3 – x dan g(x) = 2x + 6 3. Diberi: f (x) = 3x + 2
Given: and Given:
3. f : x → 3 – x f 2g(x) = f 2(x2) gf 2(x) = g f f (x) gf (–2) = g[3 – (–2)] Jika fg(x) = –9, cari nilai x. Cari nilai x jika f 2(x) – 26 = 0.
= f f (x2) = g f (3 – x) = g(5) If fg(x) = –9, find the value of x. Find the value of x if f 2(x) – 26 = 0.
g:x→x 2
= f(3 – x2) = g[3 – (3 – x)] = 52
= 3 – (3 – x2) = g(x) = 25 fg(x) = –9 f 2(x) – 26 = 0
= x2 = x2 f(2x + 6) = –9 f(3x + 2) – 26 = 0
3 – (2x + 6) = –9 3(3x + 2) + 2 – 26 = 0
–2x – 3 = –9 9x + 6 + 2 – 26 = 0
x+3,x≠2 fg(x) = f  x  gf(x) = g x + 3 6+3
gf (6) = g 
6–2
4. f : x → 2x = 6 9x = 18
x–2 9 x–2 x=3 x=2
x
g:x→ x +3 x+3 = g 9 
9 = 9x = x–2 4
–2 9 9
9 x+3 ,x≠2
= = 4
x + 27 , x ≠ 18 9(x – 2) 9
=
x – 18 1
=
4
4. Diberi: f (x) = 2x + 4 dan g(x) = 3x + 2 5. Diberi: f (x) = 3 – x dan g(x) = 2x + 1
Given: and Given: 4 and
5. f : x → x + 4 2
f 2g(x) = f 2  x 
x
g2f (x) = g2  + 4 fg(4) = f  2  Cari nilai x jika fg(x) = 2x. Jika gf (x) = f (x), cari nilai x.
2 2 4 Find the value of x if fg(x) = 2x. If gf(x) = f(x), find the value of x.

g:x→ 2,x≠0 = f f  2x  = gg  x + 8  = f 1


x 2 2 fg(x) = 2x gf(x) = f(x)

   
2 =g 2 1 f(3x + 2) = 2x
=f x +4 x+8 2 g 3 – x  = 3 – x
2 = +4 2(3x + 2) + 4 = 2x 4 4
2 2
6x + 4 + 4 = 2x 2 3 – x  + 1 = 3 – x
= f  1x + 4 = g
4 1
x + 8
=4 4x = –8 4 4
4
1 x = –2 6– x +1=3– x
x +4 = 2 2 4
= +4 4
2 4= x
1 + 12x , x ≠ 0 x+8 4
= x = 16
2x = x+8
2

7 8

2
C. Diberi fungsi f dan fungsi gubahan fg, cari fungsi g.
3.3
2. Diberi f (x) = 2 – 3x dan fg(x) = –x – 1, cari 3. Diberi f (x) = 4(x + 1) dan fg(x) = 4(x2 + 4), cari
Given the function f and composite function fg, find the function g.
Given f(x) = 2 – 3x and fg(x) = –x – 1, find gf (x).
CONTOH 1. f (x) = x – 3 ; fg(x) = 2x (a) g(x + 2). Given f(x) = 4(x + 1) and fg(x) = 4(x2 + 4), find gf(x).
(b) g(3).
f (x) = 5x + 1 ; fg(x) = 5x2 – 4 fg(x) = 4(x2 + 4)
fg(x) = 2x
Perhatikan bahawa g (a) fg(x) = –x – 1 4[g(x) + 1] = 4(x2 + 4)
fg(x) = 5x2 – 4 ialah fungsi dalaman. g(x) – 3 = 2x 2 – 3g(x) = –x – 1 g(x) = x2 + 3
5g(x) + 1 = 5x2 – 4 g(x) = 2x + 3
g(x) = x + 3
5g(x) = 5x2 – 5 3 gf(x) = g[4(x + 1)]
2 g(x + 2) = (x + 2) + 3 = [4(x + 1)]2 + 3
g(x) = 5x – 5 3
5 = 16(x + 1)2 + 3
2 = +5
x
g(x) = x – 1 3

(b) g(3) = 3 + 3
3
=2

7 E. Diberi fungsi g dan fungsi gubahan fg, cari fungsi f.


2. f (x) = 4x + 5 ; fg(x) = 2x + 3. f (x) = 2 , x ≠ 0 ; fg(x) = 1 , x ≠ – 3 Given the function g and composite function fg, find the function f. 3.3
2 x 2x + 3 2
CONTOH 1. g(x) = 2x – 4 ; fg(x) = 2x – 9
1
fg(x) = 2x + 7 fg(x) = 2
g(x) = x – 4 ; fg(x) = 2x – 1 2
2 2x + 3
fg(x) = 2x – 9
4g(x) + 5 = 2x + 7 2 = 1
fg(x) = 2x2 – 1 Perhatikan bahawa f ialah fungsi f(2x – 4) = 2x – 9
2 g(x) 2x + 3 luar. Dalam Bahagian C, fungsi g
4g(x) = 2x – 3 g(x) = 4x + 6 f (x2 – 4) = 2x2 – 1 ialah fungsi dalaman. Katakan y = 2x – 4
2 Katakan y = x2 – 4. Maka, 2x = y + 4

4 
g(x) = 1 2x – 3
2  Maka, x2 = y + 4 f(y) = (y + 4) – 9
=y–5
f (y) = 2(y + 4) – 1
g(x) = x – 3 Maka, f(x) = x – 5
2 8 = 2y + 8 – 1
= 2y + 7

Maka, f (x) = 2x + 7 Tulis jawapan dalam sebutan x


dengan menukar pemboleh ubah y
kepada x.
D. Selesaikan masalah yang berikut.
Solve the following problems. 3.2 ; 3.3 2
2. g(x) = x ; fg(x) = x + 2 3. g(x) = 5x – 4 ; fg(x) = 6 – 5x
CONTOH 2 4
1. Diberi f (x) = 2x – 4 dan fg(x) = 6(x + 1), cari 2
Given f(x) = 2x – 4 and fg(x) = 6(x + 1), find x fg(x) = 6 – 5x
Diberi f (x) = 5x – 4 dan fg(x) = 1 – 5x, cari fg(x) = +2
(a) g(x). 4 f(5x – 4) = 6 – 5x
Given f(x) = 5x – 4 and fg(x) = 1 – 5x, find
x = x2
(a) g(x). (b) g(–1). f  +2 Katakan y = 5x – 4
2 4 Maka, 5x = y + 4
(b) g(–2).
(a) fg(x) = 6(x + 1) Katakan y = x f(y) = 6 – (y + 4)
2g(x) – 4 = 6x + 6 2
(a) fg(x) = 1 – 5x =6–y–4
2g(x) = 6x + 10 Maka, x = 2y
5g(x) – 4 = 1 – 5x =2–y
5g(x) = 5 – 5x g(x) = 3x + 5 (2y)2
f(y) = +2 Maka, f(x) = 2 – x
g(x) = 1 – x (b) g(–1) = 3(–1) + 5 4
= –3 + 5 = y2 + 2
(b) g(–2) = 1 – (–2)
=1+2 =2
Maka, f(x) = x2 + 2
=3

9 10

F. Selesaikan masalah yang berikut. 1.4 Fungsi Songsang SPM K1 ’15, ‘16 K2 ’14

Solve the following problems. 3.2 ; 3.3


A. Cari nilai m dan nilai n dengan menggunakan pemetaan songsang.
CONTOH Find the values of m and n by using inverse mapping. 4.1
1. Diberi bahawa f (x) = mx + 2, g(x) = nx – 4 dan
Diberi bahawa f (x) = ax + 3 dan g(x) = 4 + 2x. fg(x) = 12x – 3. Cari nilai m dan nilai n. CONTOH FAKTA UTAMA
Cari nilai a jika fg(3) = 23. Given f(x) = mx + 2, g(x) = nx – 4 and fg(x) = 12x – 3.
f
Given f(x) = ax + 3 and g(x) = 4 + 2x. Find the value of a Find the values of m and n. • Jika fungsi f memetakan x kepada y, maka fungsi yang
x 2x + 3
memetakan y kembali kepada x ialah fungsi songsang
if fg(3) = 23.
2 7 bagi f dan ditulis sebagai f –1.
fg(x) = 12x – 3 If a function f maps x to y, then the function that maps y
fg(3) = 23 f(nx – 4) = 12x – 3 m 11 back to x is the inverse function of f and is written as f –1.

f [4 + 2(3)] = 23 m(nx – 4) + 2 = 12x – 3 n 17 f

f (10) = 23 mnx – 4m + 2 = 12x – 3


x y
a(10) + 3 = 23 Maka,
–4m + 2 = –3 dan mn = 12 f –1(11) = m f –1(17) = n f –1
10a = 20
–4m = –5 5 n = 12 ff –1(11) = f (m) ff –1(17) = f (n) • f(x) = y
a=2 f –1f(x) = f –1(y)
4 11 = 2m + 3 17 = 2n + 3
1
m=1 n=93 x = f –1(y) atau/or f –1(y) = x
4 5 8 = 2m 14 = 2n
1
m=4 n=7 • f –1(x) ≠
y

2. Diberi dua fungsi, f (x) = x + 1 dan g(x) = ax + 3. 3. Diberi bahawa fungsi f (x) = 4 , x ≠ 0, dan fungsi 1.
Cari x x
f
3x – 2 f –1(10) = m f –1(13) = n
Given two functions, f(x) = x + 1 and g(x) = ax + 3. Find gubahan fg(x) = 2x. Cari f f –1(10) = f(m) f f –1(13) = f(n)
3 7 10 = f(m) 13 = f(n)
(a) f (5). Given the function f(x) =
4
, x ≠ 0, and composite function
(b) nilai a jika gf(5) = 21. x m 10 10 = 3m – 2 13 = 3n – 2
the value of a if gf(5) = 21. fg(x) = 2x. Find 12 = 3m 15 = 3n
n 13
(a) g(x). (b) gf(2). m=4 n=5
(a) f(5) = 5 + 1
=6 (a) fg(x) = 2x (b) gf(2) = g  4 
4 2
= 2x = g(2)
(b) gf(5) = 21 g(x)
2
g(6) = 21 g(x) = 2 , x ≠ 0 =
2
a(6) + 3 = 21 x 2.
=1
f
3 +2 f –1(5) = m f –1(3.5) = n
6a = 18 x x
f f –1(5) = f(m) f f –1(3.5) = f(n)
a=3 6 2.5 5 = f(m) 3.5 = f (n)
3 +2
3.5 = 3 + 2
m 5
5=
n 3.5
m n
3
4. Diberi bahawa f (x) = ax + b, a  0 dan f 2(x) 5. Diberi bahawa f (x) = x + 4, g(x) = 4 – 2x dan =3 1.5 = 3
= 9x + 20. Cari nilai a dan nilai b. fg(x) = a – bx. Cari nilai a dan nilai b. m n
Given f(x) = ax + b, a  0 and f (x) = 9x + 20. Find the
2 Given f(x) = x + 4, g(x) = 4 – 2x and fg(x) = a – bx. Find m=1 n=2
values of a and b. the values of a and b.

2
f (x) = 9x + 20 fg(x) = a – bx
f(ax + b) = 9x + 20 f(4 – 2x) = a – bx 3. f x+5 f –1(4) = m f –1(13) = n
a(ax + b) + b = 9x + 20 4 – 2x + 4 = a – bx x
2 f f –1(4) = f(m) f f –1(13) = f(n)
a2x + ab + b = 9x + 20 8 – 2x = a – bx 1 3 m+5 n+5
Maka, a2 = 9 Maka, a = 8 dan b = 2. 4= 13 =
m 4 2 2
a = 3 (a  0) 8=m+5 26 = n + 5
n 13
ab + b = 20 m=3 n = 21
3b + b = 20
4b = 20
b=5

11 12

3
B. Selesaikan masalah yang berikut. C. Selesaikan masalah yang berikut.
Solve the following problems. 4.1 ; 4.2 Solve the following problems. 4.3

CONTOH CONTOH 1. Diberi f (x) = 2x + m dan f –1(x) = kx – 1, dengan


x
1. Diberi f (x) = + 3, cari f –1(x) dan f –1(4). Diberi g(x) = 4x + 1 dan h(x) = x2 + 2x – 3, cari keadaan m dan k ialah pemalar. Cari nilai m dan
Diberi f (x) = 3x – 6, cari f –1(x) dan f –1(3). 2
x Given that g(x) = 4x + 1 and h(x) = x2 + 2x – 3, find nilai k.
Given that f(x) = 3x – 6, find f –1(x) and f –1(3). Given that f(x) = + 3, find f –1(x) and f –1(4).
2 (a) g–1(x). (b) hg–1(–3). Given f(x) = 2x + m and f –1(x) = kx – 1, where m and k are
–1 constants. Find the values of m and k.
Katakan y = f (x)
Katakan y = f –1(x) (a) Katakan y = g (x) –1
(b) hg–1(–3)
f (y) = ff –1(x) Katakan y = f –1(x)
f(y) = x g(y) = gg–1(x)
f (y) = x
3y – 6 = x
y
+3=x g(y) = x
=h 
–3 – 1
4  f(y) = x
2 4y + 1 = x 2y + m = x
3y = 6 + x = h(–1)
y
=x–3 4y = x – 1 = (–1)2 + 2(–1) – 3 y= x–m
2
y= 6+x 2
3 y = 2x – 6 y= x–1 =1–2–3
Maka, f (x) = x – m = x – m
–1
4 = –4 2 2 2
Maka, f –1(x) = 6 + x Gantikan y dengan f –1(x). Maka, f –1(x) = 2x – 6 x–1 –1
Bandingkan dengan f (x) = kx – 1 yang diberi.
3 Maka, g–1(x) =
6+3 f –1(4) = 2(4) – 6 4
f –1(3) = =3 =2 Maka, k = 1 dan m = 1
3 2 2
m=2
2. Diberi f (x) = 2x – 5, cari f –1(x) dan f –1(3). 3. Diberi f (x) = 4x + 6, cari f –1(x) dan f –1(2).
Given that f(x) = 2x – 5, find f –1(x) and f –1(3). Given that f(x) = 4x + 6, find f –1(x) and f –1(2). 2. Diberi g(x) = 2x + 1, cari 3. Diberi g(x) = 4x + 3 dan h(x) = x2 – 4x, cari
Given that g(x) = 2x + 1, find Given that g(x) = 4x + 3 and h(x) = x2 – 4x, find
Katakan y = f –1(x) Katakan y = f –1(x) (a) g(2). (b) nilai m jika g –1(m) = 4. (a) g –1(11). (b) g –1h(x).
the value of m if g–1(m) = 4.
f(y) = x f (y) = x (a) Katakan y = g –1(x) Maka,
2y – 5 = x 4y + 6 = x (a) g(2) = 2(2) + 1 x–3
g(y) = x
2y = x + 5 4y = x – 6 =5 g–1(x) =
4y + 3 = x 4
y=x+5 y=x–6 (b) Katakan –1
Maka, g (x) =
x–1 4y = x – 3 –1
g (11) =
11 – 3
2 4 2 x–3 4
y = g–1(x) y=
g–1(m) = 4 4 =2
Maka, f –1(x) = x + 5 Maka, f –1(x) = x – 6 g(y) = x
m–1
2 4 2y + 1 = x =4 (b) g–1h(x) = g–1(x2 – 4x)
2
f –1(3) =
3+5 f –1(2) = 2 – 6 y=
x–1 m–1=8 =
(x2 – 4x) – 3
2 4 2 4
= –1 m=9 x2 – 4x – 3
=4 =
4
2 5. Diberi f (x) = x – 1 , x ≠ –1, cari f –1(x) dan f –1(–2).
4. Diberi f (x) = , x ≠ –1, cari f –1(x) dan f –1(–3).
x+1 x+1 4. Diberi h(x) = 2x + 3 dan g(x) = 3x – 1, cari 4 , x ≠ 2 , cari
Given that h(x) = 2x + 3 and g(x) = 3x – 1, find
5. Diberi f (x) = x + 8 dan g(x) =
Given that f(x) =
2
, x ≠ –1, find f –1(x) and f –1(–3). Given that f(x) =
x–1
, x ≠ –1, find f –1(x) and f –1(–2). 3x – 2 3
x+1 x+1 (a) hg –1(x). (b) gh–1(x). Given that f(x) = x + 8 and g(x) =
4 2
, x ≠ , find
KBAT 3x – 2 3
Katakan y = f –1(x) y = f –1(x) Katakan (a) hg–1(x)
Katakan (a) g –1(4). (b) f –1g(x).
f(y) = x f (y) = x y = h–1(x) = hx + 1
2 =x y–1 h(y) = x 3 (a) Katakan (b) Katakan
=x
y+1 y+1 2y + 3 = x = 2 x + 1  + 3 y = g –1(4) y = f –1(x)
y – 1 = x(y + 1) y= x–3 3 g(y) = 4 f(y) = x
y+1= 1
2 x y – 1 = xy + x 2 = 2x + 2 + 3 4 =4 y+8=x
3
2 y – xy = x + 1 Maka, h–1(x) = x – 3 3y – 2 y=x–8
y+1=
x y(1 – x) = x + 1 2 = 2x + 11 3y – 2 = 1
Katakan 3 3y = 3 Maka, f –1(x) = x – 8
x+1
y= 2 –1 y= y = g–1(x) (b) gh–1(x) y=1
x 1–x
g(y) = x f –1g(x) = f –1  4 
2 – 1, x ≠ 0 = g x – 3  Maka, g–1(4) = 1
3x – 2
Maka, f –1(x) = Maka, f –1(x) = x + 1 , x ≠ 1 3y – 1 = x 2 = 4 – 8,
x 1–x 3x – 2
y=x+1 = 3 x – 3  – 1
f –1(–3) =
2 –1=–5
f –1(–2) =
–2 + 1 = – 1 3 2 x≠ 2
–3 3 1 – (–2) 3 Maka, g–1(x) = x + 1 = 3x – 11 3
3 2

13 14

6. Diberi fungsi f : x → 3x + 5 dan g : x → 2x – 3, cari 9. Diberi fungsi f (x) = 2x dan g(x) = kx + h, dengan
PRAKTIS FORMATIF Kertas 1 ANALISIS SOALAN SPM
KLONgf(x). KLONkeadaan h dan k ialah pemalar. Ungkapkan h dalam
Subtopik 2013 2014 2015 2016 SPM SPM
1.1 S. 1 S. 1 – – ’13 Given the functions f : x → 3x + 5 and g : x → 2x – 3, find gf(x). ’15 sebutan k dengan keadaan gf(2) = 8.
Jawab semua soalan. 1.2 S. 3 S. 2 – S. 11 1.3
5.2
[2]
1.3
5.2
Given the functions f(x) = 2x and g(x) = kx + h, where h and k
1.3 S. 2 – S. 1(a), S. 2 S. 12, S. 13(b)
Answer all the questions. are constants. Express h in terms of k such that gf(2) = 8.
1.4 – – S. 1(b) S. 13(a)
gf(x) = g(3x + 5) [3]
1. Rajah di bawah menunjukkan hubungan antara set P = 2(3x + 5) – 3
KLON dan set Q dalam bentuk graf.
3. Diberi bahawa fungsi f(x) = p – 4x, dengan keadaan p = 6x + 7 g f (2) = 8
SPM
’13 The diagram shows the relation between set P and set Q in the
KLON
SPM
ialah pemalar. Cari nilai p dengan keadaan f(p) = 9. g(2 × 2) = 8
1.1
5.2 ’13 It is given that the function f(x) = p – 4x, where p is a constant.
graph form. 1.2
5.2 g(4) = 8
Find the value of p such that f(p) = 9.
d
[2] k(4) + h = 8
c 4k + h = 8
Set Q b f(p) = 9 h = 8 – 4k
p – 4p = 9
a
–3p = 9
p = –3 7. Diberi fungsi h : x → mx + 3, k : x → 2x – 1 dan
2 4 6 8 KLON
SPM
hk(x) = 2mx + p. Ungkapkan m dalam sebutan p.
Set P ’16 Given the functions h : x → mx + 3, k : x → 2x – 1 and
1.3
5.2
hk(x) = 2mx + p. Express m in terms of p.
Nyatakan
State 4. Rajah di bawah menunjukkan fungsi f : x → x – 2m, [3]
(a) julat hubungan itu. KLON dengan keadaan m ialah pemalar.
hk(x) = 2mx + p 10. Diberi fungsi g : x → 2x + 4 dan fg : x → 4x2 +
SPM
KLON
the range of the relation. ’14 The diagram shows the function f : x → x – 2m, where m is h(2x – 1) = 2mx + p SPM 16x + 10. Cari
(b) jenis hubungan antara set P dan set Q. 1.2
5.2
a constant. ’16 Given the functions g : x → 2x + 4 and fg : x → 4x2 +
f m(2x – 1) + 3 = 2mx + p 16x + 10. Find
the type of relation between set P and set Q. x x – 2m 2mx – m + 3 = 2mx + p (a) g–1(x).
[2]
–m + 3 = p 1.4

m=3–p (b) f(x).


(a) Julat = {a, b, d} 6 10
1.3
[3]
(b) Hubungan banyak kepada satu
(a) Katakan y = g –1(x)
Cari nilai m. g(y) = x
Find the value of m. 2y + 4 = x
[2] 2y = x – 4
2. Rajah di bawah menunjukkan hubungan antara set A 8. Rajah di bawah menunjukkan fungsi gubahan gf yang
KLON dan set B dalam bentuk gambar rajah anak panah.
f(6) = 10 y= x–4
SPM 6 – 2m = 10 KLON
SPM memetakan a kepada c. 2
’14 The diagram shows the relation between set A and set B in the
1.1
5.2
arrow diagram form.
2m = –4 ’15 The diagram shows the composite function gf that maps a Maka, g –1(x) = x – 4 .
m = –2 to c. 2
2 5
Set B gf (b) fg(x) = 4x2 + 16x + 10
f(2x + 4) = 4x2 + 16x + 10
5. Diberi fungsi f : x → 2x – 4, cari
KLON Given the function f : x → 2x – 4, find a b c
SPM Katakan y = 2x + 4
Set A (a) nilai x apabila f(x) memeta kepada diri sendiri.
–1 0 1
’16 y–4
1.2
5.2
the value of x when f(x) maps onto itself. x=
(a) Wakilkan hubungan itu dalam bentuk pasangan Nyatakan 2
(b) nilai h dengan keadaan f(3h – 1) = 3h. y–4 2 y–4
f(y) = 4
2 
+ 16
2 
bertertib. the value of h such that f(3h – 1) = 3h. State + 10
Represent the relation in the form of ordered pairs. [4] (a) fungsi yang memetakan a ke b.
(b) Nyatakan domain hubungan itu. 1.3 the function that maps a to b. = (y – 4)2 + 8(y – 4) + 10
State the domain of the relation. (b) g–1(c). = y2 – 8y + 16 + 8y – 32 + 10
(a) f(x) = x
[2] 1.4
[2] = y2 – 6
2x – 4 = x
x=4 Maka, f(x) = x2 – 6
(a) {(–1, 5), (0, 2), (1, 5)} (a) Fungsi f
(b) f(3h – 1) = 3h –1
(b) {–1, 0, 1} 2(3h – 1) – 4 = 3h (b) g (c) = b
6h – 2 – 4 = 3h
6h – 6 = 3h
3h = 6
h=2

15 16

4
PRAKTIS FORMATIF Kertas 2
Subtopik
ANALISIS SOALAN SPM
2013 2014 2015 2016
FOKUS KBAT
1.1 – – – –
Jawab semua soalan. 1.2 – S. 3(a)(ii) – –

Answer all the questions. 1.3


1.4


S. 3(b)
S. 3(a)(i)




Kemahiran Kognitif: Menganalisis
Konteks: Fungsi Gubahan, Fungsi Songsang
1. Dalam rajah di bawah, fungsi f memetakan set A kepada
KLON
SPM
set B dan fungsi g memetakan set B kepada set C. 2. Diberi dua fungsi, f : x → 4x – 5 dan g : x → x – 2. Cari Dalam rajah di bawah, fungsi f memetakan set A kepada set B dan fungsi g memetakan set B kepada set C, dengan
’14 In the diagram, the function f maps set A to set B and the 4 keadaan p ialah pemalar.
x
function g maps set B to set C. Given two functions, f : x → 4x – 5 and g : x → – 2. Find In the diagram, the function f maps set A to set B and the function g maps set B to set C, where p is a constant.
4
f g
A B C (a) f –1(x), lakarkan graf f –1(x) dan kemudiannya A B C
1.4
nyatakan domain x.
x 3x – 2 12x – 5 f –1(x), sketch the graph of f –1(x) and then state the f g
domain of x. 6 –18 P
[3]
Cari (b) f –1g(x).
Find Diberi f(x) = kx2 – 5x, dengan keadaan k ialah pemalar.
1.4
[2]
(a) dalam sebutan x, fungsi (c) h(x) dengan keadaan hg(x) = x – 8. Given that f(x) = kx2 – 5x, where k is a constant.
in terms of x, the function 1.3 h(x) such that hg(x) = x – 8. (a) Tentukan sama ada hubungan yang memetakan set B kepada set A ialah suatu fungsi atau bukan. Nyatakan sebab
(i) yang memetakan set B kepada set A. [3]
1.4 which maps set B to set A. anda.
Determine whether the relation which maps set B to set A is a function. State your reason.
(ii) g(x). (a) Diberi f(x) = 4x – 5. (b) Cari nilai k.
[5] Info KBAT
1.2
Katakan y = f –1(x) Find the value of k.
(b) nilai x dengan keadaan fg(x) = 5x + 14. f(y) = x (c) Seterusnya, cari fungsi yang memetakan set A kepada set C, dalam
1.3 the value of x such that fg(x) = 5x + 14. Fungsi yang memetakan set A kepada set C
4y – 5 = x sebutan x, dengan keadaan fungsi yang memetakan set C kepada set B ialah gf manakala fungsi yang memetakan
[2] 4y = x + 5 18x
ialah . set C kepada set B ialah g –1.
y= x+5 x–1 The function which maps set A to set C is gf
(a) (i) Fungsi yang memetakan set B kepada set A 4 while the function which maps set C to set B
Hence, find the function which maps set A to set C, in terms of x, where the function
ialah f –1(x). is g–1.
18x
Berdasarkan rajah yang diberi, f(x) = 3x – 2. Maka, f –1(x) = x + 5 which maps set C to set B is
x–1
.
Katakan y = f –1(x) 4
Maka, f(y) = x Lakaran graf f –1(x):
18x
3y – 2 = x f –1(x)
(a) Bukan fungsi. (c) g–1(x) =
3y = x + 2 Kerana hubungan yang memetakan set B kepada set A x–1
ialah hubungan satu kepada banyak. Katakan g(x) = y
y=x+2 5
3 4 Maka, g–1(y) = x
x+2 (b) f(x) = kx2 – 5x
Maka, f –1(x) = 18y
3 x f(6) = –18 =x
–5 0 y–1
(ii) Berdasarkan rajah yang diberi, k(62) – 5(6) = –18 18y = xy – x
gf(x) = 12x – 5 Domain x ialah semua nilai nyata. 36k – 30 = –18 xy – 18y = x
Maka, g(3x – 2) = 12x – 5 36k = 12 y(x – 18) = x
(b) f –1g(x) = f –1  x – 2 1 x
Katakan u = 3x – 2. 4 k= y=
u+2 3 x – 18
Maka, x = dan x
3  – 2 + 5 x 1
dan f(x) = x2 – 5x
u+2 –5 = 4 g(x) =
x – 18 3
g(u) = 12
3 
4
x + 12 1 2
= 4u + 8 – 5 = gf(x) = g  x – 5x
= 4u + 3 16 3
Maka, g(x) = 4x + 3 1 2
(c) hg(x) = x – 8 x – 5x
3
x =
(b) fg(x) = 5x + 14 h – 2 = x – 8 1 2
f(4x + 3) = 5x + 14 4  3 – 5x – 18
x
3(4x + 3) – 2 = 5x + 14 x
Katakan u = – 2, x2 – 15x
12x + 9 – 2 = 5x + 14 4 = 2 , x ≠ –3, 18
maka x = 4u + 8. x – 15x – 54
7x = 7
x=1 h(u) = (4u + 8) – 8
= 4u
Maka, h(x) = 4x.

17 18

2.2 Penyelesaian Persamaan Kuadratik SPM K1 ’13, ’14, ’16


BAB

2
PERSAMAAN KUADRATIK A. Selesaikan persamaan kuadratik yang berikut secara pemfaktoran.
2.1 (a)
QUADRATIC EQUATIONS Solve the following quadratic equations by factorization.

CONTOH 1. x2 – 2x – 3 = 0 2. 2x2 + 5x – 12 = 0
x(x – 11) = –30
2.1 Persamaan Kuadratik dan Punca-puncanya SPM K1 ’13, ’14, ’15 x2 – 2x – 3 = 0 2x2 + 5x – 12 = 0
x(x – 11) = –30
A. Tulis nilai-nilai a, b dan c bagi setiap persamaan kuadratik yang berikut. (x – 3)(x + 1) = 0 (2x – 3)(x + 4) = 0
x2 – 11x + 30 = 0
Write down the values of a, b and c for each of the following quadratic equations. 1.1 (x – 6)(x – 5) = 0 x – 3 = 0 atau x + 1 = 0 2x – 3 = 0 atau x + 4 = 0
x=3 x = –1 3
1. 3x2 + 4x = 0 2. x2 – x + 3 = 0 3. –2x2 + 9 = 0 x – 6 = 0 atau x–5=0 x= x = –4
2
x=6 x=5
a = 3, b = 4, c = 0 a = 1, b = –1, c = 3 a = –2, b = 0, c = 9
3. x2 + 3x = 10 4. 2x(x + 2) = –2 – x 5. x(2x + 9) = 18
B. Tentukan sama ada setiap persamaan yang berikut ialah persamaan kuadratik atau bukan. Nyatakan sebabnya x2 + 3x – 10 = 0 2x2 + 4x + x + 2 = 0 2x2 + 9x – 18 = 0
dengan jelas. (x + 5)(x – 2) = 0 2x2 + 5x + 2 = 0 (2x – 3)(x + 6) = 0
Determine whether each of the following equations is a quadratic equation. State the reason clearly. 1.1
(2x + 1)(x + 2) = 0
x + 5 = 0 atau x – 2 = 0 2x – 3 = 0 atau x + 6 = 0
1. 4(x – 1) = 2x + 3 2. (x – 4)(2x + 3) = 0 3. 22 + x − 3 = 0 x = –5 x=2 2x + 1 = 0 atau x + 2 = 0 x=
3
x = –6
x 1 2
x=– x = –2
2
4(x – 1) = 2x + 3 (x – 4)(2x + 3) = 0
4x – 4 = 2x + 3 2x2 + 3x – 8x – 12 = 0 Bukan persamaan kuadratik.
2x – 7 = 0 2x2 – 5x – 12 = 0 Kerana kuasa tertinggi x ialah 1. B. Selesaikan persamaan kuadratik yang berikut secara penyempurnaan kuasa dua.
Solve the following quadratic equations by completing the square. 2.1 (b)
Bukan persamaan kuadratik. Ya. Kerana kuasa tertinggi x
Kerana kuasa tertinggi x ialah 1. ialah 2. CONTOH 1. x2 – 8x + 10 = 0
2
x + 6x – 12 = 0
C. Tulis semula setiap persamaan kuadratik dalam bentuk am, ax2 + bx + c = 0. x2 – 8x + 10 = 0
x2 + 6x – 12 = 0 x2 – 8x = –10
Rewrite each quadratic equation in the general form, ax2 + bx + c = 0. 1.1
x2 + 6x = 12 –8 2 = –10 + –8 2
Tambahkan  2  x2 – 8x + 
2 2
b 2
CONTOH 1. (x + 2)2 = 4x 2. 5x + 6 – x2 = 3x(x + 2) 6 2
x2 + 6x +   = 12 +  
6 2
2 2 di kedua-dua belah
x(x – 3) = 2x2 + 5 persamaan. (x – 4)2 = –10 + 16
x2 + 4x + 4 = 4x 5x + 6 – x2 = 3x2 + 6x (x + 3)2 = 12 + 32 (x – 4)2 = 6
x2 + 4 = 0 4x2 + x – 6 = 0 (x + 3)2 = 21 x–4=± 6
x2 – 3x = 2x2 + 5
x + 3 = ± 21 x=4± 6
x2 + 3x + 5 = 0
x = –3 ± 21 = 4 + 6 atau 4 – 6
= –3 + 21 atau –3 – 21 = 6.450 atau 1.551
D. Selesaikan setiap yang berikut. = 1.583 atau –7.583 (4 A.B.)
Solve each of the following. 1.2

1. Cari nilai m jika x = 2 ialah punca persamaan 2. Cari nilai m dan nilai n jika 1 dan –4 ialah punca 2. –x2 – 4x + 2 = 0 3. –2x2 + 12x – 8 = 0
x2 + mx – 8 = 0. persamaan mx2 + nx – 4 = 0.
Find the value of m if x = 2 is a root of the equation Find the values of m and n if 1 and –4 are the roots of the –x2 – 4x + 2 = 0 –2x2 + 12x – 8 = 0
x2 + mx – 8 = 0. equation mx2 + nx – 4 = 0. x2 + 4x = 2 x2 – 6x + 4 = 0
4 2 4 2 x2 – 6x = –4
x + 4x +   = 2 +  
2
Gantikan x = 2 ke dalam x2 + mx – 8 = 0. m(1)2 + n(1) – 4 = 0 Gantikan x = 1. 2 2 –6 2
x2 – 6x +   = –4 +  
–6 2
m+n=4 …… ➀ (x + 2)2 = 2 + 4 2 2
22 + m(2) – 8 = 0 (x + 2)2 = 6 (x – 3)2 = –4 + 9
4 + 2m – 8 = 0 m(–4)2 + n(–4) – 4 = 0 Gantikan x = –4.
x+2=± 6 (x – 3)2 = 5
2m = 4 16m – 4n = 4
x = –2 ± 6 x–3=± 5
m=2 4m – n = 1 …… ➁
= –2 + 6 atau –2 – 6 x=3± 5
➀ + ➁: 5m = 5 dan 1 + n = 4 = 0.4495 atau –4.4495 = 3 + 5 atau 3 – 5
m=1 n=3 = 5.236 atau 0.7639

19 20

5
C. Selesaikan persamaan kuadratik dengan menggunakan rumus: x = –b ± b2 – 4ac E. Diberi α dan β ialah punca persamaan kuadratik. Bentukkan persamaan kuadratik baharu dengan
Solve the quadratic equations by using the formula: 2a 2.1 (c) hasil tambah dan hasil darab punca-puncanya seperti berikut.
Given α and β are the roots of quadratic equations. Form the new quadratic equations with the given sum and product of the
CONTOH 1. x2 + 4x – 3 = 0 roots as follows. 2.2
–2x2 – x + 5 = 0
CONTOH
a = –2, b = –1, c = 5 a = 1, b = 4, c = –3 1. α + β = 2, αβ = –15
2
x = –4 ± 4 – 4(1)(–3)
2
α + β = 1, αβ =
x = –(–1) ± (–1) – 4(–2)(5)
2
9 x2 – (α + β)x + αβ = 0
2(–2) 2(1) x2 – 2x + (–15) = 0
–4 ± 28 α + β = HTP x2 – 2x – 15 = 0
= 1 ± 41 x – (α + β)x + αβ = 0
2
= αβ = HDP
–4 2(1) 2
Gunakan kalkulator untuk = 0.6458 atau –4.6458 x2 – (1)x + = 0
= –1.851 atau 1.351 mendapat nilai ini. 9
9x2 – 9x + 2 = 0
2. 2x2 – 5x – 4 = 0 3. 3x2 + 18x + 5 = 0
5 3
a = 2, b = –5, c = –4 a = 3, b = 18, c = 5 2. α + β = –7, αβ = 12 3. α + β = , αβ =
2 2
x = –(–5) ± (–5) – 4(2)(–4) x = –18 ± 18 – 4(3)(5)
2 2
x – (α + β)x + αβ = 0
2 x – (α + β)x + αβ = 0
2
2(2) 2(3) x2 – (–7)x + 12 = 0 5 3
x2 – x + = 0
= 5 ± 57 = –18 ± 264 x2 + 7x + 12 = 0 2 2
4 6 2x2 – 5x + 3 = 0
= 3.1375 atau –0.6375 = –0.2920 atau –5.7080

4. –3x2 – 6x + 10 = 0 5. 6x2 – 7x + 2 = 0

a = –3, b = –6, c = 10 a = 6, b = –7, c = 2 F. Cari hasil tambah dan hasil darab punca-punca bagi setiap persamaan kuadratik yang berikut.
Find the sum and the product of the roots of each of the following quadratic equations. 2.2
x = –(–6) ± (–6) – 4(–3)(10) x = –(–7) ± (–7) – 4(6)(2)
2 2

2(–3) 2(6) CONTOH 1. x2 – 6x + 8 = 0 2. x2 + 7x + 6 = 0


±
= 7± 1
= 6 156 2x2 + 5x – 6 = 0
–6 12 x2 – 6x + 8 = 0 x2 + 7x + 6 = 0
= –3.0817 atau 1.0817 = 2 atau 1 2x2 + 5x – 6 = 0 x2 – (–7)x + 6 = 0
3 2 HTP = 6
5
x2 + x – 3 = 0 HDP = 8 HTP = –7
2
HDP = 6
D. Bentukkan persamaan kuadratik dengan setiap pasangan punca x1 dan x2 yang diberikan. x2 – – 5  x + (–3) = 0
2
Form a quadratic equation with each given pair of roots, x1 and x2. 2.2

CONTOH HTP = – 5
1. x1 = 3, x2 = 5 2
x1 = –3, x2 = 4 HDP = –3
(x – 3)(x – 5) = 0
[x – (–3)](x – 4) = 0 x2 – 8x + 15 = 0
(x + 3)(x – 4) = 0 3. x2 – 5x – 6 = 0 4. –x2 + 9x + 20 = 0 5. 3x2 + 12x – 21 = 0
x2 – 4x + 3x – 12 = 0
x2 – x – 12 = 0 x2 – 5x – 6 = 0 –x2 + 9x + 20 = 0 3x2 + 12x – 21 = 0
x2 – 5x + (–6) = 0 x2 – 9x – 20 = 0 x2 + 4x – 7 = 0
x2 – 9x + (–20) = 0 x2 – (–4)x + (–7) = 0
2. x1 = 4, x2 = –6 3. x1 = –5, x2 = –7 HTP = 5
HDP = –6 HTP = 9 HTP = –4
(x – 4)[x – (–6)] = 0 [x – (–5)][x – (–7)] = 0 HDP = –20 HDP = –7
(x – 4)(x + 6) = 0 (x + 5)(x + 7) = 0
x2 + 2x – 24 = 0 x2 + 12x + 35 = 0

21 22

G. Selesaikan masalah yang berikut. 2.3 Jenis Punca Persamaan Kuadratik SPM K1 ‘13, ’14 K2 ’15

Solve the following problems. 2.1 ; 2.2


FAKTA UTAMA
5
1. Gunakan hasil tambah dan hasil darab punca-punca untuk menentukan sama ada 2 dan adalah • Jenis punca bagi suatu persamaan kuadratik ax2 + bx + c = 0 boleh • Peta pokok berikut menunjukkan jenis punca:
2 ditentukan dengan nilai b2 – 4ac. The following tree map shows the types of roots:
punca-punca persamaan kuadratik 2x2 + 4x + 5 = 0 atau bukan. The type of roots of a quadratic equation ax2 + bx + c = 0 can be
5 determined by the value of b2 – 4ac. JENIS PUNCA
Use the sum and the product of roots to determine whether 2 and are the roots of the quadratic equation TYPE OF ROOTS
2
2x2 + 4x + 5 = 0. Nilai b2 – 4ac Jenis punca
Value of b2 – 4ac Type of roots

Jika punca-punca persamaan ialah 2 dan 5 , maka


Dua punca yang berbeza b2 – 4ac  0 b2 – 4ac = 0 b2 – 4ac  0
2x2 + 4x + 5 = 0 b2 – 4ac  0 Two different roots
2
5 5 9
x2 + 2x + = 0 HTP = 2 + = ≠ –2 b2 – 4ac = 0
Dua punca yang sama atau satu punca sahaja
2 2 2 Two equal roots or one root only Dua punca yang Dua punca yang
x – (–2)x + 5 = 0
2 berbeza sama Tiada punca nyata
HDP = 2 × 5 = 5 ≠ 5
Tiada punca nyata atau tiada punca
2 b2 – 4ac  0 No real roots or no roots
Two different roots Two equal roots No real roots
2 2
HTP = –2 Oleh kerana HTP dan HDP tidak sama, maka 2 dan 5 2
5 2 A. Cari nilai b – 4ac. Seterusnya, tentukan jenis punca bagi setiap persamaan kuadratik yang berikut.
HDP = bukan punca persamaan kuadratik. Find the value of b2 – 4ac. Hence, determine the type of roots of each of the following quadratic equations. 3.1
2
Persamaan Nilai b2 – 4ac Jenis punca
Equation Value of b2 – 4ac Type of roots
2. Jika punca-punca persamaan kuadratik x2 + 3x + 2 = 0 ialah r1 dan r2 , bentukkan persamaan kuadratik 1. x2 + 5x + 6 = 0 b2 – 4ac = 52 – 4(1)(6) Dua punca yang berbeza
1 1 = 25 – 24
baharu dengan punca-punca dan .
r1 r2 =10
1 1
If the roots of the quadratic equation x2 + 3x + 2 = 0 are r1 and r2 , form a new quadratic equation with and as
r1 r2
the roots.
2. 2x2 – 7x + 8 = 0 b2 – 4ac = (–7)2 – 4(2)(8) Tiada punca nyata atau tiada punca
x2 + 3x + 2 = 0 Persamaan kuadratik baharu ialah = 49 – 64
2 = –15  0
x – (–3)x + 2 = 0 x2 – – 3  x + 1 = 0
r1 + r2 = –3 dan r1 r2 = 2 2 2
x2 + 3 x + 1 = 0
Untuk persamaan kuadratik baharu: 2 2 3. x2 + 6x + 9 = 0 b2 – 4ac = 62 – 4(1)(9) Dua punca yang sama atau
2x2 + 3x + 1 = 0 = 36 – 36 satu punca sahaja
1 + 1 = r2 + r1 = –3
r1 r2 =0
r1 r2 2
1 × 1 = 1 = 1
r1 r2 r1 r2 2
4. –12x + 4x2 = –9 –12x + 4x2 = –9 Dua punca yang sama atau
4x2 – 12x + 9 = 0 satu punca sahaja
2 2
b – 4ac = (–12) – 4(4)(9)
3. Diberi r1 dan r2 ialah punca-punca persamaan kuadratik 2x2 + 8x + 6 = 0. Cari nilai r12 + r22 tanpa = 144 – 144
menyelesaikan persamaan itu. =0
Given r1 and r2 are the roots of the quadratic equation 2x2 + 8x + 6 = 0. Find the value of r12 + r22 without solving
the equation.
KBAT

2x2 + 8x + 6 = 0 (r1 + r2)2 = r12 + r22 + 2r1 r2 5. 7x + 3 = 5x2 7x + 3 = 5x2 Dua punca yang berbeza
x2 + 4x + 3 = 0 (–4)2 = r12 + r22 + 2(3) 5x2 – 7x – 3 = 0
x2 – (–4)x + 3 = 0
r12 + r22 = 16 – 6 b2 – 4ac = (–7)2 – 4(5)(–3)
r1 + r2 = –4 = 10 = 49 + 60
r1 r2 = 3 = 109  0

23 24

6
B. Untuk setiap persamaan kuadratik, cari julat nilai m atau nilai m supaya menghasilkan dua punca berbeza, PRAKTIS FORMATIF Kertas 1 ANALISIS SOALAN SPM
dua punca yang sama atau tiada punca. Subtopik 2013 2014 2015 2016
2.1 S. 4(a) S. 5(a) S. 5 –
For each quadratic equation, find the range of values of m or value of m such that it produces two different roots, two equal
roots or no roots. 3.2 Jawab semua soalan. 2.2 S. 4(b) S. 5(b) – S. 17, S. 25
2.3 S. 4(c) S. 3 – –
Answer all the questions.
Persamaan Dua punca berbeza Dua punca yang sama Tiada punca
Equation Two different roots Two equal roots No roots
1. Diberi bahawa –3 dan m – 2 ialah punca-punca bagi 3. Diberi persamaan kuadratik 2x2 + mx – 20 = 0, dengan
2.1
5.2
persamaan kuadratik x2 – (n + 1)x + 15 = 0, dengan KLON
SPM
keadaan m ialah pemalar, cari nilai m jika
m dan n ialah pemalar. Cari nilai m dan nilai n. ’14 Given a quadratic equation 2x2 + mx – 20 = 0, where m is
CONTOH
b2 – 4ac  0 b2 – 4ac = 0 b2 – 4ac  0 It is given that –3 and m – 2 are the roots of the quadratic a constant, find the value of m if
 2x2 – 12x + m = 0 equation x2 – (n + 1)x + 15 = 0, where m and n are constants. (a) satu daripada punca-punca persamaan itu ialah 4.
(–12)2 – 4(2)(m)  0 144 – 8m = 0 144 – 8m  0 Find the values of m and n. 2.1
5.2 one of the roots of the equation is 4.
144 – 8m  0 –8m = –144 –8m  –144 [3] (b) hasil tambah punca-punca persamaan itu ialah –2.
–8m  –144 m = 18
m  –144 (x + 3)(x + 2 – m) = 0
2.2
5.2 the sum of roots of the equation is –2.
m  –144 –8
x2 + (2 – m)x + 3x + 6 – 3m = 0
[4]
–8 m  18
m  18 x2 + (2 – m + 3)x + 6 – 3m = 0 (a) Gantikan x = 4 ke dalam persamaan
2
x + (5 – m)x + 6 – 3m = 0 2x2 + mx – 20 = 0
2(42) + m(4) – 20 = 0
Bandingkan dengan x2 – (n + 1)x + 15 = 0.
1. mx2 – 2x + 5 = 0 b2 – 4ac  0 b2 – 4ac = 0 b2 – 4ac  0 32 + 4m – 20 = 0
(–2)2 – 4(m)(5)  0 4 – 20m = 0 4 – 20m  0 Maka, 6 – 3m = 15 4m = –12
4 – 20m  0 –20m = –4 –20m  –4 –3m = 9 m = –3
–20m  –4 m = –3
m=
1 m 1 (b) Hasil tambah punca-punca:
5 5
m 1 dan 5 – m = –(n + 1)
– m = –2
5 5 – (–3) = –(n + 1) 2
8 = –(n + 1) m=4
n + 1 = –8
n = –9
2. 3x2 + 6x – m = 0 b2 – 4ac  0 b2 – 4ac = 0 b2 – 4ac  0
62 – 4(3)(–m)  0 36 + 12m = 0 36 + 12m  0
36 + 12m  0 12m = –36 12m  –36
12m  –36 m = –3 m  –3 4. Diberi bahawa persamaan kuadratik 2x2 – 5x + 9
m  –3
KLON
SPM
= 0 mempunyai punca-punca α dan β. Bentukkan
’16
2.2
5.2
persamaan kuadratik dengan punca-punca 2α dan 2β.
It is given that the quadratic equation 2x2 – 5x + 9 = 0 has roots
α and β. Form a quadratic equation with roots 2α and 2β.
[3]
2. Diberi –6 ialah satu daripada punca persamaan 2x2 – 5x + 9 = 0
3. x2 – 5x + 2m = 0 b2 – 4ac  0 b2 – 4ac = 0 b2 – 4ac  0 KLON
SPM
kuadratik (x + k)2 = 25, dengan keadaan k ialah
(–5)2 – 4(1)(2m)  0 25 – 8m = 0 25 – 8m  0 ’15 pemalar. Cari nilai-nilai k. x2 – 5 x + 9 = 0
2.1
5.2
It is given that –6 is one of the roots of the quadratic equation 2 2
25 – 8m  0 –8m = –25 –8m  –25
(x + k)2 = 25, where k is a constant. Find the values of k. Maka, α + β = 5 dan αβ = 9
–8m  –25 m = 25 m  25 [2] 2 2
8
m  25 8
(x + k)2 = 25
Untuk persamaan baharu:
8 HTP: 2α + 2β = 2(α + β)
(–6 + k)2 = 25
–6 + k = ± 5 = 2 5 
k=6±5 2
= 11 atau 1 =5
4. mx2 + 4x – 2 = 0 b2 – 4ac  0 b2 – 4ac = 0 b2 – 4ac  0 HDP: 2α × 2β = 4αβ
42 – 4m(–2)  0 16 + 8m = 0 16 + 8m  0
16 + 8m  0 8m = –16 8m  –16 = 4 9 
2
8m  –16 m = –2 m  –2 = 18
m  –2
Maka, persamaan kuadratik baharu ialah
x2 – 5x + 18 = 0.

25 26

5. Persamaan kuadratik mx2 + (1 – 2m)x + m + 1 = 0 8. Diberi persamaan kuadratik (1 – a)x2 – 3x + 6 = 0 PRAKTIS FORMATIF Kertas 2 ANALISIS SOALAN SPM
2.3
5.2
mempunyai dua punca yang sama. Cari nilai m. KLON
SPM
tidak mempunyai punca. Cari julat nilai a. Subtopik 2013 2014 2015 2016
2
The quadratic equation mx + (1 – 2m)x + m + 1 = 0 has two ’14 Given the quadratic equation (1 – a)x2 – 3x + 6 = 0 has no roots. 2.1 – – – –

equal roots. Find the value of m.


2.3
5.2
Find the range of values of a. Jawab semua soalan. 2.2 – – – –
2.3 – – S. 5 –
[3] [2] Answer all the questions.

b2 – 4ac = 0 b2 – 4ac  0 1. Persamaan kuadratik 3x2 + hx + k = 0, dengan keadaan dan 6k = 2p2 = 2(–6)2 = 72
(1 – 2m)2 – 4(m)(m + 1) = 0 (–3)2 – 4(1 – a)(6)  0 2.2
2.3
5.2
h dan k ialah pemalar, mempunyai punca –2 dan 5. k = 12
1 + 4m2 – 4m – 4m2 – 4m = 0 9 – 24(1 – a)  0 KBAT A quadratic equation 3x2 + hx + k = 0, where h and k are
(b) Punca-punca baharu:
1 – 8m = 0 9  24(1 – a) constants, has roots –2 and 5.
(a) Cari nilai h dan nilai k. p + 2 = –6 + 2 = –4
8m = 1 3
1–a Find the values of h and k. p – 5 = –6 – 5 = –11
8
m= 1 [4] Persamaan kuadratik baharu ialah
8 5
a (b) Seterusnya, cari julat nilai m dengan keadaan (x + 4)(x + 11) = 0
8
3x2 + hx + k = m mempunyai dua punca yang x2 + 15x + 44 = 0
berbeza.
Hence, find the range of the values of m such that 3. Diberi α dan β ialah punca persamaan kuadratik
6. Persamaan kuadratik x(x – 8) = h + 2k, dengan keadaan 9. Rajah di bawah menunjukkan sebuah bingkai gambar 3x2 + hx + k = m has two different roots. KLON x(x – 9) = 2 – 5h, dengan keadaan h ialah pemalar.
SPM
2.3
5.2
h dan k ialah pemalar, mempunyai dua punca yang KLON berbentuk segi empat tepat yang diperbuat daripada [3] ’15 It is given that α and β are the roots of the quadratic equation
SPM
sama. Ungkapkan h dalam sebutan k. ’16 empat kepingan kayu dengan lebar yang sama. Luas (a) (x + 2)(x – 5) = 0 2.3
5.2
x(x – 9) = 2 – 5h, where h is a constant.
The quadratic equation x(x – 8) = h + 2k, where h and k are
2.2
5.2
gambar yang ternampak ialah 96 cm2. x2 – 3x – 10 = 0 (a) Cari julat nilai h jika α ≠ β.
constants, has two equal roots. Express h in terms of k. KBAT The diagram shows a rectangular picture frame which is made 3x2 – 9x – 30 = 0 Find the range of values of h if α ≠ β.
[3] of four pieces of wood with the same width. The area of the Bandingkan dengan 3x2 + hx + k = 0. [3]
β
(b) Diberi α dan
picture that can be seen is 96 cm2.
Maka, h = –9 dan k = –30. ialah punca bagi satu lagi
x(x – 8) = h + 2k 10 cm 2 2
2
x – 8x – (h + 2k) = 0 (b) 3x2 – 9x – 30 = m 2
persamaan 2x + kx + 9 = 0, dengan keadaan k
3x2 – 9x – (30 + m) = 0 ialah pemalar, cari nilai k dan nilai h.
b2 – 4ac = 0 Bagi dua punca yang berbeza, b2 – 4ac  0. β
α
(–8)2 – 4(1)(–h – 2k) = 0 (–9)2 + 4(3)(30 + m)  0 Given
2
and
2
are the roots of another quadratic
64 + 4h + 8k = 0 81 + 12(30 + m)  0 equation 2x2 + kx + 9 = 0, where k is a constant. Find the
16 + h + 2k = 0 14 cm 12(30 + m)  –81 values of k and h.
h = –16 – 2k [4]
30 + m  –6 3
4 (a) x(x – 9) = 2 – 5h
m  –36 3 x2 – 9x – 2 + 5h = 0
7. Diberi bahawa persamaan kuadratik x(x – 6) = 5. 4 b2 – 4ac  0 (Sebab α ≠ β)
KLON
SPM
It is given that the quadratic equation x(x – 6) = 5. Hitung lebar, dalam cm, kepingan kayu itu. (–9)2 – 4(1)(5h – 2)  0
’13 (a) Ungkapkan persamaan itu dalam bentuk Calculate the width, in cm, of the woods.
2. Persamaan kuadratik x2 + 6(3x + k) = 0, dengan k ialah 81 – 20h + 8  0
2.1
5.2
ax2 + bx + c = 0. [3]
pemalar, mempunyai punca p dan 2p, p ≠ 0. –20h  –89
Express the equation in the form ax2 + bx + c = 0. 9
(b) Nyatakan hasil tambah punca bagi persamaan itu. Jumlah luas = 10 cm × 14 cm A quadratic equation x2 + 6(3x + k) = 0, where k is a constant, h4
has roots p and 2p, p ≠ 0. 20
2.2
5.2 State the sum of roots of the equation. = 140 cm2 (b) x2 – 9x + 5h – 2 = 0
(a) Cari nilai p dan nilai k.
(c) Tentukan jenis punca bagi persamaan itu. Luas kepingan kayu = 140 cm2 – 96 cm2 2.3 Find the values of p and k. α+β=9 ...... ➀
2.3
5.2 Determine the type of roots of the equation. = 44 cm2 [5] αβ = 5h – 2 ...... ➁
[4]
Katakan lebar kepingan kayu = x cm (b) Seterusnya, bentukkan persamaan kuadratik yang
2x2 + kx + 9 = 0
2.2
mempunyai punca p + 2 dan p – 5.
(a) x(x – 6) = 5 2x(14 – 2x) + 2(10x) = 44
Hence, form a quadratic equation which has the roots x2 + k x + 9 = 0
x2 – 6x – 5 = 0 x(14 – 2x) + 10x = 22 p + 2 and p – 5.
2 2
14x – 2x2 + 10x = 22 [3] α + β =– k
(b) Hasil tambah punca = 6 2 2 2
2x2 – 24x + 22 = 0 (a) Punca persamaan kuadratik ialah p dan 2p.
(c) b2 – 4ac = (–6)2 – 4(1)(–5) x2 – 12x + 11 = 0 (x – p)(x – 2p) = 0 α + β = –k ...... ➂
= 36 + 20 (x – 1)(x – 11) = 0 x2 – 3px + 2p2 = 0 ...... ➀ α × β = 9
= 56  0 x = 1 atau x = 11 2 2 2
Diberi x2 + 6(3x + k) = 0 αβ = 18 ...... ➃
Abaikan x = 11 kerana ini jawapan mustahil.
Persamaan ini mempunyai dua punca nyata yang x2 + 18x + 6k = 0 ...... ➁ ➀ = ➂: –k = 9
berbeza. Maka, lebar kepingan kayu itu ialah 1 cm. k = –9
Bandingkan ➀ dan ➁.
–3p = 18 ➁ = ➃: 5h – 2 = 18
p = –6 5h = 20
h=4

27 28

7
FOKUS KBAT BAB

3
FUNGSI KUADRATIK
QUADRATIC FUNCTIONS
Kemahiran Kognitif: Menganalisis
Konteks: Penyelesaian Persamaan Kuadratik
Rajah di bawah menunjukkan suatu kawasan kediaman yang berbentuk segi empat tepat. Kawasan A, B, C dan D ialah 3.1 Fungsi Kuadratik dan Grafnya SPM K1 ’13, ’15
tapak perumahan manakala bahagian berlorek mewakili jalan dengan lebar yang sama.
The diagram shows a residential area which has a rectangular shape. The areas, A, B, C and D, are housing sites while the shaded region FAKTA UTAMA
represents roads with the same width.
• Fungsi kuadratik dalam pemboleh ubah x ialah fungsi dengan kuasa tertinggi x ialah 2.
A quadratic function in the variable x is a function with the highest power of x is 2.
• Bentuk am: f(x) = ax2 + bx + c, dengan keadaan a, b dan c ialah pemalar dan a ≠ 0.
A C General form: f(x) = ax2 + bx + c, where a, b dan c are constants and a ≠ 0.
• Bentuk dan kedudukan graf fungsi kuadratik:
80 m Shapes and positions of graphs of quadratic functions:

B D Nilai b2 – 4ac Kedudukan graf f(x)


Value of b2 – 4ac Position of graph of f(x)

150 m a0 a0 • Graf f(x) menyilang paksi-x pada dua titik yang
berlainan.
b2 – 4ac  0
x x Graph of f(x) intersects the x-axis at two different
Jika jumlah luas tapak perumahan ialah 10 656 m2, berapakah lebar jalan itu?
If the total area of the housing sites is 10 656 m2, what is the width of the roads?
Info KBAT points.

Jumlah luas tapak perumahan • Graf f(x) menyentuh paksi-x pada satu titik sahaja.
a0 a0
= Luas segi empat tepat x Graph of f(x) touches the x-axis at only one point.
– Luas kawasan berlorek b2 – 4ac = 0
Total area of the housing sites • Paksi-x ialah tangen kepada graf f(x).
Katakan lebar jalan = x m x The x-axis is a tangent to the graph of f(x).
= Area of a rectangle
– Area of the shaded region
xm a0 a0 • Graf f(x) tidak menyilang atau menyentuh paksi-x.
x Graph of f(x) does not intersect or touch the x-axis.
A C
b2 – 4ac  0
• Graf f(x) berada di atas atau di bawah paksi-x.
x Graph of f(x) is above or below the x-axis.
xm
80 m

B D A. Tentukan sama ada setiap fungsi berikut ialah fungsi kuadratik atau bukan.
Determine whether each of the following functions is a quadratic function. 1.1

150 m CONTOH
(a) f (x) = 3x2 + 2x – 4 (b) f (x) = x –2 + 4x + 6 (c) f (x) = 3x + 2
Luas segi empat tepat – Luas kawasan berlorek = Jumlah luas tapak perumahan
Ya Kuasa tertinggi x Bukan Kuasa tertinggi x Bukan Kuasa tertinggi x
ialah 2. ialah 1. ialah 1.
150(80) – (80x + 150x – x2) = 10 656
12 000 – 80x – 150x + x2 = 10 656
x2 – 230x + 1 344 = 0 1. f (x) = 2x2 + 3x – 1 2. f (x) = –2x2 + 4x + 5 3. f (x) = x3 – 2x + 4
(x – 6)(x – 224) = 0
x = 6 atau x = 224 Ya Ya Bukan
Berdasarkan rajah, x  80. Maka x = 6.

Lebar jalan itu ialah 6 m.


B. Nyatakan nilai-nilai a, b dan c dalam setiap fungsi kuadratik yang berikut.
State the values of a, b and c in each of the following quadratic functions. 1.1

1. f (x) = x2 + x + 1 2. f (x) = 3x2 – 4x + 5 3. f (x) = x(2x – 3)

a = 1, b = 1, c = 1 a = 3, b = –4, c = 5 a = 2, b = –3, c = 0

29 30

C. Berdasarkan graf fungsi kuadratik yang diberi, lengkapkan jadual berikut. E. Cari julat nilai p jika graf fungsi f(x) menyilang paksi-x pada dua titik yang berlainan.
Based on the given graphs of quadratic functions, complete the following table. 1.2 Find the range of the values of p if the graph of function f(x) intersects the x-axis at two different points. 1.4

Graf bagi f (x) Nilai b2 – 4ac Jenis punca bagi f (x) = 0 CONTOH 1. f (x) = x2 – 3x – p
Value of b2 – 4ac
Graph of f(x) Type of roots of f(x) = 0 f (x) = x2 – 4x + p
(–3)2 – 4(1)(–p)  0
CONTOH (–4)2 – 4(1)(p)  0 b2 – 4ac  0 9 + 4p  0
16 – 4p  0 4p  –9
b2 – 4ac  0 f (x) = 0 tiada punca nyata. –4p  –16 p  –2 1
x p4 4
1. f (x) = 0 mempunyai dua punca nyata
x b2 – 4ac  0 2. f (x) = px2 + 6x + 4 3. f(x) = x2 – 6x + p – 1
yang berbeza.
62 – 4(p)(4)  0 (–6)2 – 4(1)(p – 1)  0
2. f (x) = 0 mempunyai dua punca nyata 36 – 16p  0 36 – 4p + 4  0
x b2 – 4ac  0
yang berbeza. –16p  –36 –4p  –40
p21 p  10
3. f (x) = 0 mempunyai dua punca nyata 4
b2 – 4ac = 0
x yang sama.
F. Cari julat nilai p jika graf fungsi f(x) tidak menyilang paksi-x.
4. x Find the range of the values of p if the graph of function f(x) does not intersect the x-axis. 1.4
b2 – 4ac  0 f (x) = 0 tiada punca nyata. CONTOH 1. f (x) = 10x2 + 20x + p
f (x) = px2 + 6x + 5
202 – 4(10)(p)  0
D. Bagi setiap fungsi kuadratik f(x) yang diberikan, tentukan jenis punca bagi f(x) = 0. Seterusnya, lakarkan 62 – 4(p)(5)  0 b2 – 4ac  0 400 – 40p  0
graf f(x) terhadap paksi-x. 36 – 20p  0 –40p  –400
For each given quadratic function f(x), determine the type of the roots of f(x) = 0. Hence, sketch the graph of f(x) with respect to 4 p  10
the x-axis. 1.3
–20p  –36 ⇒ p1
5
Fungsi kuadratik Jenis punca bagi f (x) = 0 Graf bagi f (x) 2. f(x) = px2 + 12x + 12 3. f(x) = 3x2 – 6x + p
Quadratic function Type of roots of f(x) = 0 Graph of f(x)
122 – 4(p)(12)  0 (–6)2 – 4(3)(p)  0
CONTOH b2 – 4ac = (–6)2 – 4(1)(9) a=10 144 – 48p  0 36 – 12p  0
= 36 – 36 –48p  –144 –12p  –36
f (x) = x2 – 6x + 9 =0 p3 p3
Dua punca nyata yang sama x

1. f (x) = x2 + 2x – 3 b2 – 4ac = 22 – 4(1)(–3) a=10 G. Cari nilai-nilai m jika graf fungsi f(x) menyentuh paksi-x pada satu titik sahaja.
= 4 + 12 x
Find the values of m if the graph of function f(x) touches the x-axis at only one point. 1.4
= 16  0
Dua punca nyata yang berbeza. CONTOH 1. f(x) = 3x2 + mx + 3
f (x) = x2 + mx + 4
2. f (x) = x2 – 8x + 16 b2 – 4ac = (–8)2 – 4(1)(16) a=10 m2 – 4(3)(3) = 0
= 64 – 64 m2 – 4(1)(4) = 0 b2 – 4ac = 0 m2 – 36 = 0
=0 m2 – 16 = 0 m2 = 36
Dua punca nyata yang sama.
x m2 = 16 m = ± 36
m = ± 16 = –6 atau 6
3. f (x) = –3x2 + 5x – 6 b2 – 4ac = 52 – 4(–3)(–6) a = –3  0 = –4 atau 4
= 25 – 72 x
= –47  0 2. f (x) = x2 + mx + 3 – m 3. f (x) = 2x2 + (m + 4)x + 8
Tiada punca nyata. m2 – 4(1)(3 – m) = 0 (m + 4)2 – 4(2)(8) = 0
m2 + 4m – 12 = 0 m2 + 8m + 16 – 64 = 0
4. f (x) = –x2 – 4x + 6 b2 – 4ac = (–4)2 – 4(–1)(6) a = –1  0
(m + 6)(m – 2) = 0 m2 + 8m – 48 = 0
= 16 + 24
x m = –6 atau 2 (m + 12)(m – 4) = 0
= 40  0
m = –12 atau 4
Dua punca nyata yang berbeza.

31 32

8
3.2 Nilai Minimum dan Nilai Maksimum Fungsi Kuadratik SPM K1 ‘13, ‘14, ‘15 K2 ’16 C. Selesaikan masalah yang berikut.
Solve the following problems. 2.1
FAKTA UTAMA
CONTOH 1. Fungsi kuadratik f (x) = x2 – 16x + 64 mempunyai
2
Bagi suatu fungsi kuadratik f(x) = a(x + p) + q, (b) nilai maksimum atau nilai minimum bagi f(x) ialah q.
For a quadratic function f(x) = a(x + p)2 + q, the maximum or minimum value of f(x) is q. Diberi fungsi kuadratik f(x) = x2 + 8x + 16 – p titik minimum pada x = p, dengan keadaan p ialah
(a) titik maksimum atau titik minimum bagi graf f(x) ialah (–p, q). (c) persamaan paksi simetrinya ialah x + p = 0 atau x = –p. mempunyai nilai minimum 3. Cari nilai p. pemalar. Cari nilai p.
the maximum or minimum point of the graph of f(x) is (–p, q). the equation of the axis of symmetry is x + p = 0 or x = –p. Given the quadratic function f(x) = x2 + 8x + 16 – p has A quadratic function f(x) = x2 – 16x + 64 has a minimum
a minimum value of 3. Find the value of p. point at x = p, where p is a constant. Find the value of p.

A. Cari setiap yang berikut bagi setiap fungsi kuadratik f(x) yang diberikan. 2
f (x) = x + 8x + 16 – p f(x) = x2 – 16x + 64
Find each of the following for each given quadratic function f(x). 2.1 16 2 16 2
8 2 8 2
= x2 + 8x +   –   + 16 – p = x2 – 16x +   –   + 64
CONTOH 2 2 2 2
1. f (x) = –(x – 5)2 + 7 2
= (x – 8) – 64 + 64
= (x + 4)2 – 16 + 16 – p
f (x) = 2(x + 4)2 + 5 = (x + 4)2 – p = (x – 8)2
(a) Titik maksimum = (5, 7)
(a) Titik minimum = (–4, 5) Diberi nilai minimum f (x) ialah 3. Nilai minimum berlaku apabila x = 8.
(b) Nilai maksimum bagi f (x) = 7
Maka, p = 8.
(b) Nilai minimum bagi f (x) = 5 (c) Paksi simetri: x – 5 = 0 atau x = 5 Maka, –p = 3 ⇒ p = –3
(c) Paksi simetri: x + 4 = 0 atau x = –4
2. Fungsi kuadratik f (x) = x2 + 2x – 5 boleh 3. Fungsi kuadratik f (x) = x2 + 10x – 4 mempunyai
2. f(x) = (x + 3)2 – 8 3. f(x) = –3(x + 6)2 + 4 diungkapkan dalam bentuk f (x) = (x + m)2 – n, titik minimum pada (p, q). Cari
dengan keadaan m dan n ialah pemalar. Cari A quadratic function f(x) = x2 + 10x – 4 has a minimum
(a) Titik minimum = (–3, –8) (a) Titik maksimum = (–6, 4) nilai m dan nilai n. point at (p, q). Find
(b) Nilai minimum bagi f (x) = –8 (b) Nilai maksimum bagi f (x) = 4 A quadratic function f(x) = x2 + 2x – 5 can be expressed in (a) nilai p dan nilai q.
the form f(x) = (x + m)2 – n, where m and n are constants. the values of p and q.
(c) Paksi simetri: x + 3 = 0 atau x = –3 (c) Paksi simetri: x + 6 = 0 atau x = –6 Find the values of m and n. (b) persamaan bagi paksi simetri.
2 the equation of the axis of symmetry.
f (x) = x + 2x – 5
B. Ungkapkan setiap fungsi f(x) dalam bentuk f(x) = a(x + p)2 + q. Seterusnya, nyatakan titik maksimum atau 10 2 10 2
(a) f(x) = x2 + 10x + 
2 2
2 2– 2 2–5
= x2 + 2x + 
2 2
minimum dan persamaan paksi simetri bagi graf f(x). – –4
Express each function f(x) in the form f(x) = a(x + p)2 + q. Hence, state the maximum or minimum point and the equation of the 2
axis of symmetry of graph f(x). 2.1
= (x + 1)2 – 1 – 5 = (x + 5) – 25 – 4
= (x + 1)2 – 6 …… ➀ = (x + 5)2 – 29
CONTOH Titik minimum = (p, q) = (–5, –29)
1. f (x) = x2 – 6x + 17 Bandingkan ➀ dengan f(x) = (x + m)2 – n.
Maka, p = –5 dan q = –29.
f (x) = –2x2 + 8x – 1 Maka, m = 1 dan n = 6.
2
f(x) = (x2 – 6x) + 17 (b) Paksi simetri ialah x + 5 = 0 atau x = –5.
f(x) = –2(x – 4x) – 1 2 2
= x2 – 6x +  6  –  6   + 17
= –2x2 – 4x +  4  –  4   – 1 4. Rajah di bawah menunjukkan graf fungsi kuadratik 5. Fungsi kuadratik f (x) = –(x + b)2 + c mempunyai
2 2
2 2
2 2 = (x – 3)2 – 9 + 17 f (x) = 2(x + p)2 + 3, dengan p ialah pemalar. dua punca pada x = 2 dan x = 6. Jika y = 28 ialah
= –2[(x – 2)2 – 4] – 1 = (x – 3)2 + 8 The diagram shows the graph of a quadratic function garis tangen kepada graf fungsi ini, cari
= –2(x – 2)2 + 8 – 1 f(x) = 2(x + p)2 + 3, where p is a constant. A quadratic function f(x) = – (x + b)2 + c has two roots at
= –2(x – 2)2 + 7 a=10 f (x) Cari x = 2 and x = 6. If y = 28 is a tangent to the graph, find
a = –2  0 Titik minimum = (3, 8) Find (a) titik maksimum graf f (x) ini.
Titik maksimum = (2, 7) Paksi simetri: x – 3 = 0 atau x = 3 (a) nilai p dan nilai q. the maximum point of the graph of f(x).
the values of p and q. (b) nilai b dan nilai c.
Paksi simetri: x – 2 = 0 atau x = 2
(2, q) (b) persamaan paksi simetri. the values of b and c.
the equation of the axis of
2. f (x) = 3x2 + 24x + 50 3. f (x) = –2x2 + 12x – 3 0
x
symmetry. (a) Titik maksimum melalui paksi simetri dan
nilai maksimum f(x) ialah 28.
f(x) = 3(x2 + 8x) + 50 f(x) = –2(x2 – 6x) – 3 (a) Berdasarkan f(x) = 2(x + p)2 + 3,
8 2 8 2 2 2 Maka, titik maksimum = 2 + 6, 28 = (4, 28)
= 3x2 + 8x +   –    + 50 = –2x2 – 6x +  6  –  6   – 3
paksi simetri ialah x = –p.
2
2 2 2 2 Berdasarkan graf, paksi simetri ialah x = 2.
= 3[(x + 4)2 – 16] + 50 = –2[(x – 3)2 – 9] – 3 Maka, –p = 2 (b) Persamaan paksi simetri ialah x + b = 0 atau
= 3(x + 4)2 – 48 + 50 = –2(x – 3)2 + 18 – 3 p = –2 x = –b.
= 3(x + 4)2 + 2 = –2(x – 3)2 + 15 Maka, –b = 4
Berdasarkan f(x) = 2(x + p)2 + 3, b = –4
a=30 a = –2  0 nilai minimum = 3.
Berdasarkan graf, nilai minimum = q. f(x) = –(x + b)2 + c, dengan keadaan c ialah
Titik minimum = (–4, 2) Titik maksimum = (3, 15)
Maka, q = 3. nilai maksimum.
Paksi simetri: x + 4 = 0 atau x = –4 Paksi simetri: x – 3 = 0 atau x = 3 Maka, c = 28.
(b) Paksi simetri ialah x = 2.

33 34

3.3 Lakaran Graf Fungsi Kuadratik SPM K2 ‘16 2. f (x) = –x2 + 4x – 3 untuk/for –1  x  5 3. f (x) = 2x2 + 8x – 10 untuk/for –∞  x  ∞
Lakarkan graf fungsi kuadratik yang berikut. a = –1  0, maka graf berbentuk . a = 2  0, maka graf berbentuk .
Sketch the graphs of the following quadratic functions. 3.1
f(x) = –x2 + 4x – 3 f(x) = 2x2 + 8x – 10
CONTOH
= –(x2 – 4x) – 3 = 2(x2 + 4x) – 10
f (x) = x2 + 6x – 5 untuk/for –6  x  1 2 2 2 2
= –x2 – 4x +  4  –  4   – 3 = 2x2 + 4x +  4  –  4   – 10
2 2 2 2
a = 1  0, maka graf berbentuk . = –[(x – 2)2 – 4] – 3 = 2(x + 2)2 – 8 – 10
= –(x – 2)2 + 1 = 2(x + 2)2 – 18
f (x) = x2 + 6x – 5
2 2 f (x) Maka, titik maksimum ialah (2, 1). Maka, titik minimum ialah (–2, –18).
6 6
= x2 + 6x + 
2 2
– –5 Pintasan-x berlaku apabila f(x) = 0. Pintasan-x berlaku apabila f(x) = 0.
= (x + 3)2 – 9 – 5 (1, 2) –(x – 2)2 + 1 = 0 2(x + 2)2 – 18 = 0
2
= (x + 3) – 14
2 (x – 2)2 = 1 (x + 2)2 = 9
0
x x – 2 = ±1 x + 2 = ± 9 = ±3
Maka, titik minimum ialah (–3, –14). –6 1
x=2±1 x = –2 ± 3
Pintasan-x berlaku apabila f (x) = 0. (–6, –5) = 3 atau 1 = –5 atau 1
–5
(x + 3)2 – 14 = 0 Pintasan-y berlaku apabila x = 0. Pintasan-y berlaku apabila x = 0.
x + 3 = ± 14 f(x) = –02 + 4(0) – 3 = –3 f(x) = 2(0)2 + 8(0) – 10
x = –3 ± 14 Apabila x = –1, = –10
= –6.7 atau 0.7 f(x) = –(–1)2 + 4(–1) – 3 = –8 f (x)
–14
Apabila x = 5,
Pintasan-y berlaku apabila x = 0. (–3, –14) f(x) = –(5)2 + 4(5) – 3 = –8
x
f (x) = 02 + 6(0) – 5 = –5 f(x) –5 0 1

(2, 1)
Apabila x = –6, f (x) = (–6)2 + 6(–6) – 5 = –5
x
0
Apabila x = 1, f (x) = 12 + 6(1) – 5 = 2 1 3
–10
–3

1. f (x) = x2 + 2x – 3 untuk/for –4  x  2
a = 1  0, maka graf berbentuk .
(–1, –8) (5, –8) (–2, –18)
f(x) = x2 + 2x – 3
2 – 2 –3 2 2
= x2 + 2x + 
2 2
f (x)

= (x + 1)2 – 1 – 3 (–4, 5) (2, 5) 3.4 Ketaksamaan Kuadratik SPM K1 ‘14, ‘15, ‘16
2
= (x + 1) – 4
A. Cari julat nilai x untuk setiap ketaksamaan kuadratik yang berikut.
Maka, titik minimum ialah (–1, –4). Find the range of values of x for each of the following quadratic inequalities. 4.1

Pintasan-x berlaku apabila f (x) = 0. CONTOH 1. x2 + 5x + 4  0


(x + 1)2 = 4
2x2 + x – 6  0
x+1=± 4 x2 + 5x + 4  0
x = –1 ± 2 (x + 1)(x + 4)  0
0
x
2x2 + x – 6  0
= –3 atau 1 –3 1
(2x – 3)(x + 2)  0 Katakan (x + 1)(x + 4) = 0.
Pintasan-y berlaku apabila x = 0. x = –1 atau 4
Katakan (2x – 3)(x + 2) = 0
f(x) = 02 + 2(0) – 3 = –3 –3 3
x = atau –2
2 –4 –1
x
Apabila x = –4, f(x) = (–4)2 + 2(–4) – 3 (–1, –4) f(x) 0
= 16 – 8 – 3 x
–2 3
=5 f (x)  0 Untuk x2 + 5x + 4  0, –4  x  –1.
2
Apabila x = 2, f(x) = 22 + 2(2) – 3 3.
=5 Untuk 2x2 + x – 6  0, –2  x 
2

35 36

9
2. 2x2 + x  6 3. x + 4  (x + 4)(2x – 1) PRAKTIS FORMATIF Kertas 1 ANALISIS SOALAN SPM
Subtopik 2013 2014 2015 2016

2x2 + x  6 x + 4  2x2 – x + 8x – 4 3.1 S. 5 – S. 3(b) –


Jawab semua soalan. 3.2 S. 6 S. 4(a),(b) S. 3(a) –
2x2 + x – 6  0 0  2x2 + 6x – 8 Answer all the questions. 3.3 – – – –
(2x – 3)(x + 2)  0 2x2 + 6x – 8  0 3.4 – S. 4(c) S. 4 S. 18

x2 + 3x – 4  0 1. Rajah di bawah menunjukkan graf fungsi kuadratik


Katakan (2x – 3)(x + 2) = 0. 3. Graf fungsi kuadratik f(x) = px2 – 4x + 2, dengan
(x + 4)(x – 1)  0 3.1
5.2
y = f(x).
3 KLONkeadaan p ialah pemalar, tidak bersilang dengan
x = atau –2 Katakan (x + 4)(x – 1) = 0. The diagram shows the graph of a quadratic function y = f(x). SPM
2 ’13paksi-x. Cari julat nilai p.
x = –4 atau 1 y 3.1
5.2
The graph of a quadratic function f(x) = px2 – 4x + 2, where
f(x) 0 p is a constant, does not intersect the x-axis. Find the range
x f(x)0 5
–2 3 x of the values of p.
2 –4 1 y = f(x) [3]
3 b 2 – 4ac  0
Untuk 2x2 + x  6, x  –2 atau x  .
2 Untuk x + 4  (x + 4)(2x – 1), –4 0 2
x (–4)2 – 4(p)(2)  0
x  –4 atau x  1 16 – 8p  0
Nyatakan 8p  16
State
p2
(a) punca-punca bagi persamaan f(x) = 0.
the roots of the equation f(x) = 0.
B. Selesaikan masalah yang berikut.
Solve the following problems. 4.1
(b) persamaan paksi simetri bagi lengkung itu.
the equation of the axis of symmetry of the curve.
[3] 4. Rajah di bawah menunjukkan graf fungsi kuadratik
1. (a) Cari julat nilai x jika 4x  x2. 2. (a) Diberi f (x) = 4x2 – 16, cari julat nilai x dengan KLONf(x) = (x – 3)2 – 2k, dengan keadaan k ialah pemalar.
Find the range of values of x if 4x  x . 2
keadaan f (x) sentiasa positif. (a) Punca-punca persamaan ialah –4 dan 2. SPM
The diagram shows the graph of a quadratic function
’13
2
(b) Cari julat nilai p jika x – (p + 1)x – p + 1 = 0 2 Given f(x) = 4x2 – 16, find the range of values of x 3.2
5.2 f(x) = (x – 3)2 – 2k, where k is a constant.
–2
mempunyai dua punca nyata. such that f(x) is always positive. (b) x = –4 + 2 = = –1
2 2 y
Find the range of values of p if x2 – (p + 1)x – p2 + 1 = 0 (b) Cari julat nilai x yang memuaskan ketaksamaan
has two real roots. (x – 3)2  x – 3. Persamaan paksi simetri ialah x = –1.
Find the range of values of x which satisfies the p
(a) 4x  x2 inequality (x – 3)2  x – 3.
x2 – 4x  0 2. Rajah di bawah menunjukkan graf fungsi kuadratik O
x
x(x – 4)  0 (a) Untuk f (x) sentiasa positif, f(x)  0. f(x) = (x + 5)2 + 3k – 4, dengan k ialah pemalar.
Maka, 4x2 – 16  0 The diagram shows the graph of a quadratic function
(h, –6)
Katakan x(x – 4) = 0. x2 – 4  0 f(x) = (x + 5)2 + 3k – 4, where k is a constant.
x = 0 atau 4 (x + 2)(x – 2)  0 f (x) Diberi (h, –6) ialah titik minimum graf itu.
f(x) = (x + 5)2 + 3k – 4 Given (h, –6) is the minimum point of the graph.
f(x) 0
x f(x) 0 (a) Nyatakan nilai h dan nilai k.
0 4 x
–2 2 State the values of h and k.
(b) Cari nilai p.
11
Untuk 4x  x2, x  0 atau x  4. Find the value of p.
Untuk 4x – 16  0,
2
0
x [4]
(b) x2 – (p + 1)x – p2 + 1 = 0 x  –2 atau x  2
(a) Nyatakan persamaan paksi simetri bagi lengkung (a) Graf fungsi f (x) = (x – 3)2 – 2k mempunyai titik
b2 – 4ac  0 minimum (3, –2k).
[–(p + 1)]2 – 4(1)(–p2 + 1)  0 (b) (x – 3)2  x – 3 3.1
itu.
Maka, (3, –2k) = (h, –6)
p2 + 2p + 1 + 4p2 – 4  0 x2 – 6x + 9  x – 3 State the equation of the axis of symmetry of the curve.
x2 – 7x + 12  0 (b) Diberi nilai minimum bagi fungsi itu ialah 11. Jadi, h = 3 dan –2k = –6
5p2 + 2p – 3  0 3.2
Cari nilai k.
(5p – 3)(p + 1)  0 (x – 3)(x – 4)  0 Given the minimum value of the function is 11. Find the
k=3
Katakan (5p – 3)(p + 1) = 0. value of k. (b) f (x) = (x – 3)2 – 6
[3]
p = 3 atau –1
x
3 4 Apabila x = 0, f (x) = (0 – 3)2 – 6
5 f(x) 0 (a) Apabila x + 5 = 0 =3
f(p) 0
x = –5 Maka, p = 3
p
Untuk (x – 3)2  x – 3, Persamaan paksi simetri ialah x = –5.
–1 3
5 3x4 (b) Nilai minimum f(x) ialah 3k – 4.
Maka, 3k – 4 = 11
Maka, p  –1 atau p  3 3k = 15
5 k=5

37 38

5. Graf fungsi kuadratik f (x) = px2 – 4x + q, dengan 7. Cari julat nilai x untuk (x – 2)2  14 – x. PRAKTIS FORMATIF Kertas 2 ANALISIS SOALAN SPM
KLON
SPM
keadaan p dan q ialah pemalar, mempunyai satu 3.4
5.2 Find the range of values of x for (x – 2)2  14 – x. Subtopik 2013 2014 2015 2016
’15 titik maksimum. [3] 3.1 – – – –

The graph of a quadratic function f(x) = px2 – 4x + q, where (x – 2)  14 – x


2 Jawab semua soalan. 3.2 – – – S. 2(a),(c)
Answer all the questions. 3.3 – – – S. 2(b)
p dan q are constants, has a maximum point. x 2 – 4x + 4  14 – x 3.4 – – – –
(a) Diberi p ialah suatu integer, nyatakan nilai p yang x 2 – 3x – 10  0
1. Lengkung fungsi kuadratik f (x) = 2(x – h)2 – 2k
3.2 maksimum. (x + 2)(x – 5)  0 x 2. Persamaan kuadratik x2 – 9x + 8 = 0 mempunyai
–2 5 KLON menyilang paksi-x pada titik-titik (2, 0) dan (4, 0).
Given p is an integer, state the maximum value of p. –2  x  5 SPM
punca-punca m dan n, dengan keadaan m  n.
(b) Dengan menggunakan jawapan dari (a), cari ’16 Garis lurus y = –2 adalah tangen kepada bucu
The quadratic equation x2 – 9x + 8 = 0 has roots m and n,
nilai q apabila graf itu menyentuh paksi-x pada lengkung itu.
3.1 where m  n.
The curve of a quadratic function f(x) = 2(x – h)2 – 2k intersects
suatu titik sahaja. the x-axis at points (2, 0) and (4, 0). The straight line y = –2 (a) Cari
Using the answer from (a), find the value of q when the Find
is the tangent to the vertex of the curve.
graph touches the x-axis at only one point. (i) nilai m dan nilai n.
8. Diberi f (x) = –2x2 – x + 13, cari julat nilai x untuk
[3] 3.4
5.2
f (x)  –2. (a) Cari nilai h dan nilai k. 2.3 the values of m and n.
(a) Fungsi itu mempunyai titik maksimum, Given that f(x) = –2x2 – x + 13, find the range of values of x
3.2 Find the values of h and k. (ii) julat nilai x jika x2 – 9x + 8  0.
maka p  0. for f(x)  –2.
[2] 3.4 the range of values of x if x2 – 9x + 8  0.
Jika p ialah integer, maka nilai maksimum p [3] (b) Seterusnya, lakar graf f (x) untuk 1  x  5. [5]
ialah –1. f (x)  –2 3.3 Hence, sketch the graph of f(x) for 1  x  5. (b) Dengan menggunakan nilai m dan nilai n dari
–2x2 – x + 13  –2 [3] 2.2
(a)(i), bentukkan persamaan kuadratik yang
(b) f(x) = –x2 – 4x + q 2x2 + x – 15  0
(c) Jika graph itu dipantulkan pada paksi-x, tulis mempunyai punca-punca m – 2 dan 2n – 3.
b2 – 4ac = 0 jika graf fungsi itu menyentuh paksi-x (2x – 5)(x + 3)  0 x Using the values of m and n from (a)(i), form the quadratic
–3 5 3.2 persamaan bagi lengkung itu.
pada satu titik sahaja.
x  –3 atau x  5 2 If the graph is reflected about the x-axis, write the equation equation which has the roots m – 2 and 2n – 3.
2 [2]
(–4)2 – 4(–1)q = 0 of the curve.
16 + 4q = 0 [1]
(a) (i) x 2 – 9x + 8 = 0
4q = –16
(a) Lengkung fungsi itu menyilang paksi-x pada (2, 0) (x – 1)(x – 8) = 0
q = –4
9. Cari julat nilai x untuk 3x2 + 11x  4. dan (4, 0). x = 1 atau x = 8
KLON Find the range of values of x for 3x2 + 11x  4. Maka, titik minimum lengkung terletak pada
SPM Oleh sebab m  n, maka m = 8 dan n = 1.
’15 [2]
6. Rajah di bawah menunjukkan graf fungsi kuadratik
3.4
5.2
3x2 + 11x  4 x = 2 + 4 = 3. (ii) x 2 – 9x + 8  0
2
KLON f(x) = (x – 2)2 – 16. 3x + 11x – 4  0
2 (x – 1)(x – 8)  0
SPM
’14 The diagram shows the graph of the quadratic function (3x – 1)(x + 4)  0 x Juga diberi y = –2 ialah garis tangen kepada bucu x  1 atau x  8
–4 1 lengkung. Ini bermakna titik (3, –2) ialah bucu
f(x) = (x – 2)2 – 16.
–4  x  1 3
lengkung itu.
y 3 1 8 x
Daripada fungsi f(x) = 2(x – h)2 – 2k, koordinat
bucu lengkung ialah (h, –2k).
x (b) Punca-punca baharu:
–2 O 6 Maka, h = 3 dan k = 1.
m–2=8–2=6
10. Diberi fungsi kuadratik f (x) = x2 + wx + 2w – 3,
KLON dengan keadaan w ialah pemalar, adalah sentiasa
(b) f(x) = 2(x – 3)2 – 2 2n – 3 = 2(1) – 3 = –1
SPM f(1) = 2(1 – 3)2 – 2 = 6
’16 positif apabila p  w  q. Cari nilai p dan nilai q.
Nyatakan 3.4
5.2
Given a quadratic function f(x) = x2 + wx + 2w – 3, where
f(5) = 2(5 – 3)2 – 2 = 6 Persamaan kuadratik baharu ialah
State w is a constant, is always positive when p  w  q. Find the
(x – 6)(x + 1) = 0
Lengkung itu melalui titik (1, 6) dan (5, 6). x 2 – 5x – 6 = 0
(a) koordinat titik minimum bagi lengkung itu. values of p and q.
3.2 the coordinates of the minimum point of the curve. f(x)
[3]
(b) persamaan paksi simetri bagi lengkung itu. (1, 6) (5, 6)
3.2 the equation of the axis of symmetry of the curve.
f(x) = x2 + wx + 2w – 3
(c) julat nilai x apabila f(x) ialah negatif.
3.4 the range of values of x when f(x) is negative. a = 1, b = w, c = 2w – 3
[3] b2 – 4ac  0 x
(a) (2, –16) w2 – 4(1)(2w – 3)  0 O

(b) x = 2 atau x – 2 = 0 w2 – 8w + 12  0 (3, –2)

(c) –2  x  6 (w – 2)(w – 6)  0
2w6 (c) Persamaan baharu bagi lengkung ialah
f(x) = –[2(x – 3)2 – 2]
Maka, p = 2 dan q = 6.
f(x) = –2(x – 3)2 + 2

39 40

10
FOKUS KBAT BAB

4
PERSAMAAN SERENTAK
SIMULTANEOUS EQUATIONS
Kemahiran Kognitif: Mengaplikasi dan Mencipta
Konteks: Fungsi Kuadratik dan Grafnya, Nilai Maksimun
Rajah di bawah menunjukkan pandangan sisi sebuah jambatan. Bahagian melengkung di bawah jambatan itu disokong 4.1 Penyelesaian Persamaan Serentak dalam Dua Anu:
1 Satu Persamaan Linear dan Satu Persamaan Tak Linear SPM K2 ‘13, ’14, ’15, ’16
oleh lima batang tiang keluli. Tinggi, dalam m, setiap tiang keluli itu diwakili oleh fungsi h(x) = ax2 + x + k, dengan
2 A. Selesaikan persamaan serentak yang berikut.
keadaan x ialah jarak sesuatu tiang dari tiang pertama, dan a dan k ialah pemalar. Solve the following simultaneous equations. 1.1
The diagram shows the side view of a bridge. The curved part at the bottom of the bridge is supported by five steel poles. The height,
in m, of each steel pole is represented by a function h(x) = ax2 +
1
x + k, where x is the distance of a pole from the first pole, and CONTOH 1. 2x + y = 6 …… ➀
2
a and k are constants. x – 3y + 7 = 0 …… ➀ Persamaan linear x2 – 3y2 + 4x + 3 = 0 …… ➁
200 m x2 + y2 – 4xy – 13 = 0 …… ➁ Persamaan tak linear Dari ➀: y = 6 – 2x …… ➂
Dari ➀: x = 3y – 7 …… ➂ Ungkapkan x dalam Gantikan ➂ ke dalam ➁.
sebutan y. x2 – 3(6 – 2x)2 + 4x + 3 = 0
Gantikan ➂ ke dalam ➁.
(3y – 7)2 + y2 – 4(3y – 7)y – 13 = 0 x2 – 108 + 72x – 12x2 + 4x + 3 = 0
2 2 2
9y – 42y + 49 + y – 12y + 28y – 13 = 0 11x2 – 76x + 105 = 0
5m –2y2 – 14y + 36 = 0 (11x – 21)(x – 5) = 0
Permudahkan y2 + 7y – 18 = 0 x = 21 atau 5
Diberi panjang jambatan itu ialah 200 m dan panjang tiang pertama ialah 5 m. Hitung panjang bagi tiang yang persamaan dan (y – 2)(y + 9) = 0 11
paling tinggi.
faktorkan.
y = 2 atau –9 Gantikan nilai-nilai x ke dalam ➂.
Given the length of the bridge is 200 m and the length of the first pole is 5 m. Find the length of the highest pole.
Gantikan nilai-nilai y ke dalam ➂. Apabila x = 21 , y = 6 – 2 21  = 24
11 11 11
h(x) = ax2 + 1 x + k
Info KBAT Apabila y = 2, x = 3(2) – 7 = –1 Apabila x = 5, y = 6 – 2(5) = –4
2 Apabila y = –9, x = 3(–9) – 7 = –34 10 2
Anggapkan hujung atas tiang pertama Penyelesaian ialah x = 1 , y = 2 dan
Pada titik (0, 5): 5 = a(02) + 1 (0) + k sebagai titik (0, 5) yang terletak pada Penyelesaian ialah x = –1, y = 2 dan x = –34, y = –9. 11 11
2 fungsi h(x). x = 5, y = –4.
k=5 Assume that the upper end of the first pole is
at point (0, 5) that lies on the function h(x). 2. 2x – 5y + 16 = 0 …… ➀ y
Maka, h(x) = ax2 + 1 x + 5 3. 2x – =1 …… ➀
2 3xy – 4x – 24y = 10 …… ➁ 3
5y – 16 4x2 + y2 – 10xy = 7 …… ➁

= a x2 + 1 x + 5
2a  h(x)
h(x) = ax2 +
1
x+k
Dari ➀: x =
2
…… ➂
Dari ➀: y = 6x – 3 …… ➂
2 2
2 Gantikan ➂ ke dalam ➁.

= a x2 + 1 x + 1 – 1
2a 4a 4a    +5 3 5y – 16 y – 4 5y – 16  – 24y = 10
Gantikan ➂ ke dalam ➁.
4x2 + (6x – 3)2 – 10x(6x – 3) = 7
2 (0, 5) (200, 5) 2 2
4x2 + 36x2 – 36x + 9 – 60x2 + 30x = 7

=a x+ 1 – 12 +5
4a 16a  O
x 15 y2 – 24y – 10y + 32 – 24y = 10
–20x2 – 6x + 2 = 0
2
2 10x2 + 3x – 1 = 0

=a x+ 1 – 1 +5
4a 16a 15y2 – 116y + 44 = 0
(3y – 22)(5y – 2) = 0 (2x + 1)(5x – 1) = 0

y = 22 atau 2 x = – 1 atau 1
Pada titik maksimum, x = 0 + 200 = 100 3 5 2 5
2
Maka, 1 = −100 Gantikan nilai-nilai y ke dalam ➂. Gantikan nilai-nilai x ke dalam ➂.
4a 5 22  – 16 Apabila x = – 1 , y = 6– 1  – 3 = –6
Apabila y = 22 , x = 3 = 31
a=− 1 2 2
400 3 2 3
Apabila x = 1 , y = 6 1  – 3 = – 9
1 5 2  – 16 5 5 5
Panjang tiang paling tinggi = – +5 Apabila y = 2 , x= 5 = –7
16 – 1  5 2 Penyelesaian ialah x = – 1 , y = –6 dan
400 2
= 25 + 5 Penyelesaian ialah x = 10 1 , y = 7 1 dan x = 1 , y = –1 4 .
3 3 5 5
= 30 m
x = –7, y = 2 .
5

41 42

B. Selesaikan persamaan serentak yang berikut. Berikan jawapan betul kepada tiga tempat perpuluhan. C. Selesaikan masalah yang berikut.
Solve each of the following simultaneous equations. Give the answers correct to three decimal places. 1.1 Solve the following problems. 1.2

CONTOH 1. David mempunyai sebidang tanah yang berbentuk segi tiga bersudut tegak. Sempadan paling panjang tanah
1. x + 3y + 1 = 0 …… ➀
itu ialah 2y m. Dua sempadan lagi masing-masing ialah x m dan (3x – 6) m. Dia menggunakan dawai pagar
x 5
+ =2 …… ➀ Persamaan tak linear x2 – 10y2 + xy + 4 = 0 …… ➁
y x sepanjang 60 m untuk memagari tanah itu. Cari panjang, dalam m, setiap sempadan tanah itu.
y Dari ➀: x = –3y – 1 …… ➂ David had a piece of right-angled triangle shaped land. The longest side of the land is 2y m. The other two sides are x m and
x– =3 …… ➁ Persamaan linear (3x – 6) m respectively. He fenced off the land with 60 m of fencing wire. Find the length, in m, of each side of the land.
2 Gantikan ➂ ke dalam ➁.
2
Dari ➀: x + 5y = 2 (–3y – 1)2 – 10y2 + (–3y – 1)y + 4 = 0
xy 9y2 + 6y + 1 – 10y2 – 3y2 – y + 4 = 0 Perimeter tanah = 60 m
x2 + 5y = 2xy …… ➂ –4y2 + 5y + 5 = 0 2y + x + (3x – 6) = 60
Dari ➁: y = 2x – 6 …… ➃ Apabila jawapan
2y + 4x = 66 2y m
xm
y = –5 ± 5 – 4(–4)(5)
2 y + 2x = 33
dikehendaki dalam
Gantikan ➃ ke dalam ➂. perpuluhan, rumus 2(–4) y = 33 – 2x …… ➀
x2 + 5(2x – 6) = 2x(2x – 6) x = –b ±
b2 – 4ac (3x – 6) m
x2 + 10x – 30 = 4x2 – 12x 2a = –5 ± 105 Berdasarkan teorem Pythagoras,
2
3x – 22x + 30 = 0
digunakan. –8 x2 + (3x – 6)2 = (2y)2 …… ➁
= –0.656 atau 1.906
–(–22) ± (–22)2 – 4(3)(30) Gantikan ➀ ke dalam ➁.
x= = 22 ± 124 Gantikan nilai-nilai y ke dalam ➂.
2(3) 6 x2 + (3x – 6)2 = 4(33 – 2x)2 Tetapi x  60.
= 5.523 atau 1.811 Apabila y = –0.656, x = –3(–0.656) – 1 = 0.968 x2 + 9x2 – 36x + 36 = 4 356 – 528x + 16x2 Maka, x = 10
2
Gantikan nilai-nilai x ke dalam ➃. Apabila y = 1.906, x = –3(1.906) – 1 = –6.718 6x – 492x + 4 320 = 0 2y = 2[33 – 2(10)] = 26
x2 – 82x + 720 = 0 3x – 6 = 3(10) – 6 = 24
Apabila x = 5.523, y = 2(5.523) – 6 = 5.046 Penyelesaian ialah x = 0.968, y = –0.656 dan (x – 10)(x – 72) = 0
Apabila x = 1.811, y = 2(1.811) – 6 = –2.378 x = –6.718, y = 1.906. x = 10 atau 72 Panjang sempadan tanah itu ialah 10 m, 24 m dan 26 m.
Penyelesaian ialah x = 5.523, y = 5.046 dan
x = 1.811, y = –2.378.
2. Rajah di sebelah menunjukkan sebuah rangka meja yang dibina
2. 3 + 1 = 4 …… ➀ 3.
3x + 4y = 5x2
– 4y = x + 1 daripada lapan batang rod keluli. Tinggi meja itu ialah x cm dan luas
y x 2 3
y – 2x = 3 …… ➁ permukaan atas meja ialah 4 000 cm2. Panjang permukaan atas meja
3x + 4y = x + 1 …… ➀ melebihi tingginya sebanyak 20 cm. Diberi jumlah panjang rod keluli
2 3 Pisahkan kepada
3x + y dua persamaan. yang digunakan ialah 500 cm. Cari ukuran, dalam cm, panjang dan
Dari ➀: =4 5x2 – 4y = x + 1 …… ➁ lebar permukaan atas meja itu.
xy
…… ➂ 9x + 8y The diagram shows the frame of a table which is constructed from eight steel x cm
3x + y = 4xy Dari ➀: =x+1 rods. The height of the table is x cm and the area of the table top is 4 000 cm2.
6
Dari ➁: y = 2x + 3 …… ➃ The length of the table top exceeds its height by 20 cm. Given the total length
y = 6 – 3x …… ➂ of steel rods used is 500 cm. Find the measurements, in cm, of the length and
Gantikan ➃ ke dalam ➂. 8
width of the table top.
3x + (2x + 3) = 4x(2x + 3) Gantikan ➂ ke dalam ➁.
5x + 3 = 8x2 + 12x 5x – 4 
8 
2 6 – 3x =x+1
8x2 + 7x – 3 = 0 Katakan: Gantikan ➀ ke dalam ➁.
5x2 – 3 + 3 x = x + 1 Lebar permukaan atas meja = y cm (x + 20)(230 – 3x) = 4 000
x = –7 ± 7 – 4(8)(–3)
2
2 Panjang permukaan atas meja = (x + 20) cm 230x – 3x2 + 4 600 – 60x = 4 000
2(8) 10x2 + x – 8 = 0 –3x2 + 170x + 600 = 0
= –7 ± 145 Jumlah panjang rod keluli = 500 cm 3x2 – 170x – 600 = 0
x = –1 ±
12 – 4(10)(–8) –1 ± 321
16 = 2y + 2(x + 20) + 4x = 500 (3x + 10)(x – 60) = 0
2(10) 20 2y + 2x + 40 + 4x = 500
= 0.315 atau –1.190 x = – 10 atau 60
= 0.846 atau –0.946 2y + 6x = 460 3
Gantikan nilai-nilai x ke dalam ➃. Tetapi x  0.
Gantikan nilai-nilai x ke dalam ➂. y + 3x = 230
Apabila x = 0.315, y = 2(0.315) + 3 = 3.630 y = 230 – 3x …… ➀ Maka, x = 60
Apabila x = –1.190, y = 2(–1.190) + 3 = 0.620 Apabila x = 0.846, y = 6 – 3(0.846) = 0.433 x + 20 = 60 + 20 = 80
8
Luas permukaan atas meja = 4 000 cm2 y = 230 – 3(60) = 50
Penyelesaian ialah x = 0.315, y = 3.630 dan Apabila x = –0.946, y = 6 – 3(–0.946) = 1.105 (x + 20)y = 4 000 …… ➁
x = –1.190, y = 0.620. 8 Maka, panjang permukaan atas meja ialah 80 cm dan
Penyelesaian ialah x = 0.846, y = 0.433 dan lebarnya ialah 50 cm.
x = –0.946, y = 1.105.

43 44

11
5. Selesaikan persamaan serentak yang berikut: 7. Seutas dawai yang panjangnya 30 cm dibengkokkan
PRAKTIS FORMATIF Kertas 2 ANALISIS SOALAN SPM
KLON Solve the following simultaneous equations: KLON untuk menjadi sebuah segi tiga bersudut tegak. Sisi
Subtopik 2013 2014 2015 2016 SPM SPM
4.1 S. 1 S. 1 S. 1 S. 3 ’15
3x – y + 5 = 0
’16terpanjang segi tiga itu ialah y cm dan dua sisi yang
Jawab semua soalan. 4.1
5.2 4.1
5.2
lain masing-masing ialah x cm dan 3(x – 1) cm. Cari
Answer all the questions. 4x2 + y2 – 2xy = 12
[5] panjang, dalam cm, setiap sisi segi tiga itu.
A piece of wire of length 30 cm is bent to form a right-angled
1. Selesaikan persamaan serentak x – 2y – 1 = 0 dan 3. Selesaikan persamaan serentak yang berikut: 3x – y + 5 = 0 triangle. The longest side of the triangle is y cm and the other
4.1
5.2
2x2 + 3y 2 – 6xy = –4. Berikan jawapan betul kepada KLON
SPM
Solve the following simultaneous equations: y = 3x + 5 ...... ➀ two sides are x cm and 3(x – 1) cm respectively. Find the length,
tiga tempat perpuluhan. ’13
4.1
5.2 3x + y = 9 4x2 + y2 – 2xy = 12 ...... ➁ in cm, of each side of the triangle.
Solve the simultaneous equations x – 2y – 1 = 0 and 2x2 – xy – y = –1 [7]
2x2 + 3y2 – 6xy = –4. Give the answers correct to three Gantikan ➀ ke dalam ➁.
[5]
decimal places.
3x + y = 9 4x2 + (3x + 5)2 – 2x(3x + 5) = 12
[5] 4x2 + 9x2 + 30x + 25 – 6x2 – 10x – 12 = 0
x – 2y – 1 = 0 y = 9 – 3x ...... ➀ y cm
x cm
2x2 – xy – y = –1 ...... ➁ 7x2 + 20x + 13 = 0
x = 1 + 2y ...... ➀ (7x + 13)(x + 1) = 0
2x 2 + 3y 2 – 6xy = –4 ...... ➁ Gantikan ➀ ke dalam ➁. 3(x – 1) cm
2x2 – x(9 – 3x) – (9 – 3x) = –1 x = – 13 atau –1
Gantikan ➀ ke dalam ➁. 7
2x2 – 9x + 3x2 – 9 + 3x = –1 Perimeter segi tiga = 30 cm
2(1 + 2y)2 + 3y 2 – 6(1 + 2y)y = –4 4
2(1 + 4y + 4y 2) + 3y 2 – 6(y + 2y 2) = –4 5x2 – 6x – 8 = 0
(5x + 4)(x – 2) = 0 7 
7 
Apabila x = – 13 , y = 3 – 13 + 5 = –
7
Maka, x + 3(x – 1) + y = 30
x + 3x – 3 + y = 30
2 + 8y + 8y 2 + 3y 2 – 6y – 12y 2 + 4 = 0
y = 33 – 4x …… ➀
–y 2 + 2y + 6 = 0 x = – 4 atau 2 Apabila x = –1, y = 3(–1) + 5 = 2
y 2 – 2y – 6 = 0 5 Berdasarkan teorem Pythagoras,
Apabila x = – 4 , y = 9 – 3– 4  = 11 2 Penyelesaian ialah x = –1 6 , y = – 4 dan x2 + [3(x – 1)]2 = y2
(–2)2 – 4(1)(–6) 7 7
y = –(–2) ± 5 5 5
x = –1, y = 2. x2 + 9(x2 – 2x + 1) = y2
2(1) Apabila x = 2, y = 9 – 3(2) = 3 10x2 – 18x + 9 = y2 …… ➁
2 ± 28
= Gantikan ➀ ke dalam ➁.
2 Penyelesaian ialah x = – 4 , y = 11 2 dan x = 2, y = 3.
= 3.646 atau –1.646 5 5 10x2 – 18x + 9 = (33 – 4x)2
10x2 – 18x + 9 = 1 089 – 264x + 16x2
Apabila y = 3.646, 6. Selesaikan persamaan serentak y – 2x = 8 dan 6x2 – 246x + 1 080 = 0
4.1
5.2
x = 1 + 2(3.646) = 8.292 4. Selesaikan persamaan serentak yang berikut: y – xy = 7x. Berikan jawapan betul kepada dua x2 – 41x + 180 = 0
Apabila y = –1.646, KLON
SPM
Solve the following simultaneous equations: tempat perpuluhan. (x – 5)(x – 36) = 0
’14 Solve the simultaneous equations y – 2x = 8 and y – xy = 7x.
x = 1 + 2(–1.646) = –2.292 4.1
5.2 2x – y – 4 = 0 x = 5 atau 36
Give the answers correct to two decimal places.
Penyelesaian ialah x = 8.292, y = 3.646 dan x2 – 2y2 – 3y + 1 = 0 [5] Tetapi x  30.
x = –2.292, y = –1.646. Beri jawapan betul kepada tiga tempat perpuluhan. y – 2x = 8 Maka, x = 5
Give the answers correct to three decimal places. y = 2x + 8 ...... ➀
2. Selesaikan persamaan serentak yang berikut: [5] y – xy = 7x ...... ➁ 3(x – 1) = 3(5 – 1) = 12
4.1
5.2 Solve the following simultaneous equations: 2x – y – 4 = 0 y = 33 – 4(5) = 13
Gantikan ➀ ke dalam ➁.
3x + y = 4 y = 2x – 4 ...... ➀ (2x + 8) – x(2x + 8) = 7x Maka, panjang sisi segi tiga itu ialah 5 cm, 12 cm
x2 + 3y 2 + 5xy – 9 = 0 x2 – 2y2 – 3y + 1 = 0 ...... ➁ 2x + 8 – 2x2 – 8x = 7x dan 13 cm.
[5] Gantikan ➀ ke dalam ➁. 2x2 + 13x – 8 = 0
3x + y = 4 x2 – 2(2x – 4)2 – 3(2x – 4) + 1 = 0 2
y = 4 – 3x ...... ➀ x2 – 2(4x2 – 16x + 16) – (6x – 12) + 1 = 0 x = –13 ± 13 – 4(2)(–8)
x2 + 3y2 + 5xy – 9 = 0 ...... ➁ x2 – 8x2 + 32x – 32 – 6x + 12 + 1 = 0 2(2)
Gantikan ➀ ke dalam ➁. –7x2 + 26x – 19 = 0 –13 ± 233
=
x2 + 3(4 – 3x)2 + 5x(4 – 3x) – 9 = 0 7x2 – 26x + 19 = 0 4
x2 + 3(16 – 24x + 9x2) + 20x – 15x2 – 9 = 0 (7x – 19)(x – 1) = 0 = 0.5661 atau –7.0661
13x2 – 52x + 39 = 0 x = –2.7143 atau 1 Apabila x = 0.5661,
x2 – 4x + 3 = 0 Apabila x = 2.7143, y = 2(0.5661) + 8 = 9.1322
(x – 3)(x – 1) = 0 y = 2(2.7143) – 4 = 1.4286
x = 3 atau 1 Apabila x = –7.0661,
Apabila x = 1, y = 2(–7.0661) + 8 = –6.1322
Apabila x = 3, y = 4 – 3(3) = –5 y = 2(1) – 4 = –2 Penyelesaian ialah x = 0.57, y = 9.13 dan
Apabila x = 1, y = 4 – 3(1) = 1 x = –7.07, y = –6.13.
Penyelesaian ialah x = 2.714, y = 1.429 dan
Penyelesaian ialah x = 3, y = –5 dan x = 1, y = 1. x = 1, y = –2.

45 46

FOKUS KBAT BAB

5
INDEKS DAN LOGARITMA
INDICES AND LOGARITHMS
Kemahiran Kognitif: Mengaplikasi dan Menganalisis
Konteks: Penyelesaian Persamaan Serentak, Ketaksamaan
Rajah di bawah menunjukkan sebuah kolam renang. Lantai di sekeliling kolam renang itu ditutupi dengan jubin. 5.1 Indeks dan Hukum Indeks SPM K1 ‘14, ‘15 K2 ’14

The diagram shows a swimming pool. The floor that surrounds the swimming pool is covered with tiles.
A. Permudahkan setiap yang berikut.
Simplify each of the following. 1.1 ; 1.2
1m
CONTOH FAKTA UTAMA
(a) x7 × x3 ÷ x2 (b) (3g 2h3)4 (4p3q)2 × pq5 Hukum indeks:
xm xm (c)
Kolam renang ym 2p4q Laws of indices:
Swimming pool = x7 + 3 – 2 = 34 × g 2 × 4 h 3 × 4 • am × an = am + n
16p6q2 × pq5 am ÷ an = am – n
= x8 = 81g 8 h 12 = •
2p4q • (am)n = amn
1m • (ab)m = ambm
= 8p6 + 1 – 4 q2 + 5 – 1
a m am
30 m = 8p3 q6
•  
b
= m
b

Panjang lantai ialah 30 m dan panjang kolam renang itu adalah melebihi 20 m. Jika perimeter dan luas kolam renang
masing-masing ialah 84 m dan 416 m2, cari nilai x dan nilai y. Seterusnya, tentukan luas kawasan lantai yang ditutupi 1. k6 × k 9 2. y7 ÷ y5 3. m3 n2 × m5 n
dengan jubin.
The length of the floor is 30 m and the length of the swimming pool is more than 20 m. If the perimeter and area of the swimming pool are = k6 + 9 = y7 – 5 = m3 + 5 n2 + 1
84 m and 416 m2 respectively, find the values of x and y. Hence, determine the area of the floor that is covered with tiles. = k15 = y2 = m8 n3

Panjang kolam renang = (30 – 2x) m Info KBAT


Lebar kolam renang = (y – 2) m
Bentuk dua persamaan yang masing-
masing melibatkan perimeter dan luas
Perimeter kolam renang = 84 m
kolam renang.
2(30 – 2x) + 2(y – 2) = 84 Form two equations which involve the r 8s3 × rs2
(30 – 2x) + (y – 2) = 42 perimeter and area of the swimming pool 4. (2a3 b2)5 5. (h6)2 ÷ h9 × h4 6.
respectively. r 7s
–2x + y = 14 = 25 × a3 × 5 b2 × 5 = h12 ÷ h9 × h4
y = 2x + 14 …… ➀ = r 8 + 1 – 7 s3 + 2 – 1
= 32a15b10 = h12 – 9 + 4
Diberi panjang kolam renang  20 m.
= r 2s4
Luas kolam renang = 416 m2 = h7
(30 – 2x)(y – 2) = 416 Maka, 30 – 2x  20
(15 – x)(y – 2) = 208 …… ➁ –2x  –10
2x  10
Gantikan ➀ ke dalam ➁. x5
(15 – x)[(2x + 14) – 2] = 208 Jadi, x = 2. ab7 × (6a)3 20p7q × p8q (x4y × xy3)2
(15 – x)(2x + 12) = 208 7. 8. 9.
(15 – x)(x + 6) = 104 Apabila x = 2, y = 2(2) + 14 24a2b6 (2p3)4 × pq5 x11y5
= 18
15x + 90 – x2 – 6x = 104 = ab × 216a
7 3
20p7q × p8q (x4 + 1y1 + 3)2
–x2 + 9x – 14 = 0 Luas kawasan yang ditutupi dengan jubin 24a2b6 = =
16p12 × pq5 x11y5
x2 – 9x + 14 = 0 = 30y – Luas kolam renang 216 1 + 3 – 2 7 – 6
(x – 2)(x – 7) = 0 = (30 × 18) – 416 = a b 20 7 + 8 – (12 + 1) 1 + 1 – 5 (x5y4)2
24 = p q = 11 5
x = 2 atau 7 = 540 – 416 16 x y
= 9a2b 5
= 124 m2 = p2q–3 x10y8
4 = 11 5
x y
5p2
= 3 = x10 – 11 y8 – 5
4q = x –1 y3
y3
=
x

47 48

12
B. Tanpa menggunakan kalkulator, cari nilai bagi setiap yang berikut. D. Diberi 2n = x dan 3n = y, ungkapkan setiap yang berikut dalam sebutan x dan/atau y.
Without using a calculator, find the value of each of the following. 1.3 Given 2n = x and 3n = y, express each of the following in terms of x and/or y. 1.3

CONTOH 1. 55 × 1252 ÷ 254 = 55 × (53)2 ÷ (52)4 CONTOH 1. 9n + 32n 2. (12)n + 1 3. 6n – 813n


3 3 = 55 × 56 ÷ 58 (a) 16n + 27n = (32)n + (25)n = 12n × 121 = (2 × 3)n – (34)3n
9 2 × 3–6 = (32) 2 × 3–6 = 55 + 6 – 8 = (3n)2 + (2n)5 = (22 × 3)n × 12 = 2n × 3n – 312n
= (24)n + (33)n
= 33 × 3−6 = y2 + x5 = 22n × 3n × 12 = xy – y12
= 53 = (2n)4 + (3n)3
= 33 + (–6)
= 3–3 = 125 = x 4 + y3 = 12x2y
1
= 3
3
1 (b) 18n × 4n + 1 4. 54n × 32n 5. 24n + 1 × 243n 6. 36n + 1 ÷ 2(12n)
=
27
= (2 × 32)n × 4n × 41 = (2 × 33)n × (25)n = 24n × 241 × (35)n = 36 × 36n
n 1

= 2n × 32n × 22n × 4 = 2n × 33n × 25n = (23 × 3)n × 24 × 35n 2 × 12


n
= 18  36 n 
1 3 1 3 1
2. 216 3 × 4–1 × 9 2 = (63) 3 × 1 × (32) 2 = x × y 2 × x2 × 4 = x × y3 × x5 = 2 3n × 3n × 24 × 35n
3. 8 × 162 = (23) 2 × (24)2 12
4 3 3 = 4x 3 y2 = x6y 3 = x 3 × y × 24 × y 5 n
= 6 × 1 × 33 32 2 (25) 2 = 24x 3y 6 = 18  36 
4 3 12
= 2 ×15 2
2 8
3
= × 27 = 18(3n)
2
1 22 = 18y
= 40 3 15
2 +8–
= 22 2

= 22 E. Permudahkan setiap ungkapan yang berikut.


=4 Simplify each of the following expressions. 1.3

CONTOH
1. 2n + 5 – 2n + 2 = 2n (25) – 2n (22)
C. Permudahkan dan cari nilai bagi setiap yang berikut. 3n – 3n + 1 + 3n + 2 = 3n – 3n (3) + 3n (32)
Simplify and find the value of each of the following. 1.3
= 2n (32) – 2n (4)
= 3n – 3n (3) + 3n (9) = 2n (32 – 4)
CONTOH y+1
× 31 – y = (3 × 7) ×3
y+1 1–y = 3n (1 – 3 + 9)
1. 21 = 2n (28)
7y 7y = 7(3n)
20n × 251 – 0.5n = (22 × 5)n × (52)1 – 0.5n = 2n (22 × 7)
y+1
× 7 y + 1 × 31 – y
22n – 1 22n – 1 =3 = 7(2n + 2)
7y
= 2 × 52n –×15
2n n 2–n
= 3 y + 1 + (1 – y) × 7 y + 1 – y
2
=3 ×7
2 1 n n
= 22n – (2n – 1) × 5n + (2 – n) 2. 3 y + 4(3 y + 2) – 3 y + 3 = 3 y + 4(3 y)(32) – 3 y (33) 3. 7(6n) + 36 2 + 6n + 1 = 7(6 n) + (62) 2 + 6 n (6)
= 63
= 21 × 52 = 3 y + 3 y (36) – 3 y (27) = 7(6 n) + 6 n + 6 n (6)
= 50 = 3 y (1 + 36 – 27) = 6 n (7 + 1 + 6)
= 10(3 y) = 14(6 n)

2. 45 n –×13 = (3 × 5)n – 1 × 3 (23 × 7) x


2 n+1 –2n
n+1 –2n
3. 56x =
3x – 1 3x – 1
5 5
7x + 1 × 4 2 7 x + 1 × (22) 2
2n + 2
×5 n+1
× 3–2n
= 3 x
= x 2+ 1 × 73x – 2
3x x 2n 2n
5n – 1 4. 5x + 1 + 5x – 1 = 5x (5) + 5 5. 4n + 1 – 22n + 1 + 8 3 = 4n (4) – 22n (2) + (23) 3
=3 2n + 2 + (–2n)
× 5n + 1 – (n – 1) 7 ×2 5
= 5 5 + 
x 1 = 4n (4) – 4n (2) + 22n
= 32 × 52 = 23x – (3x – 2) × 7 x – (x + 1) 5 = 4n (4) – 4n (2) + 4n
= 22 × 7 –1
= 225 = 5x 26  = 4n (4 – 2 + 1)
= 4 5
7 = 26(5x – 1) = 3(4n)

49 50

5.2 Logaritma dan Hukum Logaritma SPM K1 ‘13, ‘14, ‘16 D. Cari nilai x bagi setiap yang berikut.
Find the value of x in each of the following. 2.1 ; 2.2
A. Ungkapkan setiap yang berikut dalam bentuk logaritma.
Express each of the following in the logarithmic form. 2.1 CONTOH
1. log3 x = 4 2. log32 x = 1
CONTOH 1. 343 = 73 2. 32 = 25
(a) log4 x = –3 5
x = 34 1
x = 4–3
625 = 54 log7 343 = 7 log2 32 = 5 = 81 x = 32 5
y = ax 1
loga y = x = 3 1
log5 625 = 4 4 = (25) 5
1
= =2
64

3.
1
= 3–2 4. M = x9 5. 8n = k
(b) logx 36 = 2
3. logx 125 = 3 4. logx 7 = – 1
9 36 = x2 Indeksnya 2
sama. 125 = x3 –1
logx M = 9 log8 k = n
log3 1 = –2 6 2 = x2 Samakan 5 3 = x3 7=x 2
9 asasnya.
x=6
x=5 x = 7–2
= 1
49
B. Ungkapkan setiap yang berikut dalam bentuk indeks.
Express each of the following in the index form. 2.1
E. Permudahkan setiap yang berikut.
CONTOH 1. log5 25 = 2 2. log3 243 = 5
Simplify each of the following. 2.3

log2 64 = 6 25 = 5 2
243 = 3 5 CONTOH
loga y = x
y = ax 1 FAKTA UTAMA
(a) loga 2 + 3 loga 6 – loga 12 (b) logx x2 – logy
y + log2
64 = 26 8
Hukum logaritma:
= loga 2 + loga 63 – loga 12 = 2 logx x – logy y–1 + log2 23 Laws of logarithms:
• loga xy = loga x + loga y
= loga  2 × 6
3 = 2(1) – (–1) logy y + 3 log2 2
3. log5
1
= –3 4. loga 5 = n 5. x = log7 y 12  =2+1+3 • loga  xy  = loga x – loga y
125 = loga 36
5 = an y = 7x =6 • loga x m = m loga x
1 = 5–3
125
1 3
1. 2 loga 12 + loga 10 – loga 15 2. logx 8 – 2 logx 6 + logx 9
3 2
C. Cari nilai bagi setiap yang berikut. = loga 122 + loga 10 – loga 15 1 3
= logx (23) 3 – logx 62 + logx (32) 2
Find the value of each of the following. 2.2
122 × 10
= loga   = logx 2 – logx 36 + logx 27
CONTOH 15
1. log10 13.8 2. log10 5 = loga 96 2
= logx  × 27
(a) log10 2.7 Tekan/Press: 8 36
= 1.140
= 0.4314 log 2 . 7 = = –0.2041 3
= logx  
2
0.43 363764

3. 6 logm xy – 3 logm x – 2 logm y 4. logb b + 2 logc c3 – log2 32


Beri jawapan akhir dalam 1
4 angka bererti. = logm (xy)6 – logm x3 – logm y2 = logb b 2 + 3(2) logc c – log2 25
3. log2 128 4. log5 0.2
x 6 y6 1
(b) log3 81 = log2 2 7
= log5 1 = logm 3 2 = logb b + 6 logc c – 5 log2 2
2
5 x y
= log3 34 = 7 log2 2 1
= log5 5–1 = logm x3y4 = +6–5
= 4 log3 3 loga a = 1
=7 2
= 4(1) = –1
3
=4 =
2

51 52

13
F. Selesaikan setiap yang berikut. G. Bagi setiap persamaan yang berikut, ungkapkan y dalam sebutan x.
Solve each of the following. 2.3 For each of the following equations, express y in terms of x. 2.3 ; 2.4

CONTOH CONTOH
1. 3 log2 x + 2 log2 y = 1
Diberi loga 2 = x dan loga 5 = y, ungkapkan setiap yang berikut dalam sebutan x dan y. log5 xy + 1 = 2 log5 x – log5 y
Given loga 2 = x and loga 5 = y, express each of the following in terms of x and y. log2 x3 + log2 y2 = 1
log5 xy + log5 5 = log5 x2 – log5 y
(a) loga 40 (b) loga 0.08a (c) loga 1257 x2
log2 x3 y2 = 1
2 16a log5 5xy = log5 x3 y2 = 21
= loga (23 × 5) = loga a y
25 = loga 125 – loga 16 – loga a7 x2 2
= loga 23 + loga 5 5xy = y2 = 3
= loga 2 + loga a – loga 52 3
y x
= 3 loga 2 + loga 5 = loga 5 2 – loga 24 – 7 loga a x x 2
= loga 2 + 1 – 2 loga 5 y2 = → y = y= 3
= 3x + y 3 5 5 x
= x + 1 – 2y = loga 5 – 4 loga 2 – 7(1)
2
= 3 y – 4x – 7 2. log3 xy – 2 = 3 log3 x – log3 y 3. log4 (x + y) – 5 log4 x = 3
2
log3 xy – log3 32 = log3 x3 – log3 y log4 (x + y) – log4 x 5 = 3
1. Diberi loga 3 = x dan loga 7 = y, ungkapkan setiap yang berikut dalam sebutan x dan y.
Given loga 3 = x and loga 7 = y, express each of the following in terms of x and y. log3 xy – log3 9 = log3 x3 – log3 y x+y
log4 =3
49a 343a5
xy x3 x5
(a) loga 63 (b) loga (c) loga log3 = log3
81 3 9 y x+y
= loga (32 × 7) xy x3 5 = 43
= loga 49 + loga a – loga 81 = x
= loga 343 + loga a5 – loga 3 9 y
= loga 32 + loga 7 x + y = 64x 5
= loga 72 + loga a – loga 34 3 5
= 2 loga 3 + loga 7 = loga 7 2 + loga a 2 – loga 3 y2 = 9x2 y = 64x 5 – x
= 2 loga 7 + 1 – 4 loga 3 3 5
= 2x + y = loga 7 + loga a – loga 3 y = 3x
= 2y + 1 – 4x 2 2
3 5
= y+ –x H. Selesaikan setiap yang berikut dengan menggunakan hukum logaritma.
2 2 Solve each of the following by using the laws of logarithms. 2.3 ; 2.4

2. Diberi log3 2 = m dan log3 5 = n, ungkapkan setiap yang berikut dalam sebutan m dan n. CONTOH
Diberi: log xy = 3 3
Given log3 2 = n and log3 5 = n, express each of the following in terms of m and n. Diberi: 2 dan/and log2 x2 = –5
(a) log3 90 (b) log3 0.3 (c) log3 3.75 Given:
log3 x2y = 1 dan/and log3 x4y3 = 4 Given: y
= log3 (2 × 5 × 32) 3 15 Cari nilai bagi log2 x dan log2 y.
= log3 = log3 Cari nilai bagi log3 x dan log3 y.
10 4 Find the values of log2 x and log2 y.
= log3 2 + log3 5 + log3 32 Find the values of log3 x and log3 y.
3 3×5
= log3 2 + log3 5 + 2(1) = log3 = log3
2×5 22 log3 x2y = 1 log2 xy = 3
=m+n+2 = log3 3 – log3 2 – log3 5 = log3 3 + log3 5 – log3 22 log3 x2 + log3 y = 1 log2 x + log2 y = 3 …… ➀
=1–m–n = 1 + log3 5 – 2 log3 2 2 log3 x + log3 y = 1 …… ➀ (× 2) 2 log2 x + 2 log2 y = 6 …… ➁


(× 3)
= 1 + n – 2m 6 log3 x + 3 log3 y = 3 …… ➁ x 3
log2 2 = –5
y
log3 x4 y3 = 4
3. Diberi logx 4 = p dan logx 5 = q, ungkapkan setiap yang berikut dalam sebutan p dan q. log2 x3 – log2 y2 = –5
log3 x + log3 y3 = 4
4
Given logx 4 = p and logx 5 = q, express each of the following in terms of p and q. 3 log2 x – 2 log2 y = –5 …… ➂
4 log3 x + 3 log3 y = 4 …… ➂
(a) logx 100 (b) logx 0.8x3 (c) logx 12.5 ➁ + ➂: 5 log2 x = 1
4 x ➁ – ➂: 2 log3 x = –1
= logx (4 × 52) = logx x3 25 log2 x =
1
5
= logx 4 + logx 52
= logx
2 x log3 x = – 1 5
= logx 4 + logx x3 – logx 5 1
2
1
= logx 4 + 2 logx 5 Gantikan log2 x = ke dalam ➀.
= logx 4 + 3 logx x – logx 5 = logx 25 – logx 2 – logx x 2 Gantikan log3 x = – 1 ke dalam ➀. 5
= p + 2q 1 2 1
=p+3–q 1
= logx 52 – logx 4 2 – logx x + log2 y = 3
2 2– 1  + log3 y = 1 5
2 1
= 2 logx 5 – 1 logx 4 – 1 (1) –1 + log3 y = 1 log2 y = 3 –
5
2 2
log3 y = 2 14
=
= 2q – 1 p – 1 5
2 2

53 54

5.3 Penukaran Asas Logaritma SPM K1 ‘13, ’14, ’15, ’16 K2 ’14 C. Selesaikan masalah yang berikut.
Solve the following problems. 3.1 ; 3.2
A. Cari nilai setiap logaritma dengan menukarkan asasnya kepada asas 10.
Find the value of each logarithm by changing the base to base 10. 3.1 CONTOH
CONTOH FAKTA UTAMA (a) Diberi log4 m = p, ungkapkan log2 16m5 dalam (b) Diberi log3 4 = x dan log3 5 = y, ungkapkan
sebutan p. log4 0.6 dalam sebutan x dan y.
log10 8.2 Tekan: logc b Given log4 m = p, express log2 16m5 in terms of p.
(a) log7 8.2 = = 1.081 ÷ .08 3 054 • loga b = Given log3 4 = x and log3 5 = y, express log4 0.6 in terms
log10 7 Press: log 8 · 2 log 7 =
logc a of x and y.
log4 16m5 log4 16 + log4 m5
1 log2 16m5 = =
log10  3  Tekan:
log ( 3 ab/c 5 ) ÷ log 2 =
• loga b =
logb a log4 2 log4 2
log4 0.6 =
log3 0.6
(b) log2 3 = 5 = –0.7370 Press: log4 42 + 5 log4 m log3 4
5 log10 2 – 0.736965594 = 1
log3 3
log4 4 2 = 5
2 log4 4 + 5p x
log10  7  = log3 3 – log3 5
1. log5 13 = log10 13
log10 4.53 9 1 log 4
2. log8 4.53 = 3. log6 7 = 4
=
x
log10 5 log10 8 9 log10 6 2
1–y
= 1.594 = 0.7265 = –0.1403 = 2(2 + 5p) =
x
= 4 + 10p

1. Diberi log3 h = k, ungkapkan log9 27h4 dalam 2. Diberi log7 2 = x dan log7 3 = y, ungkapkan
B. Bagi setiap yang berikut, ungkapkan P dalam sebutan Q. sebutan k. log24 49 dalam sebutan x dan y.
For each of the following, express P in terms of Q. 3.1 ; 3.2 Given log3 h = k, express log9 27h4 in terms of k. Given log7 2 = x and log7 3 = y, express log24 49 in terms
CONTOH of x and y.
1. log2 P + log8 Q = 1 log3 27h4
log9 27h4 = log7 49
log3 P + log9 Q = 2 log2 Q log3 9 log24 49 =
log2 P + = log2 2 log7 24
log3 Q log2 8 log3 27 + log3 h4
log3 P + = log3 32 = log7 72
log3 9 log2 Q log3 32 =
log2 P + = log2 2 log7 (23 × 3)
3
log3 P +
log3 Q
= log3 9 1 log3 33 + 4 log3 h 2 log7 7
log2 P + log2 Q = log2 2 = =
2 1 2 log3 3 log7 23 + log7 3
3
log3 P + log3 Q 2 = log3 9 1 3 log3 3 + 4k 2
log2 P + log2 Q 3 = log2 2 = =
2 3 log7 2 + y
log3 P = log3 9 – log3 Q log2 P = log2 2 – log2 3 Q
= 3 + 4k =
2
log3 P = log3 9 log2 P = log2 2 2 3x + y
Q 3
Q
P= 9 P= 2 2
4. Diberi log5 4 = a dan log3 4 = b, ungkapkan
Q 3 3. Diberi log25 m = n, ungkapkan logm m dalam
Q 125 log2 1.8 dalam sebutan a dan b.
sebutan n. 2 Given log5 4 = a and log3 4 = b, express log2 1.8 in terms
Given log25 m = n, express logm m in terms of n. of a and b.
2. log5 P – 6 log25 Q = 3 3. 4 log49 P – 2 log7 Q – 1 = 0 125 KBAT
2
logm m = logm m2 – logm 125
2
6 log5 Q 4 log7 P 125 log2 1.8 = log2 3 = log2 32 – log2 5
log5 P – = log5 53 – 2 log7 Q – log7 7 = 0 5
log5 25 log7 49 log25 125 = 2 log2 3 – log2 5
= 2 logm m –
6 log5 Q 4 log7 P log25 m
log5 P –
2
= log5 125
2
– log7 Q 2 – log7 7 = 0 3 = 2 – 1
2 log3 2 log5 2
log5 P – 3 log5 Q = log5 125 2 log7 P – log7 Q 2 – log7 7 = 0 = 2 – log25 25 2 1
n = –
log5 P – log5 Q3 = log5 125 log7 P 2 – log7 Q 2 – log7 7 = 0 3 log 25 1 1
25 log3 4 2 log5 4 2
=2– 2
log5 P = log5 125 + log5 Q3 log7 P 2 = log7 7 + log7 Q 2 n 2 1
= –
log5 P = log5 125Q3 log7 P 2 = log7 7Q 2 =2– 3 1 log 4 1 log 4
3 5
2n 2 2
P = 125Q3 P 2 = 7Q 2 4n – 3
=
2n = 4 – 2
P = 7Q b a

55 56

14
5.4 Penyelesaian Persamaan yang Melibatkan Indeks dan Logaritma SPM K1 ‘13, ‘16 C. Selesaikan persamaan logaritma yang berikut.
Solve the following logarithmic equations. 4.2
A. Selesaikan persamaan yang berikut.
Solve the following equations. 4.1 CONTOH 1. log2 (5x + 2) = log2 (x – 2) + 3
CONTOH 1. 343x – 1 = 49 2. 16y + 2 = 32 log5 (2x + 7) – log5 (x + 2) = 1
log2 (5x + 2) = log2 (x – 2) + log2 23
(a) 9x + 1 = 27
(32)x + 1 = 33
(73)x – 1 = 72 (2 4) y + 2 = 25 log5 2x + 7 = 1 log2 (5x + 2) = log2 (x – 2) + log2 8
73x – 3 = 72 24y + 8 = 25 x+2 log2 (5x + 2) = log2 8(x – 2)
32x + 2 = 33 2x + 7 = 51
Jika ax = ay, 3x – 3 = 2 4y + 8 = 5 5x + 2 = 8(x – 2)
2x + 2 = 3 x+2
Maka x = y. 3x = 5 4y = –3 5x + 2 = 8x – 16
2x = 1 2x + 7 = 5(x + 2)
x= 5 y = –3 3x = 18
x= 1 3 4 2x + 7 = 5x + 10
2 x=6
3x = –3
x
3. 27 = 1x 4. 2n × 8n + 1 = 4
(b) 625(5x) = 1 x 3 9 x = –1
125 2n × (23)n + 1 = 22
54(5x) = (5–3)x (3 )(3–1) = (3–2) x
3x
2n × 23n + 3 = 22 2. log4 x + 1 = log4 (x + 9) 3. log3 (x – 8) + log3 x = 2
54 + x = 5–3x 33x – 1 = 3–2x 2n + 3n + 3 = 22
4 + x = –3x 3x – 1 = –2x 4n + 3 = 2 log4 x + log4 4 = log4 (x + 9) log3 (x – 8)x = 2
4x = –4 5x = 1 4n = –1 log4 4x = log4 (x + 9) x(x – 8) = 32
x = –1 x= 1 n=–1 4x = x + 9 x2 – 8x = 9
5 4 2
3x = 9 x – 8x – 9 = 0
(c) 2x + 2x + 1 = 24 5. 6x + 6x + 1 = 252 6. 3x + 2 – 2(3x + 1) = 1 x=3 (x + 1)(x – 9) = 0
2x + (2x)(2) = 24
2x(1 + 2) = 24 6 x + (6 x)(61) = 252 (3x)(32) – 2(3x)(31) = 1 Disebabkan x  0, maka x = 9.
2x(3) = 24 6 x(1 + 6) = 252 3x(9 – 6) = 1
2x = 8 6 x(7) = 252 3x(3) = 1
2x = 23 6 x = 36 1
3x = = 3–1
x=3 6 x = 62 3
x=2 x = –1 D. Selesaikan persamaan logaritma yang berikut.
Solve the following logarithmic equations. 4.2

B. Selesaikan persamaan yang berikut. CONTOH


Solve the following equations. 4.1 1. log6 (3x + 4) – 2 log36 x = 1 2. log2 (4x + 3) – 6 log8 x = 2
log3 x – log9 (2x + 15) = 0
CONTOH 1. 6x + 1 = 9x 2 log6 x 6 log2 x
log6 (3x + 4) – =1 log2 (4x + 3) – =2
4 x – 3 = 7x log3 (2x + 15) log6 36 log2 8
log10 6 x + 1 = log10 9 x log3 x – =0
log10 4x – 3 = log10 7x log3 9 2 log6 x 6 log2 x
(x + 1) log10 6 = x log10 9 log6 (3x + 4) – =1 log2 (4x + 3) – =2
(x – 3) log10 4 = x log10 7 log3 (2x + 15) log6 62 log2 23
x log10 6 + log10 6 = x log10 9 log3 x – =0
x log10 4 – 3 log10 4 = x log10 7 log3 32 2 log6 x 6 log2 x
log10 6 = x log10 9 – x log10 6 log6 (3x + 4) – =1 log2 (4x + 3) – =2
x log10 4 – x log10 7 = 3 log10 4 log3 (2x + 15) 2
log10 6 = x(log10 9 – log10 6) log3 x – =0 3
x(log10 4 – log10 7) = 3 log10 4 2 log6 (3x + 4) – log6 x = 1
Gunakan kalkulator log2 (4x + 3) – 2 log2 x = 2
3 log10 4 untuk mencari log10 6 log3 (2x + 15)
x= = –7.432 nilai ini.
x=
log10 9 – log10 6
= 4.419 log3 x = log6 3x + 4 = 1 log2 (4x + 3) – log2 x2 = 2
log10 4 – log10 7 2 x
2 log3 x = log3 (2x + 15) 3x + 4 = 61 log2 4x +2 3 = 2
2. 52x = 4x – 1 3. 13x – 1 – 7x + 3 = 0 2 x x
log3 x = log3 (2x + 15)
13x – 1 = 7 x + 3 3x + 4 = 6x 4x + 3 = 22
log10 52x = log10 4x – 1 x2 = 2x + 15 3x = 4
log10 13x – 1 = log10 7 x + 3 x2
2x log10 5 = (x – 1) log10 4 x2 – 2x – 15 = 0 4x + 3 = 4x2
(x – 1) log10 13 = (x + 3) log10 7 x= 4
2x log10 5 = x log10 4 – log10 4 (x + 3)(x – 5) = 0 3 4x2 – 4x – 3 = 0
log10 4 = x log10 4 – 2x log10 5 x log10 13 – log10 13 = x log10 7 + 3 log10 7 (2x + 1)(2x – 3) = 0
x log10 13 – x log10 7 = 3 log10 7 + log10 13 Disebabkan x  0, maka x = 5.
log10 4 = x(log10 4 – 2 log10 5) Disebabkan x  0, maka x = 3 .
log10 4 x(log10 13 – log10 7) = 3 log10 7 + log10 13 2
x= = –0.7565 3 log10 7 + log10 13
log10 4 – 2 log10 5 x= = 13.574
log10 13 – log10 7

57 58

PRAKTIS FORMATIF Kertas 1 ANALISIS SOALAN SPM 7. Diberi logp 3 = x dan logp 6 = y, ungkapkan log6 27p3 10. Selesaikan persamaan:
5.4
5.2
Subtopik 2013 2014 2015 2016 KLONdalam sebutan x dan y. Solve the equation:
SPM
5.1 – S. 6 S. 7 – ’15 Given logp 3 = x and logp 6 = y, express log6 27p3 in terms 8(22x – 4) = 1
Jawab semua soalan. 5.2 S. 7(a) S. 7(a) – S. 14(a) 5.3
5.2
of x and y. [3]
5.3 S. 7(b) S. 7(b) S. 6 S. 14(b)
Answer all the questions.
5.4 S. 8 – – S. 15
[3] 8(22x – 4) = 1
log6 27p3 = log6 27 + log6 p3 1
1. Ringkaskan: (6x3y2)2 22x – 4 = = 2–3
logp 27 logp p3
Simplify: 4x2y 4. Diberi log2 x = m dan log2 y = n, ungkapkan log2 x2 = + 8
KLON
SPM 5.2 y logp 6 logp 6 2x – 4 = –3
’14 [2] dalam sebutan m dan n. 3
logp 3 3 logp p 2x = 1
5.1
5.2
x = +
(6x3y2)2 36x6y4 Given log2 x = m and log2 y = n, express log2 in terms of y y 1
= y2 x=
4x2y 4x2y m and n. 3 logp 3 3(1) 2
= +
= 36 × x6 – 2 × y4 – 1 [3] y y
4
log2 x2 = log2 x – log2 y2 = 3x + 3
= 9x4y3 y
y y 11. Selesaikan persamaan:
= 3(x + 1)
5.4
5.2
= log2 x – 2 log2 y Solve the equation:
y 1 + log3 x = log3 (x + 6)
= m – 2n
[3]
1 + log3 x = log3 (x + 6)
2. Diberi 23x = k, 2y = h dan 2y – 3x = 6 + 8x. Ungkapkan log3 (x + 6) – log3 x = 1
8. Diberi log2 3 = m dan log2 5 = n, ungkapkan log4 75
KLON h dalam sebutan k.
SPM 5. Diberi logk 27 = 3, cari nilai 5.3
5.2
dalam sebutan m dan n. log3  x + 6  = 1
’15 It is given that 23x = k, 2y = h and 2y – 3x
= 6 + 8x. Express KLON Given logk 27 = 3, find the value of x
5.1
5.2 SPM Given log2 3 = m and log2 5 = n, express log4 75 in terms of
h in terms of k. x + 6 = 31
 
1 .
’14
(a) k. (b) log27 m and n.
KBAT [3] k x
5.2
5.2 5.3
5.2 [3]
y – 3x x x + 6 = 3x
2 =6+8 [3] log2 75
2y log4 75 = 2x = 6
= 6 + (23)x (a) logk 27 = 3 log2 4
x=3
23x
27 = k3 log2 (3 × 52)
h = 6 + 23x =
k 33 = k3 log2 22
h =6+k k=3 log2 3 + 2 log2 5 12. Diberi 2p = 5k = 10r, ungkapkan p dalam sebutan k
=
k 2 log2 2 KLONdan r.
SPM
h = k(6 + k) 1 Given 2 p = 5 k = 10 r, express p in terms of k and r.
 k  = log27 k–1 = m + 2n
’16
(b) log27 5.4
5.2
2 [3]
= –log27 k KBAT
2p = 5k
logk k log2 2p = log2 5k
=–
3. Diberi a = 14 , cari logk 27 p log2 2 = k log2 5
x 1
KLON
=– p = k log2 5
SPM
Given a =
1
, find 3 9. Selesaikan persamaan:
’13
x4 5.4
5.2 Solve the equation: p
KBAT
log2 5 =
(a) logx a. (b) 8 loga x. 32x – 32x – 2 = 24 k
5.2
5.2 5.3
5.2
6. Diberi loga 6 = m, ungkapkan setiap yang berikut
[3] KLON dalam sebutan m. [4] 2p = 10r
SPM

(a) logx a = logx 14


’16 Given loga 6 = m, express each of the following in terms of m. 32x – 32x – 2 = 24 log2 2p = log2 10r
x (a) loga 36 (b) log6 1 296a3 2x p log2 2 = r log2 10
32x – 3 2 = 24 p = r log2 (2 × 5)
= logx x–4 5.2
5.2 5.3
5.2
[4] 3
2x p = r (log2 2 + log2 5)
= –4 logx x 2x
3 – 3 = 24
2 9 p
= –4(1) (a) loga 36 = loga 6
1 p = r 1 + 
= –4 = 2 loga 6 1 – 9 (32x) = 24 k
= 2m p = r
k 
k + p
8
(b) 8 loga x = 8  log1x a  (b) log6 1 296a3 = log6 1 296 + log6 a3
 9 (32x) = 24
pk = kr + pr
loga a3 32x = 27
= 8 1  = log6 64 +
loga 6 32x = 33
pk – pr = kr
–4 p(k – r) = kr
2x = 3
= –2 =4+ 3 p = kr
m x = 11 k–r
2
= 4m + 3
m

59 60

15
PRAKTIS FORMATIF Kertas 2 ANALISIS SOALAN SPM
FOKUS KBAT
Subtopik 2013 2014 2015 2016
5.1 – S. 4(a) – –
Jawab semua soalan. 5.2 – – – –
5.3 – S. 4(b) – –
Answer all the questions.
5.4 – – – – Kemahiran Kognitif: Mengaplikasi dan Membanding
Konteks: Penyelesaian Persamaan Indeks
1. Diberi bahawa p = 3x dan q = 3y.
It is given that p = 3x and q = 3y. 2. (a) Permudahkan:
KLON
1. Diberi 2x = 9y = 24z, ungkapkan y dalam sebutan x dan z.
Info KBAT
SPM
KBAT
5.3 Simplify:
’14
27x + y Given that 2 x = 9 y = 24 z, express y in terms of x and z.
(a) Ungkapkan dalam sebutan p dan q. log2 (2x + 3) – 3 log4 x2 + 2 log2 x
KBAT
5.1
9y Gunakan suatu anu untuk mewakili ketiga-tiga
27 x + y [4] Katakan 2x = 9y = 24z = k
Express in terms of p and q. nombor indeks itu. Aplikasikan hukum indeks dan
9y seterusnya bandingkan indeks bagi anu itu.
[3] (b) Seterusnya, selesaikan persamaan: Maka, 2x = k , 9y = k , 24z = k Use an unknown to represent the three index numbers.
5.4 Hence, solve the equation: 1 1 1
9q2 2 = kx , 9 = ky , 24 = k z
Apply the law of indices and then compare the
(b) Cari log9 dalam sebutan x dan y. indices of the unknown.
5.3
p log2 (2x + 3) – 3 log4 x + 2 log2 x = 3 2
2 [2] 3 × 8 = 24
Find log9 9q in terms of x and y. Daripada
p 1 Bandingkan indeks di kedua-dua belah persamaan.
[5] (a) log2 (2x + 3) – 3 log4 x2 + 2 log2 x 9 2 × 23 = 24 1 + 3 = 1
1
1 3
log2 x2
k y  2 × k x  2y x z
1 1

(a) 27 y
x+y
=
3 3(x + y) = log2 (2x + 3) – 3
log2 4
+ log2 x2   =k z
1 = 1 – 3
9 32y 1 3 1
2y z x
log2 x2
= 33x + 3y – 2y
=3 3x + y
= log2 (2x + 3) – 3
2
+ log2 x2   k 2y × k x = k z
1
+ 3 1
1 = x – 3z
2y x 2y xz
= (3x)3 × 3y 3 k = kz
= log2 (2x + 3) – log2 x2 + log2 x2 y = xz
= p 3q 2 2x – 6z
1
= log2 (2x + 3) – log2 x2
2
(b) p = 3x dan q = 3y 1
Kemahiran Kognitif: Mengaplikasi
= log2 (2x + 3) – log2 (x 2) 2
log3 p = x log3 q = y Konteks: Penyelesaian Masalah dengan Logaritma
= log2 (2x + 3) – log2 x
9q2
log9 = log9 9q2 – log9 p = log2 2x + 3 2. Rajah di sebelah ialah graf yang menunjukkan keuntungan bagi sebuah
p x
Keuntungan, RMy
Profit, RMy syarikat. Cari keuntungan syarikat itu pada tahun ke-7. Beri jawapan anda
= log9 9 + log9 q2 – log3 p
log3 9 (b) log2 (2x + 3) – 3 log4 x 2 + 2 log2 x = 3 kepada puluh ribu ringgit yang terdekat.
y = xn + 4 The diagram is a graph showing the profits of a company. Find the profits of the
log3 q2 log3 p 62n
=1+
log3 32 –
log3 32 Maka, log2 2x + 3 = 3 company in the 7th year. Give your answer to the nearest ten thousand ringgit.
x
=1+
2 log3 q

log3 p 2x + 3 = 23
x
Info KBAT
2 2
2x + 3 = 8x Gantikan koordinat (7, 62n ) ke dalam persamaan y = x n + 4.
=1+y– x 6x = 3 O 7
x (Tahun/Year)
Substitute the coordinates of (7, 62n) into the equation y = x n + 4.
2
x= 1
2
Diberi y = x n + 4 Keuntungan syarikat
Pada x = 7, y = 62n = 62(4.753)
= RM24 952 024
Maka, 62n = 7 n + 4 ≈ RM24 950 000
log10 62n = log10 7 n + 4
2n log10 6 = (n + 4) log10 7 Atau
2n log10 6 = n log10 7 + 4 log10 7
Keuntungan syarikat
2n log10 6 – n log10 7 = 4 log10 7
= 7(4.753 + 4)
n(2 log10 6 – log10 7) = 4 log10 7
= RM24 954 171
4 log10 7
n= ≈ RM24 950 000
2 log10 6 – log10 7
= 4.753

61 62

6.2 Pembahagian Tembereng Garis SPM K1 ‘13, ‘14 K2 ’15, ‘16


BAB

6
GEOMETRI KOORDINAT A. Cari titik tengah bagi setiap tembereng garis AB.
COORDINATE GEOMETRY Find the midpoint of each line segment AB. 2.1

CONTOH FAKTA UTAMA


A(6, 11), B(–8, 7)
Titik tengah AB
6.1 Jarak di anta
antara Dua Titik Titik tengah AB = 
6 + (–8) , 11 + 7
2 
SPM K1 ‘15 Midpoint of AB B(x2, y2)
2
 
x1 + x2 y1 + y2
A. C
A Carii jarak
j k di antara
t setiap pasangan titik yang berikut. –2 18 = ,
= ,
2 2 
Find the distance between each of the following pairs of points. 1.1
2 2
A(x1, y1)
CONTOH FAKTA UTAMA = (–1, 9)
C(3, –1), D(5, 2) Q(x2, y2) 1. A(4, –7), B(10, –3) 2. A(–2, –11), B(9, –5)
Jarak CD = (3 – 5)2 + (–1 – 2)2 P(x1, y1)
= (–2)2 + (–3)2 Titik tengah AB =  4 + 10 , –7 + (–3)  Titik tengah AB =  –2 + 9 , –11 + (–5) 
Jarak PQ Jarak PQ 2 2 2 2
= 4+9 Distance of PQ atau Distance of PQ
or =  14 , –10  =  7 , –16 
= 13 = (x1 – x2)2 + (y1 – y2)2 = (x2 – x1)2 + (y2 – y1)2 2 2 2 2
= 3.606 unit = (7, –5) = 3 1 , –8
2
1. P(8, 1), Q(–4, 6) 2. M(−5, 13), N(−11, 4) 3. S(1, 14), T(9, 12)
Jarak PQ Jarak MN Jarak ST B. Cari nilai r bagi setiap yang berikut.
Find the value of r in each of the following. 2.1
= [8 – (–4)]2 + (1 – 6)2 = [–5 – (–11)]2 + (13 – 4)2 = (1 – 9)2 + (14 – 12)2
CONTOH 1. P(11, –9), Q(–1, r) 2. X(–2, 3r), Y(0, –4)
= 122 + (–5)2 = 62 + 92 = (–8)2 + 22
A(r, 6), B(1, 8)
= 144 + 25 = 36 + 81 = 64 + 4 Titik tengah PQ = (5, 4) Titik tengah XY = (–1, 10)
= 169 = 117 = 68 Titik tengah AB = (–2, 7) Midpoint of PQ Midpoint of XY
= 13 unit = 10.817 unit Midpoint of AB
= 8.246 unit
r+1 6+8  11 +2(–1) , –9 2+ r  = (5, 4)  –2 2+ 0 , 
3r + (–4) = (–1, 10)

B. Cari nilai h bagi setiap yang berikut.  2 , 2  = (–2, 7) 2

Find the values of h for each of the following. 1.1 Maka, –9 + r = 4 Maka, 3r – 4 = 10
r + 1 = –2 2 2
Maka,
CONTOH 2 –9 + r = 8 3r – 4 = 20
1. E(3, h), F(–1, 7), EF = 5 unit
A(–2, 5), B(h, –10), AB = 17 unit r + 1 = –4 r = 17 3r = 24
EF = 5 unit r = –5 r=8
AB = 17 unit
[3 – (–1)]2 + (h – 7)2 = 5
(–2 – h)2 + [5 – (–10)]2 = 17 Kuasa duakan 42 + (h – 7)2 = 25
(–2 – h)2 + 152 = 289 kedua-dua belah C. Bagi setiap yang berikut, ungkapkan h dalam sebutan k.
persamaan. 16 + h2 – 14h + 49 = 25
4 + 4h + h2 + 225 = 289 h2 – 14h + 40 = 0
For each of the following, express h in terms of k. 2.1
h2 + 4h – 60 = 0 (h – 4)(h – 10) = 0 CONTOH
(h – 6)(h + 10) = 0 1. A(7, h), B(3k, 9) 2. P(2h, k), Q(–6, –2)
h = 4 atau 10 F(k, –1), G(4, h)
h = 6 atau –10 Titik tengah AB = (r, 5r) Titik tengah PQ = (3n, n)
Titik tengah FG = (p, 2p) Midpoint of AB Midpoint of PQ
Midpoint of FG
2. K(h, 1), L(–3, 6), KL = 41 unit 3. y
(7, 2h) (0 – 7)2 + (6 – 2h)2 = 25
 7 +2 3k , h +2 9  = (r, 5r) 2h +2(–6) , 
k + (–2) = (3n, n)
KL = 41 unit (–7)2 + (6 – 2h)2 = 625
49 + 36 – 24h + 4h2 = 625
 k +2 4 , –1 2+ h  = (p, 2p) 7 + 3k = r dan h + 9 = 5r
2
2h – 6 = 3n dan k – 2 = n
25 unit
[h – (–3)] + (1 – 6)2 = 41
2
4h2 – 24h – 540 = 0 k+4 =p –1 + h = 2p 2 2 2 2
dan
(h + 3)2 + (−5)2 = 41 6 h2 – 6h – 135 = 0 2 2
h2 + 6h + 9 + 25 = 41 Maka, h + 9 = 5 7 + 3k  Maka, 2h – 6 = 3  k – 2 
(h + 9)(h – 15) = 0 2 2
h2 + 6h – 7 = 0 O
x
Maka, –1 + h = 2  k + 4  2 2
h = –9 atau 15 2 2 h + 9 = 35 + 15k 2h – 6 = 3k – 6
(h – 1)(h + 7) = 0 –1 + h = 2k + 8
2h  6, maka h = 15. h = 15k + 26
h = 1 atau –7 h = 2k + 9 h= 3k
2

63 64

16
D. Cari koordinat titik P yang membahagikan tembereng garis AB. 6.3 Luas Poligon SPM K2 ‘13, ‘15, ‘16
Find the coordinates of point P which divides the line segment AB. 2.2
A. Hitung luas setiap poligon berdasarkan bucu-bucu yang diberikan.
CONTOH FAKTA UTAMA Calculate the area of each polygon based on the given vertices. 3.2 ; 3.3

A(6, −4), B(−1, 10), AP : PB = 3 : 4 B(–1, 10) n CONTOH FAKTA UTAMA


m B(x2, y2)
P
(a) A(2, –5), B(0, 4), C(–2, 6) y
Koordinat titik P =  4(6) + 3(–1) , 4(–4) + 3(10) 
4 D(x4 , y4)
A(x1, y1) A(x1, y1)
3+4 3+4
24 – 3 –16 + 30 Luas segi tiga ABC
= 
P Koordinat titik P/Coordinates of point P
, 3
7
= (3, 2)
7
A(6, –4)
= 
nx1 + mx2 ny1 + my2
m+n
,
m+n  
= 1 2 0 –2 2
2 –5 4 6 –5  B(x2, y2)
C(x3, y3)

x
1 0
= (8 + 0 + 10) – (0 – 8 + 12) 
2 • Luas segi tiga ABC
1. A(5, 8), B(11, –7), AP : PB = 2 : 1 2. A(–1, –5), B(9, 3), AP : PB = 3 : 7 1 Area of triangle ABC
= 14 
Koordinat titik P =  1(5) + 2(11) , 1(8) + 2(–7)  Koordinat titik P =  7(–1) + 3(9) , 7(–5) + 3(3) 
2 = 
1 x1 x2 x3 x1
2 y1 y2 y3 y1 
2+1 2+1 3+7 3+7 = 7 unit2
=  5 + 22 , 8 – 14  =  –7 + 27 , –35 + 9 
=
1
2 
(x1 y2 + x2 y3 + x3 y1) – (x2 y1 + x3 y2 + x1 y3) 
3 3 10 10 (b) A(1, 6), B(5, –1), C(2, –3), D(–2, 0) • Luas sisi empat ABCD
= (9, –2) = 2, – 13  Luas sisi empat ABCD Area of quadrilateral ABCD
5
= 
1 1 5 2 –2 1
Pastikan semua bucu

disusun ikut arah jam
= 
1 x1 x2 x3 x4 x1
2 y1 y2 y3 y4 y1 
3. A(−2, 4), B(−7, 19), AB : PB = 5 : 3 4. A(1, –3), B(–2, –17), 2AP = 5PB 2 6 –1 –3 0 6 atau lawan arah jam.
1
=
1
2 
(x1 y2 + x2 y3 + x3 y4 + x4 y1) – (x2 y1 + x3 y2 + x4 y3 + x1 y4) 
Diberi AB : PB = 5 : 3, maka AP : PB = 2 : 3. Diberi 2AP = 5PB, maka AP : PB = 5 : 2. = (–1 – 15 + 0 – 12) – (30 – 2 + 6 + 0) 
2 • Nilai positif atau negatif bagi luas yang diperoleh bergantung
1 sama ada tertib bucu disusun ikut arah jam atau lawan arah
Koordinat titik P =  3(–2) + 2(–7) , 3(4) + 2(19)  Koordinat titik P =  2(1) + 5(–2) , 2(–3) + 5(–17)  = –62  jam. Apabila luas yang diperoleh bernilai negatif, tukarkannya
2+3 2+3 5+2 5+2 2
kepada positif.
= 31 unit2 Luas sentiasa bernilai positif.
=  –6 – 14 , 12 + 38  =  2 – 10 , –6 – 85  The positive or negative value of the area obtained depends on
5 5 7 7 whether the coordinates of vertices are arranged in a clockwise
or anticlockwise direction. When the area obtained is negative,
= (–4, 10) = – 8 , –13 convert it to positive.
7

E. Cari nilai h dan nilai k dengan keadaan titik P membahagikan tembereng garis AB. 1. P(7, 2), Q(–1, –2), R(8, 0) 2. E(2, –4), F(–5, 6), G(–2, 3)
Find the values of h and k where point P divides the line segment AB. 2.2
Luas segi tiga PQR Luas segi tiga EFG
CONTOH
   
1 7 –1 8 7 1 2 –5 –2 2
1. A(h, –1), B(5, k), P(7, 4), AP : PB = 1 : 2 = =
A(h, 9), B(2, k), P(–2, 6), AP : PB = 3 : 1 2 2 –2 0 2 2 –4 6 3 –4
1 1
1(h) + 3(2) 1(9) + 3(k)  2(h)1 ++ 21(5)
, 2(–1) + 1(k)  = (7, 4) = (–14 + 0 + 16) – (–2 – 16 + 0) = (12 – 15 + 8) – (20 – 12 + 6) 
 3 + 1 , 3 + 1  = (–2, 6) 1+2 2
1
2
1
=  20  = –9 
h + 6 9 + 3k  3 3 
2h + 5 , –2 + k = (7, 4)
 4 , 4  = (–2, 6) 2
= 10 unit2
2
= 4.5 unit2
Maka, 2h + 5 = 7 dan –2 + k = 4
Maka, h + 6 = –2 dan 9 + 3k = 6 3 3
4 4 2h + 5 = 21 –2 + k = 12
h + 6 = –8 9 + 3k = 24
h=8 k = 14 3. K(0, –1), L(4, –3), M(8, –2), N(–1, 10) 4. P(5, 9), Q(6, 0), R(3, –1), S(–5, –2)
h = –14 k=5
2. A(0, h), B(2k, 8), P(12, 3), AP : PB = 4 : 1 3. A(3h, k), B(–4, –3), P(−13, 6), AP : PB = 2 : 9 Luas sisi empat KLMN Luas sisi empat PQRS
   
1 0 4 8 –1 0 1 5 6 3 –5 5
= =
 1(0)4 ++ 4(2k)
1
, 1(h) + 4(8)  = (12, 3)
4+1 9(3h)2 ++ 92(–4) , 2+9 
9(k) + 2(–3) = (–13, 6) 2 –1 –3 –2 10 –1
1
2 9 0 –1 –2 9
1
= (0 – 8 + 80 + 1) – (–4 – 24 + 2 + 0)  = (0 – 6 – 6 – 45) – (54 + 0 + 5 – 10) 
 8k5 , h +5 32  = (12, 3)  11 11 
27h – 8 , 9k – 6 = (–13, 6) 2
1
2
1
8k = 12 h + 32 = 3 =
2
 99  =
2
 –106 
Maka, dan Maka, 27h – 8 = –13 dan 9k – 6 = 6
5 5 11 11 = 49.5 unit2 = 53 unit2
8k = 60 h + 32 = 15 27h – 8 = –143 9k – 6 = 66
k = 7.5 h = –17 h = –5 k=8

65 66

B. Selesaikan masalah yang berikut. 6.4 Persamaan Garis Lurus SPM K1 ‘14, ’15, ’16 K2 ’13, ’14, ’16
Solve the following problems. 3.2 ; 3.3
A. Cari kecerunan bagi setiap garis lurus yang menyambungkan dua titik yang diberikan.
CONTOH Find the gradient of each straight line which is connecting two given points. 4.2 ; 4.3

(a) Diberi titik A(–7, 1), B(p, 3) dan C(2, –1) adalah (b) Bucu-bucu sebuah segi tiga XYZ ialah X(1, h), CONTOH FAKTA UTAMA
segaris. Cari nilai p. Y(–2, 5) dan Z(–1, 3). Diberi luas segi tiga itu (a) A(2, 7) dan/and B(6, –3) • Kecerunan garis lurus PQ ialah
Given the points A(–7, 1), B(p, 3) and C(2, –1) are collinear. ialah 4 unit2. Cari nilai-nilai h yang mungkin. The gradient of the straight line PQ is
Find the value of p. The vertices of a triangle XYZ are X(1, h), Y(–2, 5) and 7 – (–3) y2 – y1
mAB = (a) m =
y
Z(–1, 3). Given the area of the triangle is 4 unit2. Find 2–6 x2 – x1
Apabila P
 
1 –7 p 2 –7 the possible values of h.
2 1 3 –1 1
=0 semua titik = 10 atau/or b (x1, y1)
adalah segaris, –4 y1 – y2
m=
 
luas poligon 1 1 –2 –1 1
(–21 – p + 2) – (p + 6 + 7)  = 0 bernilai sifar. =4
=–5
x1 – x2
(x2, y2)
2 h 5 3 h
–19 – p – (p + 13)  = 0 2 (b) m = –
Pintasan-y
=–
b
 (5 – 6 – h) – (–2h – 5 + 3)  = 8 Pintasan-x a x
–2p – 32  = 0 (b)
0 a
Q
 –1 – h – (–2h – 2]  = 8 y m=–
y-intercept
=–b
–2p – 32 = 0 x-intercept a
 h + 1 = 8 4
–2p = 32 Q • Persamaan garis lurus PQ ialah
h+1=8 atau h + 1 = –8 The equation of the straight line PQ is
p = –16
h=7 h = –9 (a) y – y1 = m(x – x1)
P x y – y1 y – y1 y – y2 y – y1
−6 O (b) = 2 atau/or = 2
x – x1 x2 – x1 x – x2 x2 – x1
1. Diberi titik P(5, k), Q(2, 0) dan R(3, –4) adalah 2. Diberi titik K(6, –2), L(3, 1) dan M(n, 2) adalah
4 = 2 x y
segaris. Cari nilai k. segaris. Cari nilai n. mPQ = – (c) + =1
Given the points P(5, k), Q(2, 0) and R(3, –4) are collinear. Given the points K(6, –2), L(3, 1) and M(n, 2) are collinear.
(–6) 3 a b
Find the value of k. Find the value of n.
1. A(5, 2) dan/and B(–3, 8) 2. C(–4, 9) dan/and D(–6, 1) 3. G(–1, –10) dan/and H(2, –3)
   
1 5 2 3 5 1 6 3 n 6
=0 =0 2–8 9–1 –10 – (–3)
2 k 0 –4 k 2 –2 1 2 –2 mAB = mCD = mGH =
5 – (–3) –4 – (–6) –1 – 2
 (0 – 8 + 3k) – (2k + 0 – 20)  = 0 (6 + 6 – 2n) – (–6 + n + 12)  = 0 –6 8
= = = –7
 –8 + 3k – 2k + 20  = 0 12 – 2n – (n + 6)  = 0 8 2 –3
=4
 k + 12  = 0 –3n + 6  = 0 =–3 = 7
4 3
k + 12 = 0 –3n + 6 = 0
k = –12 3n = 6 4. y 5. 6. y
y
n=2 P A
6 L x
x −12 O
O 6

Q B
3. Bucu-bucu sebuah segi tiga ABC ialah A(r, 6), 4. Bucu-bucu sebuah segi tiga CDE ialah C(3, –5), O 7
x −10 K −9
B(2, 0) dan C(–1, –2). Diberi luas segi tiga itu ialah D(–1, q) dan E(4, –8). Diberi luas segi tiga itu ialah
7 unit2. Cari nilai-nilai r yang mungkin. 10 unit2. Cari nilai-nilai q yang mungkin. 6 (–10) = 5 (–9) = – 3
mPQ = – mKL = – mAB = –
The vertices of a triangle ABC are A(r, 6), B(2, 0) and The vertices of a triangle CDE are C(3, –5), D(–1, q) and 7 6 3 (–12) 4
C(–1, –2). Given the area of the triangle is 7 unit2. Find E(4, –8). Given the area of the triangle is 10 unit2. Find
the possible values of r. the possible values of q.
B. Cari nilai k bagi setiap yang berikut.
Find the value of k for each of the following. 4.2 ; 4.3

   
1 r 2 –1 r 1 3 –1 4 3
=7 = 10 CONTOH
2 6 0 –2 6 2 –5 q –8 –5 1. P(4, k), Q(2, 7), mPQ = 2 2. X(1, 8), Y(2k, –3), mXY = –1
 (0 – 4 – 6) – (12 + 0 – 2r)  = 14  (3q + 8 – 20) – (5 + 4q – 24)  = 20 A(k, 5), B(–1, 3), mAB = 3
k–7 =2 8 – (–3) = –1
 –10 – (12 – 2r)  = 14  3q – 12 – (4q – 19)  = 20 5–3 =3 4–2 1 – 2k
 2r – 22  = 14  7 – q  = 20 k – (–1) k–7=4 11 = –1 + 2k
2 = 3k + 3 k = 11 12 = 2k
2r – 22 = 14 atau 2r – 22 = –14 7 – q = 20 atau 7 – q = –20 3k = –1 k=6
2r = 36 2r = 8 q = –13 q = 27 k=–1
r = 18 r=4 3

67 68

17
C. Cari persamaan garis lurus yang mempunyai kecerunan m dan/atau melalui titik-titik yang diberikan. E. Lengkapkan jadual yang berikut dengan menukarkan persamaan garis lurus kepada bentuk yang dinyatakan.
Find the equation of the straight line that has a gradient of m and/or passes through the given point(s). 4.4 (a), (b) Complete the following table by changing the equations of straight lines to the stated forms. 4.6

CONTOH Bentuk am Bentuk pintasan Bentuk kecerunan


1. m = 4, (–1, 3) 2. m = – 2 , (7, –6) General form Intercept form Gradient form
(a) m = –2, (3, 5) 3
y – 3 = 4[x – (–1)] 2 CONTOH 4x – 5y = 20 –5y = –4x + 20
y – 5 = –2(x – 3) y – (–6) = – (x – 7)
y – 3 = 4(x + 1) 3
y – 5 = –2x + 6 4x – 5y = 1 y = – 1 (–4x + 20)
y = –2x + 11 y – 3 = 4x + 4 y + 6 = – 2 x + 14 4x – 5y – 20 = 0 20 20 5
y = 4x + 7 3 3
x – y =1 y= 4x–4
y = – 2x – 4 5 4 5
3 3
(b) (–2, 1), (6, 5)
y–1 3x + 2y = 12 2y = –3x + 12
5–1
= 3. (9, –10), (7, –4) 4. (–3, –4), (2, –1) 3x 2y 1
x – (–2) 6 – (–2) 3x + 2y – 12 = 0 + =1 y = (–3x + 12)
y–1 1 12 12 2
= y – (–10) –4 – (–10) y – (–4) –1 – (–4) x y
x+2 2 = x – (–3)
=
2 – (–3) + =1 y=–3x+6
x–9 7–9 4 6 2
1 y+4 3
y – 1 = (x + 2) y + 10 =
2 = –3 x+3 5 y
1 x–9 x
y = +1
y= x+1+1 y + 10 = –3(x – 9) y + 4 = 3 (x + 3) 6 – x +  = 6 6 3
2 5 y
3 6 –x + =1
1 y = –3x + 27 – 10 y = 6  x + 1
y= 3x+ 9 –4
y= x+2 –2x + y = 6 3 6
y = –3x + 17 3
2 5 5 –2x + y – 6 = 0 y = 2x + 6
y = 3 x – 11
5 5 1
1 x + y = –3
x+y+3=0 4
4 1
x y y=– x–3
1 + =1
D. Cari persamaan bagi setiap garis lurus yang berikut dalam bentuk pintasan. 4  x + y + 3 = 0 4(–3) (–3) 4
4
Find the equation of each of the following straight lines in the intercept form. 4.4 (c) x y
x + 4y + 12 = 0 – – =1
12 3
CONTOH FAKTA UTAMA
(a) y Jenis persamaan garis lurus:
Types of equations of straight lines: F. Cari kecerunan setiap garis lurus berdasarkan persamaan yang diberikan.
(0, 5) Find the gradient of each straight line based on the given equation. 4.5 ; 4.6
x + y • Bentuk am / General form
=1
8 5 ax + by + c = 0 ; a, b, c = pemalar / constants CONTOH y y
x • Bentuk pintasan / Intercept form y 1. x + =1 2. – x + =1
O y 4 10 3 12
(a) x – = 1
(8, 0)
x y
+ = 1 ; a = pintasan-x / x-intercept 6 2
a b b Pintasan-x = 4 Pintasan-x = –3
(b) y b = pintasan-y / y-intercept
(−5, 0) x + y Pintasan-y = 10 Pintasan-y = 12
x =1 • Bentuk kecerunan / Gradient form Pintasan-x = 6
O (–5) (–4) x
Kecerunan, m = – 10 Kecerunan, m = – 12
y = mx + c ; m = kecerunan / gradient O a Pintasan-y = –2
y
–x – =1 c = pintasan-y / y-intercept 4 (–3)
5 4 Kecerunan, m = – (–2)
=–5
(0, −4)
6 =4
2
= 1
1. y 2. y 3. y 3
(−10, 0)
(0, 9) x
(0, 3) O
(b) 7x + 2y – 5 = 0 3. 6x – 8y + 3 = 0 4. 5x – 3y – 6 = 0

2y = –7x + 5 –8y = –6x – 3 –3y = –5x + 6


x 6 3
y= 5x– 6
O (0, −7)
(−4, 0)
y=–7x+ 5 y= x+
x y = mx + c 8 8 3 3
O (4, 0) 2 2
x + y =1 x + y =1 y= 3x+ 3 y= 5x–2
x + y =1 (–4) 3 (–10) (–7) Kecerunan, m = – 7 4 8 3
y y
4 9 –x + =1 – x – =1
2 Kecerunan, m = 5
4 3 10 7 Kecerunan, m = 3 3
4

69 70

G. Cari koordinat titik persilangan bagi setiap pasangan garis lurus. 6.5 Garis Lurus Selari dan Garis Lurus Serenjang SPM K1 ’13, ’14, ’15, ’16 K2 ’13, ’14, ’16
Find the coordinates of the point of intersection for each pair of straight lines. 4.7
A. Diberi setiap pasangan garis lurus yang berikut adalah selari. Ungkapkan p dalam sebutan q.
CONTOH 1. y Given each of the following pairs of straight lines are parallel. Express p in terms of q.
y
2x – y + 10 = 0
CONTOH FAKTA UTAMA
2x + 3y = 18 x – 2y = 2
x + 3y = 9 (p + 1)x + 3y = 9 , qy – 5x + 7 = 0 • Garis lurus selari
Parallel straight lines
Bagi (p + 1)x + 3y = 9: Bagi qy – 5x + 7 = 0: y
x 3y = –(p + 1)x + 9 qy = 5x – 7 m1
O
p+1x+3 m1 = m2
O
x
y=– y= 5x– 7 m2
3 q q
2x + 3y = 18 …… ➀
x + 3y = 9 …… ➀ m1 = – p + 1 m2 = 5 O
x
x – 2y = 2 …… ➁ 3 q
2x – y + 10 = 0 …… ➁ • Garis lurus serenjang
Dari ➁: x = 2y + 2 …… ➂ Maka, –
p+1 = 5 Bagi garis lurus selari, m1 = m2.
Perpendicular straight lines
Dari ➁: y = 2x + 10 …… ➂ 3 q y
Gantikan ➂ ke dalam ➀. m1 m1m2 = –1
Gantikan ➂ ke dalam ➀. 2(2y + 2) + 3y = 18 p + 1 = – 15
q m2 atau/or
x + 3(2x + 10) = 9 4y + 4 + 3y = 18 m2 = – 1
x + 6x + 30 = 9 7y = 14 p = – 15 – 1 x
m1
7x = –21 q O
y=2
x = –3
Gantikan y = 2 ke dalam ➂.
Gantikan x = –3 ke dalam ➂. x = 2(2) + 2 1. px – 4y = 5 , (q – 2)x + 6y = 3 2. (2p + 3)x – 8y = 7 , 2y – qx – 4 = 0
y = 2(–3) + 10 =6
=4 Bagi px – 4y = 5: Bagi (q – 2)x + 6y = 3: Bagi (2p + 3)x – 8y = 7: Bagi 2y – qx – 4 = 0:
Titik persilangan ialah (6, 2). 4y = px – 5 6y = –(q – 2)x + 3 8y = (2p + 3)x – 7 2y = qx + 4
Titik persilangan ialah (–3, 4). p q–2 2p + 3 q
y= x– 5 y=– x+
1 y= x–
7 y= x+2
4 4 6 2 8 8 2
2. y 3. y
m1 =
p
m2 = –
q–2
m1 =
2p + 3
m2 =
q
4x + 3y = 12
9 4 6 8 2
10y – 4x + 5 = 0
x
O x y p q–2 2p + 3 q
– =1 Maka, =– Maka, =
8 6
x
4 6 8 2
O 2q 4
p = 2q – 3
6
p=– +
3 3 2
4x + 3y = 12 …… ➀ 10y – 4x + 5 = 0 …… ➀
x – y Bagi garis lurus yang menyilang paksi koordinat:
=1 …… ➁ B. Cari persamaan garis lurus yang melalui titik P dan selari dengan garis lurus yang diberikan.
8 6 x + y
3 x – 6 …… ➂ =1 Find the equation of the straight line that passes through point P and is parallel to the given straight line. 5.2
Dari ➁: y= 6 9
y
4 y x
=– +1
CONTOH 1. P(5, –4) , x – = 1
Gantikan ➂ ke dalam ➀. 9 6 3 6
3 P(–6, 1) , 4x + 3y = 10 x – y
4x + 3  x – 6 = 12 3
y = – x + 9 …… ➁ =1 Cara lain untuk
4 2 4x + 3y = 10 3 6 mencari nilai m:
y
4x + 9 x – 18 = 12 Gantikan ➁ ke dalam ➀. 3y = –4x + 10 – =–x +1 Diberi x –
y
= 1:
4 6 3 3 6
25 x = 30 10 – 3 x + 9 – 4x + 5 = 0 y = – 4 x + 10 y = 2x – 6 (–6)
2 3 3 m=– =2
3
4
–15x + 90 – 4x + 5 = 0 Maka, m = – 4 Maka, m = 2
x = 24 –19x = –95 3
5 Persamaan garis lurus yang dicari ialah
x=5 Persamaan garis lurus yang dicari ialah
Gantikan x = 24 ke dalam ➂. y – (–4) = 2(x – 5)
5 Gantikan x = 5 ke dalam ➁. y – 1 = – 4 [x – (–6)]
3 y + 4 = 2x – 10
y = 3  24  – 6 = – 12 y = – 3 (5) + 9 = 3
4 5 5 2 2 y – 1 = – 4x – 8 y = 2x – 14
3
Titik persilangan ialah  24 , – 12  . Titik persilangan ialah 5, 3 .
5 5 2 y = – 4x – 7
3
71 72

18
2. P(8, 5) , 2y – x + 4 = 0 3. P(–1, –2) , 5x + 7y = 9 D. Selesaikan masalah yang berikut.
Solve the following problems. 5.5
2y – x + 4 = 0 5x + 7y = 9
2y = x – 4 7y = –5x + 9 1. Rajah di bawah menunjukkan segi empat tepat PQRS. Titik S terletak pada paksi-y. Persamaan garis
lurus PQ ialah 2y – x = 11.
1 y=–5x+ 9
y= x–2 The diagram shows a rectangle PQRS. Point S lies on the y-axis. The equation of the straight line PQ is 2y – x = 11.
2 7 7
y
Maka, m = 1 Maka, m = – 5 Q
2 7
Persamaan garis lurus yang dicari ialah Persamaan garis lurus yang dicari ialah
y – 5 = 1 (x – 8) y – (–2) = – 5 [x – (–1)] P R(12, 4)
2 7
y–5= 1x–4 y+2=–5x– 5
x
O
2 7 7 S

y= 1x+1 y = – 5 x – 19
2 7 7 (a) Cari persamaan garis lurus QR. (c) Hitung luas segi empat tepat PQRS.
Find the equation of the straight line QR. Calculate the area of the rectangle PQRS.

C. Cari persamaan garis lurus PQ dalam setiap yang berikut. (b) Cari koordinat titik Q.
Find the coordinates of point Q.
Find the equation of the straight line PQ in each of the following. 5.4

CONTOH 1. y (a) Persamaan garis lurus PQ: 2y – x = 11


1 11
y Bagi 2y – 3x + 5 = 0: y= x+
2 2
P 2 – 3x + 5 = 0
2y y + 2x = 9 Bagi y + 2x = 9:
y= 3x– 5 y = –2x + 9 1
Maka, mPQ = dan mQR = –2.
2 2 x 2
O m = –2
m= 3
Q
(6,
(6,1) Persamaan garis lurus QR ialah y – 4 = –2(x – 12)
x 2 mPQ = 1 y – 4 = –2x + 24
O 2
Q mPQ = – 2 P (2, −7)
y = –2x + 28
3
(b) Q ialah titik persilangan PQ dan QR.
Persamaan garis lurus PQ ialah m2 = – 1
Persamaan garis lurus PQ ialah 2y – x = 11 …… ➀
m1 1
y – 1 = – 2 (x – 6) y – (–7) = (x – 2) y = –2x + 28 …… ➁
3 2
Gantikan ➁ ke dalam ➀.
y–1=–2x+4 y+7= 1x–1 2(–2x + 28) – x = 11
3 2
–4x + 56 – x = 11
y=–2x+5 y= 1x–8 –5x = –45
3 2
x=9
2. y 3. y
Gantikan x = 9 ke dalam ➁.
6 R P mRS = – 6 = – 3 y = –2(9) + 28 = 10
P(−5, 8) 8 4
Koordinat titik Q ialah (9, 10).
mPQ = 4
3
x – 3y + 12 = 0 Q
Q S
x (c) PQ dan SR adalah selari. Maka, mSR = 1
x O 2
O 8 s–4 = 1
0 – 12 2
Bagi x – 3y + 12 = 0: Titik tengah RS =  0 + 8 , 6 + 0  s – 4 = –6
2 2
y= 1x+4 = (4, 3)
s = –2
3
Koordinat titik S ialah (0, –2).
m= 1 Persamaan garis lurus PQ ialah
3 Luas segi empat tepat PQRS = 2 × Luas segi tiga QRS
mPQ = –3 y – 3 = 4 (x – 4)
3
 
1 0 12 9 0
=2×
Persamaan garis lurus PQ ialah y – 3 = 4 x – 16 2 –2 4 10 –2
y – 8 = –3[x – (–5)] 3 3
=  (0 + 120 – 18) – (–24 + 36 + 0) 
y – 8 = –3x – 15 y= 4x– 7 = 90 unit2
y = –3x – 7 3 3

73 74

2. Rajah di sebelah menunjukkan sebatang jalan lurus AB yang diwakili oleh Rumah Phua 6.6 Lokus SPM K1 ’13 K2 ’14, ’15, ’16

persamaan 3y + 2x = 16. Sebuah stesen petrol terletak di sepanjang jalan AB A Phua’s house
A. Cari persamaan lokus bagi titik bergerak P(x, y) yang memuaskan syarat yang diberikan.
dan pada jarak terdekat dari rumah Phua. (8, 13) Find the equation of the locus of a moving point P(x, y) that satisfies the given condition. 6.1
The diagram shows a straight road AB which is represented by the equation 3y + 2x = 16.
Jalan
A petrol station is located along the road AB and at the nearest distance from Phua’s house.
Road CONTOH
B
(a) Cari lokasi stesen petrol itu.
Find the location of the petrol station. (a) Titik P bergerak dengan keadaan jaraknya dari titik Q(5, –1) ialah 6 unit.
(b) Sebatang lampu isyarat terletak di antara rumah Phua dan stesen petrol itu dengan keadaan jarak rumah A point P moves such that its distance from point Q(5, –1) is 6 units.
Phua dari stesen petrol adalah 3 kali jaraknya dari lampu isyarat itu. Tentukan lokasi lampu isyarat itu. PQ = 6 unit
A traffic light is located in between Phua’s house and the petrol station where the distance of Phua’s house from the
petrol station is 3 times the distance from the traffic light. Determine the location of the traffic light. (x – 5)2 + [y – (–1)]2 = 6
(c) Hitung jarak di antara lampu isyarat dan rumah Phua jika 1 unit mewakili 50 m. (x – 5)2 + (y + 1)2 = 36 Kuasa duakan kedua-dua belah persamaan.
Calculate the distance between the traffic light and Phua’s house if 1 unit represents 50 m. x2 – 10x + 25 + y2 + 2y + 1 = 36
KBAT
x2 + y2 – 10x + 2y – 10 = 0

(b) Titik P bergerak dengan keadaan jaraknya dari titik A(6, 3) dan B(0, –7) adalah sama.
(a) Katakan P = Rumah Phua A point P moves such that it is equidistant from the points A(6, 3) and B(0, –7).
dan S = Stesen petrol
Daripada 3y + 2x = 16 …… ➀ PA = PB
y = – 2 x + 16 (x – 6)2 + (y – 3)2 = (x – 0)2 + [y – (–7)]2
3 3 (x – 6) + (y – 3)2 = x2 + (y + 7)2
2

Maka, mAB = – 2 dan mPS = 3 . x2 – 12x + 36 + y2 – 6y + 9 = x2 + y2 + 14y + 49


3 2 12x + 20y + 4 = 0
Persamaan garis lurus PS ialah y – 13 = 3 (x – 8) 3x + 5y + 1 = 0 Permudahkan.
2
y – 13 = 3 x – 12 1. Titik P bergerak dengan keadaan jaraknya dari 2. Diberi titik bergerak P dan titik T(1, 4) dengan
2
titik Q(–2, 9) ialah 4 unit. keadaan PT = 7 unit.
y = 3 x + 1 …… ➁ A point P moves such that its distance from point Given a moving point P and a point T(1, 4) is such that
2 Q(–2, 9) is 4 units. PT = 7 units.
Gantikan ➁ ke dalam ➀.
3 3 x + 1 + 2x = 16 PQ = 4 unit PT = 7 unit
2 [x – (–2)]2 + (y – 9)2 = 4 (x – 1)2 + (y – 4)2 = 7
13 x = 13
(x + 2)2 + (y – 9)2 = 16 (x – 1)2 + (y – 4)2 = 49
2
x=2 x2 + 4x + 4 + y2 – 18y + 81 = 16 x2 – 2x + 1 + y2 – 8y + 16 = 49
x2 + y2 + 4x – 18y + 69 = 0 x2 + y2 – 2x – 8y – 32 = 0
Gantikan x = 2 ke dalam ➁.
y = 3 (2) + 1 = 4
2
Maka, lokasi stesen petrol diwakili oleh koordinat (2, 4).
P(8, 13)
(b) Katakan T = Lampu isyarat
1
PS = 3PT, maka PT : TS = 1 : 2 3. Titik P bergerak dengan keadaan jaraknya dari 4. Titik P bergerak dengan keadaan jaraknya dari
T titik A(–3, 0) dan B(8, 1) adalah sama. titik A(–1, –4) dan B(3, –2) adalah sama.
Koordinat titik T =  2(8) + 1(2) , 2(13) + 1(4)  2 A point P moves such that it is equidistant from the A point P moves such that it is equidistant from the
2+1 2+1 points A(–3, 0) and B(8, 1). points A(–1, –4) and B(3, –2).
=  18 , 30  S(2, 4)
3 3 PA = PB PA = PB
= (6, 10) [x – (–3)]2 + (y – 0)2 = (x – 8)2 + (y – 1)2 [x – (–1)]2 + [y – (–4)]2 = (x – 3)2 + [y – (–2)]2
Maka, lokasi lampu isyarat diwakili oleh koordinat (6, 10). (x + 3)2 + y2 = (x – 8)2 + (y – 1)2 (x + 1)2 + (y + 4)2 = (x – 3)2 + (y + 2)2
x2 + 6x + 9 + y2 = x2 – 16x + 64 + y2 – 2y + 1 x2 + 2x + 1 + y2 + 8y + 16 = x2 – 6x + 9 + y2 + 4y + 4
(c) PT = (8 – 6)2 + (13 – 10)2 22x + 2y – 56 = 0 8x + 4y + 4 = 0
= 13 11x + y – 28 = 0 2x + y + 1 = 0
= 3.606 unit
Jarak di antara lampu isyarat dan rumah Phua = 3.606 × 50 m
= 180.3 m

75 76

19
B. Cari persamaan lokus bagi titik bergerak P(x, y) yang memuaskan syarat yang diberikan. C. Selesaikan masalah yang berikut.
Find the equation of the locus of a moving point P(x, y) that satisfies the given condition. 6.1 (b) Solve the following problems. 6.2

CONTOH CONTOH
Diberi dua titik Q(4, 3) dan R(–1, 0), titik P(x, y) bergerak dengan keadaan PQ : PR = 2 : 1. Suatu titik P bergerak dengan keadaan jaraknya dari titik S(–3, 5) adalah dua kali jaraknya dari titik T(–2, –1).
Given two points Q(4, 3) and R(–1, 0), a point P(x, y) moves such that PQ : PR = 2 : 1. A point P moves such that its distance from point S(–3, 5) is twice its distance from point T(–2, –1).
(a) Cari persamaan lokus bagi titik P.
PQ = 2 Find the equation of the locus of point P.
PR 1 (b) Tentukan sama ada lokus bagi titik P menyilang paksi-x atau tidak.
PQ = 2PR Determine whether the locus of point P intersects the x-axis.

(x – 4)2 + (y – 3)2 = 2 [x – (–1)]2 + (y – 0)2 (a) Katakan koordinat titik P ialah (x, y). (b) Pada paksi-x, y = 0.
(x – 4)2 + (y – 3)2 = 4[(x + 1)2 + y2] PS = 2PT Gantikan y = 0 ke dalam persamaan
x – 8x + 16 + y2 – 6y + 9 = 4(x2 + 2x + 1 + y2)
2 2 2
[x – (–3)] + (y – 5) = 2 [x – (–2)] + [y – (–1)]2 2 lokus titik P.
Maka, 3x2 + 10x – 14 = 0
x2 + y2 – 8x – 6y + 25 = 4x2 + 4y2 + 8x + 4 (x + 3)2 + (y – 5)2 = 4[(x + 2)2 + (y + 1)2] b2 – 4ac = 102 – 4(3)(–14)
3x2 + 3y2 + 16x + 6y – 21 = 0 x2 + 6x + 9 + y2 – 10y + 25 = 4(x2 + 4x + 4 + y2 + 2y + 1) = 268 ( 0)
x2 + y2 + 6x – 10y + 34 = 4x2 + 4y2 + 16x + 8y + 20 Maka, lokus bagi titik P menyilang
Persamaan lokus bagi titik P ialah 3x2 + 3y2 + 16x + 6y – 21 = 0. 3x2 + 3y2 + 10x + 18y – 14 = 0 paksi-x pada dua titik yang berlainan.
1. Diberi dua titik M(1, 5) dan N(–2, –3), titik P(x, y) bergerak dengan keadaan PM : PN = 3 : 2. 1. Diberi dua titik E(–7, 4) dan H(1, 6), titik P(x, y) bergerak dengan keadaan lokusnya adalah pembahagi
Given two points M(1, 5) and N(–2, –3), a point P(x, y) moves such that PM : PN = 3 : 2. dua sama serenjang bagi garis lurus EH.
PM 3 Given two points E(–7, 4) and H(1, 6), a point P(x, y) moves such that its locus is the perpendicular bisector of the
= straight line EH.
PN 2
(a) Cari persamaan lokus bagi titik P.
2PM = 3PN Find the equation of the locus of point P.
2 (x – 1)2 + (y – 5)2 = 3 [x – (–2)]2 + [y – (–3)]2 (b) Tentukan sama ada lokus bagi titik P selari atau berserenjang dengan garis lurus 4y – x = 5.
Determine whether the locus of point P is parallel or perpendicular to the straight line 4y – x = 5.
4[(x – 1)2 + (y – 5)2] = 9[(x + 2)2 + (y + 3)2]
(a) Lokus titik P ialah pembahagi dua sama serenjang (b) Bagi 4x + y + 7 = 0:
4(x – 2x + 1 + y2 – 10y + 25) = 9(x2 + 4x + 4 + y2 + 6y + 9)
2
bagi EH. Maka, y = –4x – 7 → m1 = –4
4x2 – 8x + 4 + 4y2 – 40y + 100 = 9x2 + 36x + 36 + 9y2 + 54y + 81 PE = PH
Bagi 4y – x = 5:
4x2 + 4y2 – 8x – 40y + 104 = 9x2 + 9y2 + 36x + 54y + 117 [x – (–7)]2 + (y – 4)2 = (x – 1)2 + (y – 6)2
(x + 7) + (y – 4)2 = (x – 1)2 + (y – 6)2
2 y = 1 x + 5 → m2 = 1
5x2 + 5y2 + 44x + 94y + 13 = 0 4 4 4
x2 + 14x + 49 + y2 – 8y + 16 = x2 – 2x + 1 + y2 – 12y + 36
m1 m2 = –4 × 1 = –1
Persamaan lokus bagi titik P ialah 5x2 + 5y2 + 44x + 94y + 13 = 0. 16x + 4y + 28 = 0 4
4x + y + 7 = 0 Maka, lokus bagi titik P berserenjang
dengan 4y – x = 5.
2. Titik P(x, y) bergerak dengan keadaan jaraknya dari titik A(–7, 2) adalah tiga kali jaraknya dari titik
B(0, 3). 2. Diberi dua titik S(4, –7) dan T(8, 1), titik P(x, y) bergerak dengan keadaan ∠SPT = 90°.
A point P(x, y) moves such that its distance from point A(–7, 2) is thrice its distance from point B(0, 3). Given two points S(4, –7) and T(8, 1), a point P(x, y) moves such that ∠SPT = 90°.
(a) Cari persamaan lokus bagi titik P.
PA = 3PB Find the equation of the locus of point P.
[x – (–7)]2 + (y – 2)2 = 3 (x – 0)2 + (y – 3)2 (b) Tentukan sama ada lokus bagi titik P menyilang paksi-y atau tidak.
Determine whether the locus of point P intersects the y-axis.
(x + 7)2 + (y – 2)2 = 9[x2 + (y – 3)2]
(a) Apabila ∠SPT = 90°, PS berserenjang dengan PT. (b) Pada paksi-y, x = 0.
x + 14x + 49 + y2 – 4y + 4 = 9(x2 + y2 – 6y + 9)
2
Maka, mPS × mPT = –1
x + y2 + 14x – 4y + 53 = 9x2 + 9y2 – 54y + 81
2 Gantikan x = 0 ke dalam persamaan
y – (–7)
8x2 + 8y2 – 14x – 50y + 28 = 0  x – 4  xy –– 18  = –1 lokus titik P.
Maka, y2 + 6y + 25 = 0
4x2 + 4y2 – 7x – 25y + 14 = 0 (y + 7)(y – 1) = –1
b2 – 4ac = 62 – 4(1)(25)
(x – 4)(x – 8)
= –64 ( 0)
Persamaan lokus bagi titik P ialah 4x2 + 4y2 – 7x – 25y + 14 = 0. y2 + 6y – 7 = –1
x2 – 12x + 32 Maka, lokus bagi titik P tidak
y2 + 6y – 7 = –x2 + 12x – 32 menyilang paksi-y.
x2 + y2 – 12x + 6y + 25 = 0

77 78

PRAKTIS FORMATIF Kertas 1 ANALISIS SOALAN SPM 4. Rajah di bawah menunjukkan garis lurus AB. 6. Persamaan suatu garis lurus diberi oleh qx – py = 1,
Subtopik 2013 2014 2015 2016 KLON The diagram shows a straight line AB. KLONdengan keadaan p dan q ialah pemalar. Cari dalam
SPM SPM

Jawab semua soalan.


6.1 – – S. 12 – ’13 ’15 sebutan p dan q,
6.2 S. 13(a) S. 12(b) – – y
The equation of a straight line is given by qx – py = 1, where
Answer all the questions. 6.3 – – – –
6.4 – S. 12(a) S. 11(a) S. 8, 9(b) p and q are constants. Find in terms of p and q,
1. Rajah di bawah menunjukkan kedudukan dua ekor 6.5 S. 13(b) S. 13 S. 11(b) S. 9(a) A(0, 6) (a) kecerunan garis lurus itu.
6.4
5.2
KLON lebah, P dan Q.
6.6 S. 14 – – – the gradient of the straight line.
SPM
’15 The diagram shows the positions of two bees, P and Q. M (b) kecerunan garis lurus yang berserenjang dengan
6.1
5.2
y
2. Rajah di bawah menunjukkan garis lurus PQ dengan 6.5
5.2
garis qx – py = 1.
KBAT
KLON
SPM persamaan
x y B(2, 0)
x the gradient of the straight line which is perpendicular to
+ = 1 bersilang dengan garis lurus O
’14 7 5 the line qx – py = 1.
AB pada titik P. [2]
Lebah P
The diagram shows a straight line PQ with an equation of Diberi M ialah titik tengah AB. Cari
Bee P
x y Given M is the midpoint of AB. Find (a) qx – py = 1
+ = 1 intersects a straight line AB at point P.
7 5 (a) koordinat titik M. py = qx – 1
6.2
5.2
x y the coordinates of point M. q 1
O B
(b) persamaan garis lurus yang berserenjang dengan y= x–
Q 6.5
5.2
p p
AB dan melalui M. q
Lebah Q the equation of the straight line which is perpendicular to Maka, kecerunan garis lurus =
Bee Q
AB and passing through M.
p
x
O P(7, 0) [4] 1 p
(b) Kecerunan = – q = –
A(4, –3) q
Koordinat bagi lebah P dan lebah Q masing-masing (a) Koordinat titik M =  0 + 2 , 6 + 0  p
(a) Nyatakan pintasan-y bagi PQ. 2 2
adalah (4, 4) dan (–8, –12). Kedua-dua lebah itu 6.4 State the y-intercept of PQ. = (1, 3)
terbang ke arah satu sama lain pada satu garis lurus
(b) Cari koordinat bagi titik B jika BP = 2PA.
dengan halaju berbeza. Halaju lebah Q adalah tiga kali 6.2 Find the coordinates of point B if BP = 2PA. 7. Rajah di bawah menunjukkan garis lurus AB.
halaju lebah P. Cari jarak lebah Q dari titik asalnya 6–0
[3] (b) Kecerunan AB = = –3 The diagram shows a straight line AB.
apabila bertemu dengan lebah P. 0–2
The coordinates of bee P and bee Q are (4, 4) and (–8, –12) (a) Pintasan-y = 5 y
Kecerunan garis lurus yang berserenjang dengan B(14, 8)
respectively. Both bees fly towards each other on a straight line
with different velocities. The velocity of bee Q is three times (b) Katakan koordinat titik B ialah (x, y). AB ialah 1 . A(–2, 4)
y 3
the velocity of bee P. Find the distance of bee Q from its initial (7, 0) =  x + 2(4) , + 2(–3) 
point when it meets with bee P. 1+2 1+2 Persamaan garis lurus yang berserenjang dengan x
[3] y 0
= x + 8 , – 6  AB dan melalui M ialah
Cari
3 3
Andaikan lebah P dan lebah Q bertemu di titik M. y – 3 = 1 (x – 1) Find
Maka, jarak QM adalah 3 kali jarak PM. x+8=7 y–6 =0 3 (a) titik tengah AB.
Maka, dan
3 3 y–3= 1x– 1 6.2 the midpoint of AB.
1 P(4, 4) x + 8 = 21 y–6=0 3 3 (b) persamaan pembahagi dua sama serenjang bagi
x = 13 y=6 y= 1x+ 8
M(x, y) 6.5
garis AB.
3 3 3 the equation of the perpendicular bisector of the line AB.
Koordinat titik B ialah (13, 6). [4]
–2 + 14 4 + 8
(a) Titik tengah AB = 
2 
Q(–8, –12) ,
2
= (6, 6)
M(x, y) =  3(4) + (1)(–8) , 3(4) + (1)(–12)  3. Garis lurus 3y = 5x + h – 6 menyilang paksi-x 5. Garis lurus y = 3x + 8 adalah selari dengan garis lurus
3+1 3+1 KLONpada 4k, dengan keadaan h dan k ialah pemalar.
SPM
KLONy = (k – 2)x + 7, dengan keadaan k ialah pemalar. (b) mAB = 8–4 = 4 = 1
=  12 – 8 , 12 – 12 
SPM
’16 Ungkapkan h dalam sebutan k. ’14 Tentukan nilai k. 14 – (–2) 16 4
4 4 6.4
5.2
The straight line 3y = 5x + h – 6 intersects the x-axis at 4k, 6.5
5.2
The straight line y = 3x + 8 is parallel to the straight line
= (1, 0) where h and k are constants. Express h in terms of k. y = (k – 2)x + 7, where k is a constant. Determine the value of k.
Kecerunan pembahagi dua sama serenjang bagi
[2] [2] garis AB ialah –4.
Jarak QM = [1 – (–8)]2 + [0 – (–12)]2 Pada titik (4k, 0), 3(0) = 5(4k) + h – 6 k–2=3 Persamaan pembahagi dua sama serenjang bagi
0 = 20k + h – 6 k=5 garis AB ialah
= 92 + 122
h = 6 – 20k y – 6 = –4(x – 6)
= 225 y – 6 = –4x + 24
= 15 unit y + 4x = 30

79 80

20
y
– x = 1 memintas paksi-x di P dan 10. Rajah di bawah menunjukkan dua garis lurus yang
8. Garis lurus
4 8 KLONbersilang di titik M.
PRAKTIS FORMATIF Kertas 2 ANALISIS SOALAN SPM
SPM Subtopik 2013 2014 2015 2016
paksi-y di Q. Cari ’16 The diagram shows two straight lines intersecting at point M. 6.1 – – – –
y x Jawab semua soalan. 6.2 – – S. 3(a)(ii) S. 11(a)(i)
A straight line – = 1 intersects the x-axis at P and the y Answer all the questions. 6.3 S. 9(d) – S. 3(a)(i) S. 11(a)(iii)
4 8
6.4 S. 9(a),(c) S. 10(a)(ii) – S.11(a)(ii)
y-axis at Q. Find
1. Rajah di bawah menunjukkan sisi empat PQRS. Garis 6.5 S. 9(b) S. 10(a)(i) – –
(a) kecerunan garis lurus itu. 6.6 – S. 10(b) S. 3(b) S. 11(b)
6.4 the gradient of the straight line. lurus QR adalah berserenjang dengan garis lurus RS.
y = 4 – qx
(b) persamaan pembahagi dua sama serenjang bagi Titik T terletak pada garis lurus RS.
The diagram shows a quadrilateral PQRS. The straight line (d) Katakan koordinat titik P ialah (h, k).
6.5
garis PQ. x QR is perpendicular to the straight line RS. Point T lies on the Luas sisi empat PQRS = 30 unit2
the equation of the perpendicular bisector of the line PQ. O

 
M straight line RS. 1 6 h 6 10 6
[3] 1
y=–x–2 = 30
2 y 2 8 k –2 6 8
Q(6, 8) (6k – 2h + 36 + 80) – (8h + 6k – 20 + 36) = 60
4
(a) Kecerunan = –   = 1 Dua garis lurus itu berserenjang antara satu sama lain. R(10, 6)
–10h + 100 = 60
–8 2 P 10h = 40
The two straight lines are perpendicular to each other.
2y + 7x h=4
(b) Koordinat titik P ialah (–8, 0). (a) Nyatakan nilai q. = 38 T
Koordinat titik Q ialah (0, 4). 6.5 State the value of q. Titik P terletak pada PS, maka 2k + 7h = 38.
(b) Cari koordinat titik M. x Apabila h = 4, 2k + 7(4) = 38
1 O
Kecerunan PQ = 6.4 Find the coordinates of point M. 2k = 10
2 [3] S
k=5
Titik tengah PQ =  –8 + 0 , 0 + 4  (a) Dua garis lurus itu adalah berserenjang.
Cari
2 2 Find Koordinat titik P ialah (4, 5).
= (–4, 2) Maka, m1 m2 = –1 (a) persamaan garis lurus RS.
1 (–q) = –1 6.5 the equation of the straight line RS. [3] 2. Rajah di bawah menunjukkan garis lurus AB.
Kecerunan garis lurus yang berserenjang dengan 2 (b) koordinat bagi S. The diagram shows a straight line AB.
garis PQ ialah –2. q=2 6.4 the coordinates of S. [2] y
(c) koordinat bagi T jika RT : TS = 2 : 3. y + 2x + 6 = 0
Persamaan pembahagi dua sama serenjang bagi (b) Persamaan dua garis lurus itu ialah
garis PQ ialah
6.2 the coordinates of T if RT : TS = 2 : 3. [2] B
x
y – 2 = –2(x + 4) y= 1x–2 …… ➀ (d) koordinat bagi P jika luas sisi empat PQRS ialah O
2 6.3
30 unit2. P
y – 2 = –2x – 8 y = 4 – 2x …… ➁ the coordinates of P if the area of the quadrilateral PQRS
y = –2x – 6 is 30 unit2. [3]
➀ = ➁: 1 x – 2 = 4 – 2x A
2 (a) mQR = 8 – 6 = 2 = – 1
1 x + 2x = 4 + 2 6 – 10 –4 2 Titik P terletak pada AB dengan AP : PB = 2 : 1.
2 Maka, mRS = 2 Point P lies on AB such that AP : PB = 2 : 1.
5x=6 (a) Cari koordinat titik P.
Persamaan garis lurus RS ialah
2 6.2 Find the coordinates of point P. [3]
x = 12 y – 6 = 2(x – 10)
5 y – 6 = 2x – 20 (b) Cari persamaan garis lurus yang melalui titik P
y = 2x – 14
6.5
dan berserenjang dengan AB.
=22 Find the equation of the straight line that passes through
5
(b) Bagi PS: 2y + 7x = 38 ...... ➀ point P and is perpendicular to AB. [3]
9. Titik B ialah (4, 0). Titik P(x, y) bergerak dengan Gantikan x = 12 ke dalam ➁.
5 Bagi RS: y = 2x – 14 ...... ➁
KLON keadaan PB = 2 unit. Cari persamaan lokus bagi P. (a) Pada titik A: x = 0, y + 6 = 0
y = 4 – 2 12  = – 4 Gantikan ➁ ke dalam ➀.
SPM
’13 The point B is (4, 0). A point P(x, y) moves such that y = –6
6.6
5.2 5 5 2(2x – 14) + 7x = 38
PB = 2 units. Find the equation of the locus of P.
4x – 28 + 7x = 38 Pada titik B: y = 0, 2x + 6 = 0
[3] Koordinat titik M ialah 2 2 , – 4 . 2x = –6
5 5 11x = 66
PB = 2 unit x=6 x = –3
Persamaan lokus bagi P ialah Gantikan x = 6 ke dalam ➁. Koordinat titik A ialah (0, –6).
y = 2(6) – 14 = –2 Koordinat titik B ialah (–3, 0).
(x – 4)2 + (y – 0)2 = 2
x2 – 8x + 16 + y2 = 4 Koordinat titik S ialah (6, –2). Koordinat titik P =  2(–3) + 0 , 0 + 1(–6) 
x2 + y2 – 8x + 16 – 4 = 0 2+1 2+1
x2 + y2 – 8x + 12 = 0
(c) Koordinat titik T =  2(6) + 3(10) , 2(–2) + 3(6)  =  –6 , –6 
2+3 2+3 3 3
= (–2, –2)
=  42 , 14 
5 5

81 82

0 – (–6) 4. Rajah di bawah menunjukkan segi empat tepat ABCD. 5. Rajah di bawah menunjukkan sisi empat PQRS. (b) Pada titik S, y = 1.
(b) mAB = = –2
–3 – 0 KLON
SPM
Persamaan garis lurus AB ialah y = 3x + 2. KLON
SPM
Titik R terletak pada paksi-y. Gantikan y = 1 ke dalam ➁.
Kecerunan garis lurus yang berserenjang dengan ’13The diagram shows a rectangle ABCD. The equation of the 14 The diagram shows a quadrilateral PQRS. Point R lies on the 2(1) = 5x – 23
KBAT straight line AB is y = 3x + 2. y-axis. 5x = 25
AB = 1 y y
x=5
2 y=1
B R S
Persamaan garis lurus yang melalui titik P dan Koordinat titik S ialah (5, 1).
berserenjang dengan AB ialah C(8, 6) x
O
Katakan koordinat titik T ialah (x, y) dan
y – (–2) = 1 [x – (–2)] Q(–2, –2) ST = 6 unit.
2
y + 2 = 1 (x + 2) O
x Persamaan lokus bagi T ialah
2 A(–1, –1)
P
(y – 1)2 + (x – 5)2 = 62
y= 1x–1
D
2 Cari Persamaan garis lurus PS ialah 2y = 5x – 23. y2 – 2y + 1 + x2 – 10x + 25 = 36
The equation of the straight line PS is 2y = 5x – 23.
Find x2 + y2 – 10x – 2y – 10 = 0
3. Rajah di bawah menunjukkan garis lurus AB. (a) persamaan garis lurus CD. (a) (i) Cari persamaan garis lurus PQ.
The diagram shows a straight line AB. 6.5 Find the equation of the straight line PQ.
6.4 the equation of the straight line CD. [2]
y (b) persamaan garis lurus AD. (ii) Cari koordinat bagi titik P.
6.4 Find the coordinates of point P.
A(6, 2) 6.5 the equation of the straight line AD. [3]
[6] 6. Rajah di bawah menunjukkan sebuah tapak
(c) koordinat bagi D.
x KLONperkhemahan OPQR yang telah dibersihkan oleh
O 6.4 the coordinates of D. [2] (b) Titik T bergerak dengan keadaan jaraknya dari SPM

B(–3, –4) (d) luas, dalam unit2, segi empat tepat ABCD. 6.6 titik S sentiasa 6 unit. Cari persamaan lokus ’15 sekumpulan pengakap. Titik-titik A, B dan C masing-
6.3 the area, in unit2, of the rectangle ABCD. [3] bagi T. KBAT masing adalah pusat khemah A, B dan C. A, B dan C

(a) mCD = mAB = 3 A point T moves such that its distance from point S is adalah segaris.
(a) Hitung luas segi tiga AOB. always 6 units. Find the equation of the locus of T. The diagram shows a campsite OPQR that had been cleared
6.3 Calculate the area of triangle AOB. [2] Persamaan garis lurus CD ialah [4] by a group of scouts. Points A, B and C are the centres of the
(b) Titik C membahagi dalam garis lurus AB dengan y – 6 = 3(x – 8) tents A, B and C respectively. A, B and C are collinear.
6.2
nisbah AC : CB = 2 : 3. Cari koordinat bagi C. y – 6 = 3x – 24 (a) (i) Persamaan garis lurus PS:
y (m)
A point C divides the straight line AB internally in the ratio
y = 3x – 18 2y = 5x – 23
AC : CB = 2 : 3. Find the coordinates of C. [2] Q(8, 14)
(c) Titik P bergerak dengan keadaan jaraknya dari (b) Kecerunan garis lurus AD = –
1 y = 5 x – 23
3 2 2
6.6
B adalah sentiasa dua kali jaraknya dari A. Cari
persamaan lokus bagi P. Persamaan garis lurus AD ialah Kecerunan garis lurus PS, mPS = 5 C(7, 9)
A point P moves such that its distance from B is always 1 2
y – (–1) = – [x – (–1)] Maka, mPQ = – 2
twice its distance from A. Find the equation of the 3 5
locus of P. [3] 1 B(1, 5) R(12, 5)
y+1=– x–1 Persamaan garis lurus PQ: P(–5, 4) A
3
 
1 0 –3 6 0 1 4
(a) Luas segi tiga AOB =
2 0 –4 2 0 y=– x– y – (–2) = – 2 [x – (–2)] x (m)
3 3 5 O
1
=  –6 – (–24) 
2 (c) y = 3x – 18 ...... ➀ y + 2 = – 2 (x + 2)
1 4 5 Diberi jarak titik C adalah 2 kali dari titik B dan
= 9 unit2 y=– x– ...... ➁ 5y + 10 = –2x – 4 3 kali dari titik A.
3 3
5y = –2x – 14 Given the distance of point C is 2 times from point B and
(b) Koordinat titik C =  3(6) + 2(–3) , 3(2) + 2(–4)  10 50 3 times from point A.
3+2 3+2 ➀ – ➁: 0= x– (ii) P ialah titik persilangan garis lurus PQ dan
3 3 (a) Cari
=  12 , – 2  10 50 garis lurus PS. Find
5 5 x= (i) luas, dalam m2, tapak perkhemahan OPQR.
3 3 Bagi PQ: 5y = –2x – 14 ...... ➀
(c) Katakan koordinat titik P ialah (x, y). x=5 6.3 the area, in m2, of the campsite OPQR.
Bagi PS: 2y = 5x – 23 ...... ➁
PB = 2PA Gantikan x = 5 ke dalam ➀. (ii) koordinat bagi A.
Persamaan lokus bagi P ialah y = 3(5) – 18 = –3 ➀ × 2: 10y = –4x – 28 ...... ➂ 6.2 the coordinates of A.
➁ × 5: 10y = 25x – 115 ...... ➃ [4]
(x + 3) 2 + (y + 4)2 = 2 (x – 6) 2 + (y – 2)2 Koordinat titik D ialah (5, –3). ➃ – ➂: 0 = 29x – 87 (b) Seorang pengakap menabur serbuk belerang
x2 + 6x + 9 + y2 + 8y + 16 (d) Luas segi empat tepat ABCD 29x = 87 6.6
di sekeliling khemah C dengan keadaan jarak
= 4(x2 – 12x + 36 + y 2 – 4y + 4) = 2 × Luas ΔACD x=3 laluan serbuk belerang dari pusat khemah C
x2 + y2 + 6x + 8y + 25 Gantikan x = 3 ke dalam ➀. adalah sentiasa 2 m. Cari persamaan laluan
 
1 –1 5 8 –1
=2× 5y = –2(3) – 14 serbuk belerang itu.
= 4x2 – 48x + 144 + 4y2 – 16y + 16 2 –1 –3 6 –1
= 4x2 + 4y2 – 48x – 16y + 160 = | (3 + 30 – 8) – (–5 – 24 – 6) | 5y = –20 A scout spread sulphur powder around tent C such that
y = –4 the distance of the sulphur powder track from the centre
2 2
3x + 3y – 54x – 24y + 135 = 0 = | 25 + 35 | of tent C is always 2 m. Find the equation of the track of
= 60 unit2 Koordinat titik P ialah (3, –4). the sulphur powder. [3]

83 84

21
(a) (i) Luas OPQR (b) Titik B bergerak dengan keadaan sentiasa sama
jarak dari titik Q dan titik R. Cari persamaan FOKUS KBAT
 
6.6
1 0 12 8 –5 0
=
2 0 5 14 4 0 lokus bagi B.
1 A point B moves such that it is always equidistant from
= | (0 + 168 + 32 – 0) – (0 + 40 – 70 + 0) |
2 the points Q and R. Find the equation of the locus of B. Kemahiran Kognitif: Menganalisis
1 | [3] Konteks: Garis Lurus, Lokus
= 230 |
2
= 115 m2 (a) (i) Katakan koordinat titik P ialah (x, y). y Rajah di sebelah ialah satah Cartes yang menunjukkan kedudukan
(ii) Maka,  1(10) + 2x , 1(19) + 2y  = (0, 9) tiga orang murid. Shafiq berdiri pada suatu kedudukan yang paling
1+2 1+2 Ruthra dekat kepada Cheng Wei tetapi sama jarak dari Norita dan Ruthra.
C(7, 9)
2 10 + 2x = 0 dan 19 + 2y = 3 × 9 (2, 10) Cari kedudukan Shafiq.
Cheng Wei The diagram is a Cartesian plane which shows the positions of three students.
1 2x = –10 2y = 8 (12, 5)
B(1, 5) Shafiq stands at a position nearest to Cheng Wei but equidistant from Norita and
A(x, y) x = –5 y=4 Ruthra. Find the position of Shafiq.
Koordinat titik P ialah (–5, 4). x
(1, 5) =  2x + 7 , 2y + 9  Norita
O Info KBAT
3 3
2x + 7 = 1 dan 2y + 9 = 5 (ii) Kecerunan garis lurus PR = –5 – 4 (−4, −2)
Titik yang sentiasa sama jarak dari dua titik
4 – (–5) merupakan lokus pembahagi dua sama
3 3
2x + 7 = 3 2y + 9 = 15 = –9 serenjang bagi kedua-dua titik itu.
9 The point which is always equidistant from two
2x = –4 2y = 6 = –1 points is the locus of the perpendicular bisector
x = –2 y=3 of both points.
Persamaan garis lurus PR ialah
Maka, koordinat titik A ialah (–2, 3). y – (–5) = –1(x – 4)
y + 5 = –x + 4 Katakan: Kedudukan Norita = N(–4, –2) Persamaan garis lurus CS ialah
(b) Persamaan laluan serbuk belerang ialah y = –x – 1 Kedudukan Ruthra = R(2, 10) y – 5 = 2(x – 12)
(x – 7)2 + (y – 9)2 = 2 (iii) Luas segi tiga PQR y – 5 = 2x – 24
Kedudukan Cheng Wei = C(12, 5)
(x – 7)2 + (y – 9)2 = 4 y = 2x – 19 …… ➁
 
1 –5 4 10 –5 Kedudukan Shafiq = S(x, y)
x2 – 14x + 49 + y2 – 18y + 81 = 4 =
2 4 –5 19 4 Kedudukan Shafiq ialah titik persilangan antara
2 2
x + y – 14x – 18y + 126 = 0 1 Kecerunan garis lurus NR = 10 – (–2) garis lurus ➀ dan garis lurus ➁.
= |(25 + 76 + 40) – (16 – 50 – 95)| 2 – (–4)
2
Selesaikan ➀ dan ➁ secara serentak.
=
1
|141 – (–129)| = 12
2 6 Maka, 2x – 19 = – 1 x + 7
= 135 unit2 =2 2 2
Kecerunan pembahagi dua sama serenjang 5 x = 45
7. Rajah di bawah menunjukkan segi tiga PQR. Sisi PQ (b) Katakan koordinat titik B ialah (x, y). 2 2
KLON
SPM
bersilang dengan paksi-y pada titik A. BQ = BR bagi NR = – 1 5x = 45
’16 The diagram shows a triangle PQR. Side PQ intersects the
2 x=9
Persamaan lokus bagi B ialah
y-axis at point A.
Titik tengah NR =  2 + (–4) , 10 + (–2) 
y
(x – 10)2 + (y – 19)2 = (x – 4)2 + (y + 5)2 2 2 y = 2(9) – 19 = –1
x2 – 20x + 100 + y2 – 38y + 361 = (–1, 4) Kedudukan Shafiq ialah pada titik (9, –1).
Q(10, 19)
= x2 – 8x + 16 + y2 + 10y + 25 Persamaan pembahagi dua sama serenjang
A(0, 9)
x2 + y2 – 20x – 38y + 461 = x2 + y2 – 8x + 10y + 41 bagi NR ialah
12x + 48y – 420 = 0 y – 4 = – 1 [x – (–1)]
P x + 4y – 35 = 0 2
O x
y – 4 = – 1x – 1
2 2
R(4, −5)
y=–1x+ 7 …… ➀
(a) Diberi PA : AQ = 1 : 2, cari 2 2
Given that PA : AQ = 1 : 2, find
(i) koordinat bagi P.
6.2 the coordinates of P.
(ii) persamaan garis lurus PR.
6.4 the equation of the straight line PR.
(iii) luas, dalam unit2, segi tiga PQR.
6.3 the area, in unit2, of triangle PQR.
[7]

85 86

B. Cari min, mod dan median bagi setiap set data yang diberikan.
BAB 1.1 ; 1.2 ; 1.3

7
STATISTIK Find the mean, mode and median of each given set of data.

STATISTICS CONTOH
Σfx = 4(9) + 5(10) + 3(11) + 2(12) + 1(13) = 156
Σf = 4 + 5 + 3 + 2 + 1 = 15
Panjang (cm)
9 10 11 12 13 Σfx 156
7.1 Sukatan Kecenderungan Memusat SPM K1 ‘14, ‘15 K2 ’16
Length (cm) Min, x– = = = 10.4 cm
Σf 15
A. Cari min, mod dan median bagi setiap set data yang diberikan. Kekerapan 4 5 3 2 1 Mod = 10 cm
Find the mean, mode and median of each given set of data. 1.1 ; 1.2 ; 1.3 Frequency 15 + 1 = Nilai ke-8 = 10 cm
CONTOH
Median = Nilai ke- 
2 
1. 2, 5, 7, 6, 10, 7, 9, 8, 4
23, 20, 17, 18, 15, 17, 16, 20 1. Jisim (kg) 2. Diameter (cm)
N=9 35 40 45 50 55 13 14 15 16 17 18
Mass (kg) Diameter (cm)
N=8 ∑x = 2 + 5 + 7 + 6 + 10 + 7 + 9 + 8 + 4
Bilangan murid Bilangan silinder
= 58 3 4 2 7 6 5 6 8 7 3 2
Σx = 23 + 20 + 17 + 18 + 15 + 17 + 16 + 20 Number of students Number of cylinders

Min, x– = ∑x = 58 = 6.444
= 146
N 9 ∑ fx = 3(35) + 4(40) + 2(45) + 7(50) + 6(55) ∑fx = 5(13) + 6(14) + 8(15) + 7(16) + 3(17) + 2(18)
Min, x– = Σx = 146 = 18.25 = 1 035 = 468
N 8 Mod = 7
∑ f = 3 + 4 + 2 + 7 + 6 = 22 ∑ f = 5 + 6 + 8 + 7 + 3 + 2 = 31
Mod = 17 dan 20 Terdapat dua mod Susun semula semua nilai: ∑ fx
Min, x– = ∑ fx = 1 035 = 47.05 kg Min, x– = =
468 = 15.10 cm
Susun semula semua nilai mengikut tertib menaik: 2 , 4 , 5 , 6 , 7 , 7 , 8 , 9 , 10 ∑f 22 ∑f 31
Mod = 50 kg Mod = 15 cm
15 , 16 , 17 , 17 , 18 , 20 , 20 , 23
Median Median = Min nilai ke-11 dan nilai ke-12 = 50 kg Median = Nilai ke-16 = 15 cm
Median
Median = Nilai ke-5
=7 C. Selesaikan masalah yang berikut.
Median = Min nilai ke-4 dan nilai ke-5 Solve the following problems. 1.1 ; 1.2 ; 1.3

= 17 + 18 CONTOH
2 1. Diberi set data 3, 5, 2, 10, 12, 2. Diberi set data 8, 5, 12, 9, 4, m,
= 17.5 8, m, n, dengan mod = min = 8 1, 7 mempunyai min 7. Cari
Diberi set data 3, 6, m, 7, n, dengan
mod = min = 6 dan m ≠ n. dan m ≠ n. Given a set of data 8, 5, 12, 9, 4, m,
2. 8, 2, 11, 9, 4, 5, 13, 9 3. 4.6, 3.1, 4.3, 3.1, 4.3, 2.8, 2.1 Given a set of data 3, 6, m, 7, n, where Given a set of data 3, 5, 2, 10, 12, 8, 1, 7, has a mean of 7. Find
mode = mean = 6 and m ≠ n. m, n, where mode = mean = 8 and (a) nilai m.
N=8 N=7 m ≠ n. the value of m.
(a) Cari nilai m dan nilai n.
∑x = 8 + 2 + 11 + 9 + 4 + 5 + 13 + 9 ∑x = 4.6 + 3.1 + 4.3 + 3.1 + 4.3 + 2.8 + 2.1 Find the values of m and n. (a) Cari nilai m dan nilai n. (b) mod dan median bagi set
= 61 = 24.3 (b) Nyatakan mediannya. Find the values of m and n. data itu.
State the median. (b) Nyatakan mediannya. the mode and median of the set
Min, x– = ∑x = 61 = 7.625 Min, x– = ∑x = 24.3 = 3.471 State the median. of data.
N 8 N 7 (a) Diberi mod = 6.
Maka, m = 6 atau n = 6. (a) Diberi mod = 8. (a) Diberi min = 7.
Mod = 9 Mod = 3.1 dan 4.3 Maka, m = 8 atau n = 8. 8 + 5 + 12 + 9 + 4 + m + 1 + 7 = 7
Susun semula semua nilai: Jika m = 6, maka
Susun semula semua nilai: Jika m = 8, maka 8
3+6+6+7+n =6 46 + m = 56
2 , 4 , 5 , 8 , 9 , 9 , 11 , 13 2.1 , 2.8 , 3.1 , 3.1 , 4.3 , 4.3 , 4.6 5 3 + 5 + 2 + 10 + 12 + 8 + 8 + n m = 10
=8
22 + n = 30 8
Median n=8 48 + n = 64 (b) Susun semula set data
Median n = 16 mengikut tertib menaik:
Dengan cara yang sama, 1 , 4 , 5 , 7 , 8 , 9 , 10 , 12
Median = Min nilai ke-4 dan nilai ke-5 Dengan cara yang sama,
Median = Nilai ke-4 jika n = 6, m = 8.
jika n = 8, m = 16. Set data ini tiada mod sebab
=8+9 = 3.1
2 (b) Set data ialah 3, 6, 6, 7, 8. (b) Set data ialah tiada nilai yang berulang.
= 8.5 Median = Nilai ke-3 2, 3, 5, 8, 8, 10, 12, 16
Median = 7 + 8 = 7.5
=6 Median = 8 + 8 = 8 2
2

87 88

22
D. Bagi setiap set data terkumpul, tentukan kelas mod dan nilai modnya. E. Cari min bagi setiap set data terkumpul yang diberikan.
For each set of grouped data, determine the modal class and the mode. 1.4 ; 1.5 Find the mean of each given set of grouped data. 1.6

CONTOH FAKTA UTAMA CONTOH 1. Upah (RM) 51 – 56 – 61 – 66 – 71 –


Jisim (kg) • Bagi set data terkumpul, kelas mod ialah kelas yang
Jarak (km) 10 – 15 – 20 – 25 – 30 – Wage (RM) 55 60 65 70 75
Mass (kg) 30 – 39 40 – 49 50 – 59 60 – 69 70 – 79 mempunyai kekerapan tertinggi.
For a set of grouped data, the modal class is the class Distance (km) 14 19 24 29 34 Kekerapan
with the highest frequency. 8 8 12 7 5
Kekerapan Kekerapan Frequency
Frequency 8 11 14 7 5 • Mod bagi set data terkumpul boleh dianggar dengan 8 17 24 20 11
Frequency
melukis sebuah histogram. Upah Titik tengah Kekerapan
The mode of a set of grouped data can be estimated by Wage Frequency fx
Kelas mod = 50 – 59 kg drawing a histogram. Jarak Titik tengah Kekerapan Midpoint
Distance Midpoint Frequency fx (RM) x f
Kekerapan
Kelas mod (km) x f 51 – 55 53 8 424
Modal class
10 – 14 12 8 96 56 – 60 58 8 464

Kekerapan
15

Frequency
15 – 19 17 17 289 61 – 65 63 12 756
10
20 – 24 22 24 528 66 – 70 68 7 476
Pemboleh ubah
5 Variable 25 – 29 27 20 540 71 – 75 73 5 365
Nilai mod dibaca di sini.
The value of mode is read here. 30 – 34 32 11 352 Σf = 40 Σfx = 2 485
0 Jisim (kg)
39.5 49.5 59.5 69.5
52.5 Σf = 80 Σfx = 1 805

Mod = 52.5 kg 2 485


Min, x = = RM62.13
40
1. 2. Σfx 1 805
Masa (minit) 10 – 15 – 20 – 25 – 30 – Panjang (cm) 41 – 51 – 61 – 71 – 81 – Min, x = = = 22.56 km
Time (minute)
Σf 80
14 19 24 29 34 Length (cm) 50 60 70 80 90
Kekerapan Kekerapan
4 5 9 3 2 6 12 24 20 17
Frequency Frequency 2. Markah 10 – 20 – 30 – 40 – 50 – 3. Masa (saat) 1– 6– 11 – 16 – 21 –
Marks 19 29 39 49 59 Time (second) 5 10 15 20 25
Kelas mod = 20 – 24 minit Kelas mod = 61 – 70 cm
Bilangan murid Kekerapan
8 19 50 45 28 1 5 15 17 12
Number of students Frequency
Kekerapan Kekerapan
Titik tengah Kekerapan Masa (saat) Titik Kekerapan
Markah tengah
Marks
Midpoint Frequency fx Time Frequency fx
10 25 x f Midpoint
(second)
x f
10 – 19 14.5 8 116
8 20 1–5 3 1 3
20 – 29 24.5 19 465.5
6
6 – 10 8 5 40
15
30 – 39 34.5 50 1 725
11 – 15 13 15 195
4 10 40 – 49 44.5 45 2 002.5
16 – 20 18 17 306
50 – 59 54.5 28 1 526
2 5 21 – 25 23 12 276
Σf = 150 Σfx = 5 835
Masa Panjang Σf = 50 Σfx = 820
0 (minit) 0 (cm)
14.5 19.5 24.5 29.5 50.5 60.5 70.5 80.5
21.5 68
5 835 = 38.9 markah
Min, x =
150
Mod = 21.5 minit Mod = 68 cm 820 = 16.4 saat
Min, x =
50

89 90

F. Bagi setiap set data, cari median dengan menggunakan (a) rumus untuk median, (b) ogif. 2. Tinggi (cm)
For each set of data, find the median by using (a) the formula for median, (b) an ogive. 1.7 ; 1.8 110 – 119 120 – 129 130 – 139 140 – 149 150 – 159 160 – 169
Height (cm)
1. Jisim (kg)
31 – 35 36 – 40 41 – 45 46 – 50 51 – 55 56 – 60 Kekerapan
Mass (kg) Frequency
7 11 16 26 13 7

Bilangan murid
Number of students
3 8 10 18 9 2 Tinggi (cm) Kekerapan Kekerapan longgokan Sempadan atas
Height (cm) Frequency Cumulative frequency Upper boundary

Jisim (kg) Kekerapan Kekerapan longgokan Sempadan atas 110 – 119 7 7 119.5
Mass (kg) Frequency Cumulative frequency Upper boundary
120 – 129 11 18 129.5
31 – 35 3 3 35.5
130 – 139 16 34 } F 139.5
36 – 40 8 11 40.5
140 – 149 26 } fm 60 149.5
41 – 45 10 21 } F 45.5
150 – 159 13 73 159.5
46 – 50 18 } fm 39 50.5
160 – 169 7 80 169.5
51 – 55 9 48 55.5
56 – 60 2 50 60.5 (a) N = 80 (b) Kekerapan longgokan

N = 40

 
N –F 80
2 Kekerapan longgokan 2
(a) Median, m = L + C (b)
fm Kelas median = 140 – 149 cm
70
Jumlah kekerapan, N = 50 50 L = 139.5
F = 34
dan N = 25 45 60
2 fm = 26
C = 10
Kelas median = 46 – 50 kg 40
50
40 – 34 (10)
Median = 139.5 + 
L = 45.5 35 26 
= 141.8 cm 40
30
F = 21

25 30
fm = 18
20
C=5 20

15
25 – 21 (5)
Median = 45.5 + 
18  10
= 46.6 kg 10

0
5
109.5 119.5 129.5 139.5 149.5 159.5 169.5
Tinggi (cm)
0 Jisim (kg)
30.5 35.5 40.5 45.5 50.5 55.5 60.5
Berdasarkan ogif, median = 141.5 cm
Berdasarkan ogif, median = 46.5 kg

91 92

23
G. Selesaikan masalah yang berikut. 7.2 Sukatan Serakan SPM K1 ‘13, ’15, ‘16 K2 ’13

Solve the following problems. 1.9


A. Cari julat dan julat antara kuartil bagi setiap set data yang diberikan.
2.1 ; 2.2
CONTOH Find the range and interquartile range of each given set of data.

Mod, median dan min bagi satu set nombor masing-masing ialah 9, 10.8 dan 11.5. Cari mod, median dan CONTOH 1. 50, 40, 46, 34, 41, 45, 39 2. 20, 15, 11, 13, 14, 18, 9, 22
min yang baharu jika setiap nombor dalam set itu 42, 40, 45, 32, 28
The mode, median and mean of a set of numbers are 9, 10.8 and 11.5 respectively. Find the new mode, median and mean Julat = 50 – 34 Julat = 22 – 9
if each number in the set is Julat = 16 = 13
= Nilai terbesar – Nilai terkecil
(a) ditolak dengan 3 dan kemudian didarab dengan 2. (b) didarab dengan 4 dan kemudian ditolak dengan 7. = 45 – 28 Susun semula semua nilai: Susun semula semua nilai:
decreased by 3 and then multiplied by 2. multiplied by 4 and then decreased by 7.
= 17
34 , 39 , 40 , 41 , 45 , 46 , 50 9 , 11 , 13 , 14 , 15 , 18 , 20 , 22
(a) Mod baharu = (9 – 3) × 2 (b) Mod baharu = 9 × 4 – 7 Susun semua nilai mengikut tertib ↓ ↓ ↓ ↓ ↓ ↓
= 12 = 29 menaik: Q1 Median Q3 Q1 Median Q3
Median baharu = (10.8 – 3) × 2 Median baharu = 10.8 × 4 – 7
28 , 32 , 40 , 42 , 45
= 15.6 = 36.2
Julat antara kuartil Q1 = 11 + 13 = 12
↓ ↓ ↓ = Q3 – Q1 2
Min baharu = (11.5 – 3) × 2 Min baharu = 11.5 × 4 – 7 Q1 Median Q3 = 46 – 39
= 17 = 39 =7 Q3 = 18 + 20 = 19
2
Q1 = 28 + 32 = 30
1. Mod, median dan min bagi satu set nombor masing-masing ialah 20, 22.5 dan 21.2. 2 Julat antara kuartil
The mode, median and mean of a set of numbers are 20, 22.5 and 21.2 respectively. = Q 3 – Q1
Q3 = 42 + 45 = 43.5 = 19 – 12
(a) Cari mod, median dan min yang baharu jika setiap nombor dalam set itu ditambah dengan 2.1 dan 2
=7
kemudian dibahagi dengan 2.
Find the new mode, median and mean if each number in the set is increased by 2.1 and then divided by 2. Julat antara kuartil
= Q3 – Q1
(b) Jika satu nombor u ditolak daripada setiap nombor dalam set itu dan kemudian bezanya didarab
= 43.5 – 30
dengan 3, mediannya menjadi 60. Cari nilai u.
If a number u is subtracted from each number in the set and the difference is multiplied by 3, the median becomes 60.
= 13.5
Find the value of u.

(a) Mod baharu = (20 + 2.1) ÷ 2 (b) (22.5 – u) × 3 = 60 B. Cari julat antara kuartil bagi setiap set data yang diberikan.
= 11.05 22.5 – u = 20 Find the interquartile range of each given set of data. 2.2
u = 2.5
Median baharu = (22.5 + 2.1) ÷ 2 CONTOH
= 12.3
Min baharu = (21.2 + 2.1) ÷ 2 Jisim (kg)
51 52 53 54 55 56
Mass (kg)
= 11.65
Kekerapan 4 
Q1 = Nilai ke- 1 × 58 
5 7 10 17 13 6 = Nilai ke-14.5
Frequency
2. Diberi satu set yang terdiri daripada tujuh nombor, x1, x2, x3, x4, x5, x6, x7, mempunyai min 17. = Nilai ke-15
Jika x4 dikeluarkan daripada set itu, nilai minnya berkurang sebanyak 2. Tentukan nilai x4. = 53 kg
Jisim (kg) Kekerapan Kekerapan longgokan
 
Given a set of seven numbers, x1, x2, x3, x4, x5, x6, x7, has a mean of 17. If x4 is removed from the set, the mean decreases
by 2. Determine the value of x4.
Mass (kg) Frequency Cumulative frequency Q3 = Nilai ke- 3 × 58
4
51 5 5 = Nilai ke-43.5
Σx = 17
7 = Nilai ke-44
52 7 12 = 55 kg
Σx = 119
Apabila x 4 dikeluarkan, min = 15. Q1 → 53 10 22
Julat antara kuartil
Σx – x 4 = Q3 – Q1
= 15 54 17 39
6 = 55 – 53
119 – x 4 = 90
Q3 → 55 13 52 = 2 kg
x 4 = 119 – 90
= 29 56 6 58

93 94

1. Skor C. Bagi set data terkumpul yang berikut, cari (a) julat dan (b) julat antara kuartil secara (i) penghitungan,
0 1 2 3 4 5 (ii) melukis ogif.
Score
For the following set of grouped data, find the (a) range and (b) interquartile range by (i) calculation, (ii) plotting an ogive.
Bilangan peserta 15 17 23 29 24 12 2.4 ; 2.5
Number of participants
Panjang (mm) 10 – 13 14 – 17 18 – 21 22 – 25 26 – 29 30 – 33
Skor Kekerapan Kekerapan longgokan Length (mm)
Q1 = Nilai ke- 1 × 120
Score Frequency Cumulative frequency 4 Kekerapan
= Nilai ke-30 8 15 49 73 32 3
Frequency
0 15 15
=1
1 17 32 Panjang (mm) Kekerapan Kekerapan longgokan Sempadan atas
Q3 = Nilai ke- 3 × 120
4 Length (mm) Frequency Cumulative frequency Upper boundary
Q1 → 2 23 55
= Nilai ke-90
10 – 13 8 8 13.5
3 29 84 =4
14 – 17 15 23 17.5
Q3 → 4 24 108 Julat antara kuartil
=4–1 Q1 → 18 – 21 49 72 21.5
5 12 120
=3
Q3 → 22 – 25 73 145 25.5
26 – 29 32 177 29.5
30 – 33 3 180 33.5

(a) Julat = Titik tengah kelas tertinggi – Titik tengah kelas terendah

2.

= 30 + 33 –
2   10 + 13
2 
Umur (tahun)
40 41 42 43 44 45 = 31.5 – 11.5
Age (years)
Bilangan ahli = 20 mm
12 13 20 30 18 17
Number of members

Umur (tahun) Kekerapan Kekerapan longgokan (b) (i) N = 180


Q1 = Nilai ke- 1 × 110 3N = 3 × 180 = 135
Age (years) Frequency Cumulative frequency 4 N = 180 = 45
= Nilai ke-27.5 4 4
40 12 12 4 4
= Nilai ke-28
41 13 25 = 42 tahun Q1 berada dalam kelas 18 – 21 mm . Q3 berada dalam kelas 22 – 25 mm .

Q1 → 42 Q3 = Nilai ke- 3 × 110 3N – F


   
20 45 N –F
4 1 3
Q1 = L1 + 4 C Q3 = L3 + 4 C
43 30 75 = Nilai ke-82.5 f1 f3
= Nilai ke-83
Q3 → 44 18 93 = 44 tahun = 17.5 +  45 – 23 (4) = 21.5 +  135 – 72 (4)
45 17 110 Julat antara kuartil 49 73
= 44 – 42 = 17.5 + 1.80 = 21.5 + 3.45
= 2 tahun
= 19.3 mm = 25.0 mm

Julat antara kuartil = Q3 – Q1


= 25.0 – 19.3
= 5.7 mm

95 96

24
(b) (ii) Kekerapan longgokan D. Cari min, varians dan sisihan piawai bagi setiap set data yang diberikan.
Find the mean, variance and standard deviation of each given set of data. 2.6 (a) ; 2.7 (a)

CONTOH 1. 19, 14, 16, 18, 10, 12


5, 6, 7, 2, 3
180
Min, x = Σx = 5 + 6 + 7 + 2 + 3 = 4.6 Min, x = 19 + 14 + 16 + 18 + 10 + 12 = 14.83
6
N 5
Varians, σ 2
Varians, σ 2 = Σx – x 2
2

N 192 + 142 + 162 + 182 + 102 + 122


160
= – 14.832
2 2 2 2 2 6
= 5 + 6 + 7 + 2 + 3 – 4.62 1 381
5 = – 14.832
= 3.44 6
= 10.24
140 Sisihan piawai, σ = 3.44 = 1.855 Sisihan piawai, σ = 10.24 = 3.2

2. 101, 112, 124, 131, 98 3. 1.3, 2.5, 4.6, 3.0, 3.4, 4.1, 2.8

Min, x– = 1.3 + 2.5 + 4.6 + 3.0 + 3.4 + 4.1 + 2.8


120
Min, x– = 101 + 112 + 124 + 131 + 98 7
5
= 113.2 = 3.1
Varians, σ 2 Varians, σ 2
100
1012 + 1122 + 1242 + 1312 + 982 1.32 + 2.52 + 4.62 + 3.02 + 3.42 + 4.12 + 2.82
= – 113.22 = – 3.12
5 7
64 886 74.31
= – 113.22 = – 3.12
5 7
80 = 162.96 = 1.006

Sisihan piawai, σ = 162.96 = 12.77 Sisihan piawai, σ = 1.006 = 1.003

60
E. Cari min, varians dan sisihan piawai bagi setiap set data yang diberikan.
Find the mean, variance and standard deviation of each given set of data. 2.6 (a) ; 2.7 (a)

CONTOH
40
Wang saku (RM) 2 3 4 5 6 7
Pocket money (RM)
Bilangan murid
12 16 15 14 12 11
Number of students
20

x f fx x2 fx 2 Σfx 351
Min, x = = = 4.388
Q1 Q3 2 12 24 4 48 Σf 80
0 Panjang (mm)
9.5 13.5 17.5 21.5 25.5 29.5 33.5 3 16 48 9 144 Σfx 2
Varians, σ 2 = – x2
Berdasarkan ogif, Q1 = 19.5 mm 4 15 60 16 240 Σf
5 14 70 25 350 = 1 753 – 4.3882
Q3 = 24.9 mm 80
6 12 72 36 432 = 2.658
Julat antara kuartil = Q3 – Q1 7 11 77 49 539
= 24.9 – 19.5 Sisihan piawai = 2.658
Σf = 80 Σfx = 351 Σfx 2 = 1 753
= RM1.63
= 5.4 mm

97 98

1. Markah 2. Jisim (g) 1. Bilangan durian


55 60 65 70 75 16 17 18 19 20 10 – 14 15 – 19 20 – 24 25 – 29 30 – 34
Marks Mass (g) Number of durians
Bilangan murid Bilangan cili Kekerapan
Number of students
5 10 20 5 10 Number of chillies
16 20 22 27 15 18 30 32 26 14
Frequency

x f fx x2 fx 2 x f fx x2 fx 2
55 5 275 3 025 15 125 16 16 256 256 4 096 Bilangan durian Titik tengah, x f fx fx 2
Number of durians Midpoint, x
60 10 600 3 600 36 000 17 20 340 289 5 780
10 – 14 12 18 216 2 592
65 20 1 300 4 225 84 500 18 22 396 324 7 128
15 – 19 17 30 510 8 670
70 5 350 4 900 24 500 19 27 513 361 9 747
75 10 750 5 625 56 250 20 15 300 400 6 000 20 – 24 22 32 704 15 488

Σf = 50 Σfx = 3 275 Σfx 2 = 216 375 Σf = 100 Σfx = 1 805 Σfx 2 = 32 751 25 – 29 27 26 702 18 954

Σfx 3 275 30 – 34 32 14 448 14 336


Min, x = = = 65.5 Min, x = 1 805 = 18.05
Σf 50 100 Σf = 120 Σfx = 2 580 Σfx 2 = 60 040
Σfx 2
Varians, σ 2 = – x 2 = 216 375 – 65.52 Varians, σ 2 = 32 751 – 18.052 = 1.708
Σf 50 100 Min, x = 2 580 Varians, σ 2 = 60 040 – 21.52 Sisihan piawai, σ = 38.08
= 37.25 120 120
= 6.171
Sisihan piawai, σ = 1.708 = 1.307 = 21.5 = 38.08
Sisihan piawai, σ = 37.25 = 6.103

F. Cari min, varians dan sisihan piawai bagi setiap set data yang diberikan. 2.
2.6 (b) ; 2.7 (b)
Harga (RM)
Find the mean, variance and standard deviation of each given set of data. 20 – 23 24 – 27 28 – 31 32 – 35 36 – 39 40 – 43
Price (RM)
CONTOH Kekerapan 7 8 10 16 17 6
Frequency
Umur (tahun)
25 – 28 29 – 32 33 – 36 37 – 40 41 – 44 45 – 48
Age (years)
Bilangan pekerja Harga (RM) Titik tengah, x
Number of staff
3 5 9 10 7 6 f fx fx 2
Price (RM) Midpoint, x

Umur (tahun) Titik tengah, x 20 – 23 21.5 7 150.5 3 235.75


Age (years) Midpoint, x
f fx fx 2
24 – 27 25.5 8 204.0 5 202.00
25 – 28 26.5 3 79.5 2 106.75
28 – 31 29.5 10 295.0 8 702.50
29 – 32 30.5 5 152.5 4 651.25
32 – 35 33.5 16 536.0 17 956.00
33 – 36 34.5 9 310.5 10 712.25
37 – 40 38.5 10 385.0 14 822.50 36 – 39 37.5 17 637.5 23 906.25
41 – 44 42.5 7 297.5 12 643.75 40 – 43 41.5 6 249.0 10 333.50
45 – 48 46.5 6 279.0 12 973.50 Σf = 64 Σfx = 2 072 Σfx 2 = 69 336
Σf = 40 Σfx = 1 504 Σfx 2 = 57 910

Σfx Σfx 2 Min, x = 2 072 Varians, σ 2 = 69 336 – 32.3752 Sisihan piawai, σ = 35.23
Min, x = Varians, σ 2 = – x2 Sisihan piawai, σ 64 64
Σf Σf = RM32.38 = 35.23 = RM5.94
= 33.99
= 1 504 = 57 910 – 37.62 = 5.830 tahun
40 40
= 37.6 = 33.99

99 100

25
G. Selesaikan masalah yang berikut.
Solve the following problems. 2.8
PRAKTIS FORMATIF Kertas 1 ANALISIS SOALAN SPM
Subtopik 2013 2014 2015 2016

CONTOH FAKTA UTAMA Jawab semua soalan.


7.1
7.2

S. 22
S. 21, 22

S. 20
S. 19

S. 1
Answer all the questions.
Julat, julat antara kuartil dan varians bagi satu set data Apabila setiap nilai dalam satu set data
When each value in a set of data
masing-masing ialah 21, 9 dan 12. Cari julat, julat antara 1. Satu set data terdiri daripada 7, 0, 5, x2 – 3 dan 2. 3. Rajah di bawah menunjukkan satu histogram bagi
• ditambah atau ditolak dengan suatu pemalar, KLON taburan skor yang diperoleh sekumpulan peserta
kuartil dan varians yang baharu jika setiap nilai dalam KLON
SPM
Diberi min ialah 4, cari SPM
sukatan serakan bagi set data baharu kekal tidak
set itu berubah.
’14 A set of data consists of 7, 0, 5, x2 – 3 and 2. Given the mean ’14 dalam satu kuiz.
7.1 7.1
The range, interquartile range and variance of a set of data are 21, is added by a constant or a constant is subtracted is 4, find The diagram shows a histogram for the distribution of scores
9 and 12 respectively. Find the new range, new interquartile range from it, the measures of dispersion of the new set of data (a) nilai positif bagi x. obtained by a group of participants in a quiz.
and new variance if each value in the set remain unchanged. the positive value of x.
Kekerapan
(a) didarab dengan 4 dan kemudian 5 ditolak daripadanya. • didarab dengan pemalar k: (b) median dengan menggunakan nilai x di (a). Frequency
is multiplied by a constant k: the median using the value of x in (a).
is multiplied by 4 and then 5 is subtracted from it. 10
(b) ditambah dengan 6 dan kemudian dibahagi dengan 3. (a) Julat baharu = k × Julat asal [4] 9
New range = k × Original range 2
is added by 6 and then divided by 3. (a) Min = 7 + 0 + 5 + (x – 3) + 2 = 4 8
(b) Julat antara kuartil baharu 5 7
= k × Julat antara kuartil asal 11 + x2 = 4 6
(a) Julat baharu = 21 × 4 = 84 New interquartile range 5 5
= k × Original interquartile range 11 + x2 = 20 4
Julat antara kuartil baharu = 9 × 4 = 36 3
(c) Sisihan piawai baharu x2 = 9 2
= k × Sisihan piawai asal x=3
Varians baharu = 12 × 42 = 192 New standard deviation
1
Skor
0
= k × Original standard deviation (b) Susun semula data mengikut tertib menaik: 4.5 6.5 8.5 10.5 12.5 14.5 Score
(b) Julat baharu = 21 ÷ 3 = 7 (d) Varians baharu = k2 × Varians asal 0,2,5,6,7
New variance = k2 × Original variance (a) Cari jumlah bilangan peserta.
Julat antara kuartil baharu = 9 ÷ 3 = 3 ↑ Find the total number of participants.
Maka, median = 5 (b) Cari skor min.
Varians baharu = 12 ÷ 32 = 1.333 Find the mean score.
[3]
2. Jadual di bawah menunjukkan skor bagi suatu kuiz
KLON yang diperoleh sekumpulan murid. (a) Jumlah bilangan peserta = 3 + 6 + 8 + 10 + 4
1. Satu set data mempunyai julat 10, julat antara 2. Satu set data terdiri daripada 4, 6, 8, 9, 12 dan 15. SPM
= 31 orang
’15 The table shows the scores of a quiz obtained by a group
kuartil 6 dan sisihan piawai 1.8. Setiap nilai dalam A set of data consists of 4, 6, 8, 9, 12 and 15. 7.1
of students.
set itu dibahagi dengan 4 dan kemudian 2 ditolak (a) Cari min, julat dan varians bagi set data ini.
Find the mean, range and variance of this set of data.
KBAT Skor (b) Titik tengah f fx
daripadanya. Cari Score
1 2 3 4 5
A set of data has a range of 10, an interquartile range of 6 (b) Setiap nilai dalam set itu didarab dengan 2 dan 5.5 3 16.5
kemudian ditambah dengan 5. Cari min, julat Kekerapan
and a standard deviation of 1.8. Each value in the set is
Frequency
4 k 12 9 8
divided by 4 and then 2 is subtracted from it. Find dan varians bagi set data baharu. 7.5 6 45
(a) julat baharu. Each value in the set is multiplied by 2 and then added (a) Cari nilai maksimum bagi k, jika mod ialah 3. 9.5 8 76
the new range. by 5. Find the mean, range and variance of the new Find the maximum value of k, if the mode is 3.
(b) julat antara kuartil baharu. set of data. (b) Cari julat nilai k, jika median ialah 3. 11.5 10 115
the new interquartile range. Find the range of values of k, if the median is 3.
(c) sisihan piawai baharu. (a) Min, x– = 4 + 6 + 8 + 9 + 12 + 15 = 9 [4] 13.5 4 54
6 (a) Jika mod ialah 3, maka nilai k  12.
the new standard deviation. Σf = 31 Σfx = 306.5
Julat = 15 – 4 = 11 Jadi, nilai maksimum k = 11.
(a) Julat baharu = 10 ÷ 4 Σfx
Varians, σ 2 (b) Kes l:
= 2.5 Skor min =
Jika median ialah skor 3 yang pertama, maka Σf
42 + 62 + 82 + 92 + 122 + 152 4 + k = (12 – 1) + 9 + 8
(b) Julat antara kuartil baharu = 6 ÷ 4 = – 92 = 306.5
6 4 + k = 28 31
= 1.5 = 13.33 k = 24 = 9.89
(c) Sisihan piawai baharu = 1.8 ÷ 4 (b) Min baharu = 9 × 2 + 5 Kes 2:
= 0.45 = 23 Jika median ialah skor 3 yang terakhir, maka
Julat baharu = 11 × 2 4 + k + (12 – 1) = 9 + 8
= 22 k + 15 = 17
Varians baharu = 22 × 13.33 k=2
= 53.33 Julat nilai k ialah 2  k  24, dengan keadaan
k ialah integer.

101 102

4. Satu set enam nombor mempunyai min 10. 6. Sekumpulan 8 orang murid mempunyai jumlah umur 8. Suatu set mempunyai enam nombor dengan sisihan 9. Satu set data mengandungi lapan nombor positif.
Diberi bahawa Σ(x – x–)2 = 96 dan Σx2 = 168. Cari
7.1
A set of six numbers has a mean of 10. 120 tahun. Hasil tambah kuasa dua umur-umur KLON piawai 2 dan suatu set lain mempunyai empat nombor KLON
SPM SPM
(a) Cari Σx. mereka ialah 1 816 tahun. Cari ’15 dengan sisihan piawai 3. Kedua-dua set nombor itu ’13 A set of data consists of eight positive numbers. It is given that
Find Σx. A group of 8 students has a total age of 120 years. The sum 7.2 7.2
Σ(x – x–)2 = 96 and Σx 2 = 168. Find
mempunyai min yang sama. Jika dua set nombor itu
(b) Apabila satu nombor m ditambah kepada set ini, of the squares of their ages is 1 816 years. Find KBAT digabungkan, cari variansnya. KBAT (a) varians.
min baharu ialah 12. Cari nilai m. (a) min umur 8 orang murid itu. A set of six numbers has a standard deviation of 2 and another the variance.
When a number m is added to this set, the new mean 7.1 the mean of the ages of the 8 students. set of four numbers has a standard deviation of 3. Both sets (b) min.
is 12. Find the value of m. (b) sisihan piawai. of numbers have an equal mean. If these two sets of numbers the mean.
[3] 7.2 the standard deviation. [3]
Σx are combined, find the variance.
(a) = 10 [3] [4] – 2
6
N = 8, Σx = 120, Σx2 = 1 816 (a) Varians = Σ(x – x )
Min = Σx , σ 2 = Σx – x– 2
2
Σx = 60 N
N N 96
(a) Min umur, x– = 120 =
(b) Σx + m = 12 8 Katakan min bagi dua set nombor itu ialah m. 8
7 = 12
= 15 tahun Bagi set 6 nombor:
60 + m = 12
(b) σ 2 = Σx – x– 2
2
7 (b) Sisihan piawai, σ m = Σx1 N
60 + m = 84 Σx 2 6
m = 24
= – x– 2 Σx1 = 6m ...... ➀ 12 = 168 – x– 2
N 8
Σx12 – 2 12 = 21 – x– 2
= 1 816 σ2 = –x
– 152 N
8 x– 2 = 9
Σx12
x– = 3
2
= 2 2 = – m2
6
= 1.414 tahun 24 = Σx12 – 6m2
5. Satu set data 12 nombor, x1 , x 2 , ..., x12 , mempunyai Σx12 = 24 + 6m2 ...... ➁
7.2
varians 30 dan diberi bahawa Σx 2 = 840. Cari
A set of 12 numbers, x1 , x2 , ... , x12 , has a variance of 30 and Bagi set 4 nombor:
it is given that Σx2 = 840. Find
7. Jisim satu kumpulan 8 orang murid mempunyai min Σx2
(a) min, x–. m= 10. Jadual di bawah menunjukkan keputusan ujian
the mean, x– .
50 kg dan sisihan piawai 4 kg. Cari 4
The masses of a group of 8 students has a mean of 50 kg and Σx2 = 4m ...... ➂
KLON
SPM
makmal tentang jangka hayat tiga jenis bateri telefon
(b) nilai bagi Σx. a standard deviation of 4 kg. Find
’16 bimbit, Semsang, Elgi dan Epal.
the value of Σx. 7.2

32 = Σx2 – m2
2 The table shows the laboratory test results on the life span of
[3] (a) hasil tambah jisim murid-murid itu.
4 three types of mobile phone batteries, Semsang, Elgi and Epal.
7.1 the sum of the masses of the students.
N = 12, σ2 = 30, Σx 2 = 840
(b) hasil tambah kuasa dua jisim murid-murid itu. 36 = Σx22 – 4m2 Min jangka Sisihan piawai
(a) σ2 = Σx – x– 2
2 7.2 the sum of the squares of the masses of the students.
Bateri hayat (Tahun) jangka hayat (bulan)
N [3] Σx22 = 36 + 4m2 ...... ➃ Battery Mean life span Standard deviation of
30 = 840 – x– 2 (a) Hasil tambah jisim murid-murid Dua set nombor itu digabungkan. (year) life span (month)
12
30 = 70 – x– 2 = 8 × 50 kg ➀ + ➂: Σx1 + Σx2 = 10m Semsang 1.8 3
x– 2 = 40 = 400 kg Elgi 1.5 1.2
➁ + ➃: Σx12 + Σx22 = 60 + 10m2
x– = 6.325
σ 2 = Σx – x– 2
2
(b) Epal 1.8 4
Σx N Varians baharu, σ2
(b) = 6.325
42 = Σx – 502
12
2 Bateri manakah menunjukkan jangka hayat yang
Σx12 + Σx22 Σx + Σx2 2
Σx = 12 × 6.325 8 = – 1  paling konsisten dalam ujian itu? Beri sebab untuk
10 10
= 75.90 Σx2 = 16 + 2 500 jawapan anda.
8 60 + 10m2 10m 2 Which battery shows the most consistent life span in the test?
–
10 
= Give a reason for your answer.
∑x2 = 8 × 2 516 10
= 20 128 [2]
= 6 + m2 – m2
Hasil tambah kuasa dua jisim murid-murid itu Jangka hayat bateri Elgi adalah paling konsisten.
=6
ialah 20 128 kg2. Sebab sisihan piawai jangka hayatnya paling rendah.

103 104

26
3. Jadual di bawah menunjukkan hasil tambah dan hasil (b) Min bagi bilangan buku yang dibaca oleh murid
PRAKTIS FORMATIF Kertas 2 ANALISIS SOALAN SPM 7.2
tambah kuasa dua bagi x, dengan keadaan x ialah perempuan dalam Kelas 4 Cerdik ialah 7 dan hasil
Subtopik 2013 2014 2015 2016
7.1 – – – S. 1 pendapatan bulanan, dalam RM, bagi Encik Mazlan tambah kuasa dua bilangan buku yang dibaca
Jawab semua soalan. 7.2 S. 6 – – – untuk 8 bulan pertama tahun 2017. ialah 720. Diberi bahawa jumlah bilangan buku
Answer all the questions. The table shows the sum and the sum of the squares of x, yang dibaca oleh murid perempuan itu ialah 105.
where x is the monthly income, in RM, of Encik Mazlan for the Cari varians bagi bilangan buku yang dibaca oleh
1. Jadual di bawah menunjukkan taburan kekerapan 2. Min bagi suatu set nombor 1, y, 7, 2y + 2, 13 dan 16 first 8 months in the year 2017.
KLON bagi gaji mingguan sekumpulan pekerja.
7.2
ialah 9. semua murid dalam Kelas 4 Cerdik.
SPM
The mean of a set of numbers 1, y, 7, 2y + 2, 13 and 16 is 9. The mean of the number of books read by the girls in
’16 The table shows the frequency distribution of the weekly wages ∑x 16 800 Class 4 Cerdik is 7 and the sum of the squares of the
7.1
of a group of workers. (a) Cari
number of books read is 720. It is given that the total
Find ∑ x2 35 385 000
number of books read by the girls is 105. Find the
Gaji (RM) Bilangan pekerja (i) nilai y. variance of the number of books read by all the students
Wages (RM) Number of workers the value of y. (a) Cari sisihan piawai bagi pendapatan bulanannya.
Find the standard deviation of his monthly income. in Class 4 Cerdik.
(ii) varians. [4]
300 – 399 12 the variance. [3]
(a) (i) Min = 6
400 – 499 13 [4] (b) Jika anak lelaki Encik Mazlan memberi RM300 Σx = 6
setiap bulan dalam tempoh masa itu, cari min 15
500 – 599 20 (b) Setiap nombor dalam set itu didarab dengan 2 dan baharu dan sisihan piawai baharu bagi pendapatan Σx = 6 × 15 = 90
600 – 699 m kemudian ditambah dengan 3. Bagi set nombor bulanannya itu.
ini, cari If Encik Mazlan's son gives him RM300 every month (ii) Varians = 32
700 – 799 10 Each number in the set is multiplied by 2 and then 3 is
added to it. For this set of numbers, find
during that period, find the new mean and new standard Σx2 – 6 2 = 32
deviation of his monthly income. 15
Diberi bahawa min gaji mingguan bagi pekerja itu (i) min. [3] Σx2 = (9 + 36) × 15
ialah RM546.60. the mean.
It is given that the mean of the weekly wages of the workers (ii) sisihan piawai. (a) x– = Σx = 16 800 = 2 100 = 675
is RM546.60. N 8
the standard deviation. (b) Jumlah bilangan buku yang dibaca oleh murid
σ 2 = Σx – x– 2
(a) Cari nilai m. 2
[4] perempuan = 105
Find the value of m. N
(a) (i) Min = 9 Maka, Σx = 105
[3]
1 + y + 7 + 2y + 2 + 13 + 16 = 9 = 35 385 000 – 2 1002
Diberi min = Σx = 7.
(b) Seterusnya, tanpa menggunakan ogif, hitung 8
median bagi gaji mingguan pekerja itu. 6
3y + 39 = 9 = 13 125 N
Hence, without using an ogive, calculate the median of
the weekly wages of the workers.
6 σ = 114.56 Maka, 105 = 7
3y + 39 = 54 N
[3] Maka, sisihan piawai bagi pendapatan bulanannya
3y = 15 ialah RM114.56. N = 105 = 15
(a) Min gaji mingguan = RM546.60 y=5 7
(ii) Varians (b) Min baharu = RM2 100 + RM300 Jumlah bilangan murid perempuan ialah 15 orang.
12(349.50) + 13(449.50) + 20(549.50) 2 2 2 2 2 2 = RM2 400
+ m(649.50) + 10(749.50) = 1 + 5 + 7 + 12 + 13 + 16 – 92 Apabila murid perempuan dan murid lelaki
= 546.60 6 Tiada perubahan pada sisihan piawai. diambil kira,
12 + 13 + 20 + m + 10
= 644 – 81 N = 15 + 15 = 30
28 522.5 + 649.5m = 546.6 Maka, sisihan piawai baharu = RM114.56
6
55 + m 1 Σx = 105 + 90 = 195
28 522.5 + 649.5m = 30 063 + 546.6m = 26
3 Σx2 = 720 + 675 = 1 395
649.5m – 546.6m = 30 063 – 28 522.5
102.9m = 1 540.5 (b) (i) Min baharu = 9(2) + 3 Min bilangan buku yang dibaca oleh semua murid
m = 15 = 21 = 195
4. (a) Bilangan buku yang dibaca oleh setiap murid 30
(b) Jumlah bilangan pekerja = 55 + 15 1
(ii) Sisihan piawai asal = 26 KLON lelaki dalam Kelas 4 Cerdik diberi oleh x1, x2, Varians bagi bilangan buku yang dibaca oleh
= 70 orang 3 SPM

semua murid = Σx – x– 2
’13 x3, ..., x15. Min bagi bilangan buku yang dibaca 2
Kelas median = 500 – 599 = 5.1316 7.2
ialah 6 dan sisihan piawainya ialah 3. Cari N
Median gaji mingguan Sisihan piawai baharu = 5.1316 × 2 KBAT The number of books read by each boy in Class 4 Cerdik 2
= 10.26 is given by x1, x2, x3, ..., x15. The mean of the number = 1 395 – 195
30 30  
 
1 of books read is 6 and the standard deviation is 3. Find
(70) – 25 = 4.25
= 499.5 + 2 (100) (i) jumlah bilangan buku yang dibaca, ∑x.
20 the total number of books read, ∑x.
= RM524.50 (ii) hasil tambah kuasa dua bagi bilangan buku
yang dibaca, ∑x2.
the sum of the squares of the numbers of books
read, ∑x2.
[3]

105 106

FOKUS KBAT BAB

8
SUKATAN MEMBULAT
CIRCULAR MEASURES
Kemahiran Kognitif: Menganalisis
Konteks: Nilai Min dan Perubahan Min
Dalam sebuah syarikat, gaji min bagi semua pekerja ialah RM2 700. Apabila seorang pekerja baharu dengan gaji sebanyak 8.1 Radian SPM K1 ‘13, ‘15 K2 ‘13, ’14, ’16
RM3 350 menyertai syarikat itu, gaji min bagi semua pekerja meningkat sebanyak RM50. Berapakah jumlah bilangan
pekerja dalam syarikat itu sekarang? A. Tukarkan setiap yang berikut kepada darjah dan minit.
Convert each of the following to degrees and minutes. 1.1
In a company, the mean salary of all the employees was RM2 700. When a new employee with a salary of RM3 350 joined the company,
the mean salary of all the employees was increased by RM50. What is the total number of employees in the company now?
CONTOH FAKTA UTAMA
Min asal = 2 700
Info KBAT (a) π rad = π × 180° (b) 1.5 rad = 1.5 × 180° • Daripada radian kepada darjah:
Σx = 2 700 6 6 π π From radians to degrees:
N Pertimbangkan jumlah bilangan pekerja semasa 180°
= 30° = 85° 57 θ rad = θ × π
Σx = 2 700N …… ➀ menggunakan formula min.
Consider the total number of employees while using

1. π rad = π × 180°
the formula for the mean.
Setelah seorang pekerja baharu menyertai syarikat itu: 2. 0.45 rad = 0.45 × 180° 3. 2.12 rad = 2.12 × 180°
4 4 π π π
Min baharu = 2 700 + 50
= 2 750 = 45° = 25° 47 = 121° 28
Σx + 3 350 = 2 750 …… ➁
N+1
B. Tukarkan setiap yang berikut kepada radian dalam sebutan π.
Convert each of the following to radians in terms of π. 1.1
Gantikan ➀ ke dalam ➁.
2 700N + 3 350 = 2 750 CONTOH FAKTA UTAMA
N+1
2 700N + 3 350 = 2 750N + 2 750
(a) 60° = 60° × π (b) 135° = 135° ×
π • Daripada darjah kepada radian:
From degrees to radians:
180° 180°
π
3 350 – 2 750 = 2 750N – 2 700N = π rad = 3 π rad θ° = θ° ×
180°
rad
600 = 50N 3 4
N = 12
1. 90° = 90° × π 2. 150° = 150° ×
π
3. 270° = 270° ×
π
Jumlah bilangan pekerja sekarang = 12 + 1 180° 180° 180°
= 13 orang = π rad = 5 π rad = 3 π rad
2 6 2

C. Tukarkan setiap yang berikut kepada radian.


Convert each of the following to radians. 1.1

CONTOH 000
Sudut Kalkulator
π 134° 35' = 2.349 rad
134° 35 = 134° 35 ×
180° Set kalkulator dalam mod radian.
= 2.349 rad Set a calculator in the radian mode.
Tekan:
Press: 1 3 4 °
, ,, 3 5 °
, ,, SHIFT DRG 1 = 2.348922285

1. 55° = 55° × π 2. 88.3° = 88.3° × π


180° 180°
= 0.960 rad = 1.541 rad

3. 115° 21 = 115° 21 × π 4. 283° 25 = 283° 25 × π


180° 180°
= 2.013 rad = 4.947 rad

107 108

27
8.2 Panjang Lengkok Bulatan SPM K1 ’13, ’16 K2 ’13, ’14, ’15, ‘16 C. Cari perimeter bagi setiap tembereng berlorek yang berikut.
Find the perimeter of each of the following shaded segments. 2.2
A. Cari panjang lengkok, s, bagi setiap bulatan yang berikut.
2.1 (a)
Find the length of arc, s, in each of the following circles. CONTOH 1.
CONTOH 1. A O
s O 20 cm
s = jθ
s = jθ 10 cm
π O
1.5 O
rad
20 cm 0.75 20 cm
= 10 × 33° ×

30°
30°
45° π O 33° s
8 cm 8 cm
rad
5 cm
= 5 × 45° × 180° 8 cm 60°
O 180° = 5.760 cm B A M B
= 3.927 cm
A B A M B
180°
0.75 rad = 0.75 ×
π
2. 3. Dalam ΔOAM, sin 30° = AM
8 = 42.97°
s = jθ 60° 8 cm
s = jθ
4 cm AM = 8 sin 30°
= 8 × 300° × π
AM
2– π rad =4× 2π O Dalam ΔOAM, sin 42.97° =
O 3 s 3 180° AB = 2 × 8 sin 30° 20
= 8.378 cm = 41.89 cm AM = 20 sin 42.97°
= 8 cm
AB = 2 × 20 sin 42.97°
Panjang lengkok AB = 8 × 60° × π
s

180° = 27.26 cm
B. Selesaikan masalah yang berikut. = 8.378 cm
Solve the following problems. [Guna/Use π = 3.142] 2.1 Panjang lengkok AB = 20 × 1.5
Perimeter tembereng berlorek = 30 cm
1. Rajah di bawah menunjukkan sektor AOB. 2. Rajah di bawah menunjukkan semibulatan OABC = 8 + 8.378
The diagram shows a sector AOB. yang berpusat O. Perimeter tembereng berlorek
= 16.378 cm
O The diagram shows a semicircle OABC with centre O. = 27.26 + 30
8 cm
θ = 57.26 cm
A B B

θ 2. 3.
(a) Cari panjang lengkok AB jika nilai θ ialah A O 10 cm C O B O
A
Find the length of arc AB if the value of θ is B
3.9 cm
(i) 0.4 rad. (ii) 0.8 rad. (iii) 0.65 rad. (a) Cari panjang lengkok AB dan lengkok BC jika
(b) Cari nilai θ jika panjang lengkok AB ialah θ = 0.78 rad. 15 cm 35°

22.35°
15 cm 15 cm 15 cm 5 cm
70° 5 cm 5 cm
Find the value of θ if the length of arc AB is Find the lengths of arc AB and arc BC if θ = 0.78 rad. A
(i) 10 cm. (ii) 18 cm. (iii) 15 cm. (b) Jika panjang lengkok AB ialah 12 cm, cari O 5 cm
A M B
nilai θ, dalam radian. Seterusnya, cari ∠BOC, O
(a) (i) Panjang lengkok AB = 8 × 0.4 dalam darjah. A M B
= 3.2 cm If the length of arc AB is 12 cm, find the value of θ, Dalam ΔOAM, sin 35° = AM
in radians. Hence, find ∠BOC, in degrees.
15 ∠AOB = 3.9
(ii) Panjang lengkok AB = 8 × 0.8 AM = 15 sin 35° 5
= 6.4 cm (a) Panjang lengkok AB = 10 × 0.78 = 7.8 cm = 0.78 rad
AB = 2 × 15 sin 35° = 44.69°
(iii) Panjang lengkok AB = 8 × 0.65 ∠BOC = 3.142 – 0.78 = 2.362 rad = 17.21 cm
= 5.2 cm
Panjang lengkok BC = 10 × 2.362 π Dalam ΔOAM, sin 22.35° = AM
Panjang lengkok AB = 15 × 70° × 5
(b) (i) Panjang lengkok AB = jθ = 23.62 cm 180°
AM = 5 sin 22.35°
10 = 8θ = 18.33 cm
θ = 10 = 1.25 rad
(b) Panjang lengkok AB = 12 cm AB = 2 × 5 sin 22.35°
8 12 = 10θ Perimeter tembereng berlorek
= 3.803 cm
= 17.21 + 18.33
(ii) 18 = 8θ θ = 12 = 1.2 rad
10 = 35.54 cm Perimeter tembereng berlorek
θ = 18 = 2.25 rad ∠BOC = 3.142 – 1.2 = 3.803 + 3.9
8 = 7.703 cm
= 1.942 rad
(iii) 15 = 8θ
= 1.942 × 180°
θ = 15 = 1.875 rad 3.142
8 = 111.25°

109 110

D. Selesaikan masalah yang berikut. 8.3 Luas Sektor Bulatan SPM K1 ‘14, ‘15, ‘16 K2 ‘13, ‘14, ’15, ’16
Solve the following problems. 2.3
A. Cari luas setiap sektor berlorek yang berikut.
1. Rajah di bawah menunjukkan dua sektor, OAD dan OBC, berpusat O. Diberi AB = 2OA dan panjang OA Find the area of each of the following shaded sectors. 3.1 (a)
adalah sama dengan panjang lengkok BC.
The diagram shows two sectors, OAD and OBC, with centre O. Given AB = 2OA and the length of OA is equal to the length CONTOH 1.
of arc BC. Luas
1 2
Luas = j θ 1 π
= × 152 × 100° ×
180° 
A B (a) Tentukan nilai θ, dalam radian. 8 cm 2
O O 2
Determine the value of θ, in radian. 1
θ O 1.22 rad = × 82 × 1.22 m
15 c 100° = 196.3 cm2
D (b) Cari panjang lengkok AD jika AB = 18 cm. 2
C
Find the length of arc AD if AB = 18 cm. = 39.04 cm2

(a) Katakan OA = j, maka AB = 2j, OB = 3j (b) Jika AB = 18 cm, maka OA = 9 cm.


dan panjang lengkok BC = j. 2. 3.
Panjang lengkok AD = 9 × 1 Luas Luas
Panjang lengkok BC = OB × θ 3 π
1 π 1
= × 202 × (360° – 45°) ×
j = 3j × θ
= 3 cm = × 102 × 45° 20 cm
2 180°
O 2 3
= 1 × 400 × 315° × π
π
θ = 1 rad cm – rad
3
= 52.36 cm2 O
3 10 2 180°
= 1 099.6 cm2

2. Rajah di bawah menunjukkan dua sektor, OAD dan OBC, berpusat O. Diberi OA : AB = 3 : 2, cari
The diagram shows two sectors, OAD and OBC, with centre O. Given OA : AB = 3 : 2, find B. Selesaikan masalah yang berikut.
Solve the following problems. [Guna/Use π = 3.142] 3.1
O
(a) panjang OA dan panjang AB.
1.4 the lengths of OA and AB. 1. Rajah di bawah menunjukkan sektor OAB. 2. Rajah di bawah menunjukkan sektor POQ.
A rad D
(b) perimeter kawasan berlorek. The diagram shows a sector OAB. The diagram shows a sector POQ.
B C the perimeter of the shaded region. (a) Cari luas sektor OAB jika Q
A (a) Cari luas sektor POQ jika
7 cm
10 cm Find the area of sector OAB if Find the area of sector POQ if
θ
(i) θ = 0.5 rad. j
(i) j = 8 cm.
O
(ii) θ = 1.1 rad. 1.05 rad (ii) j = 18 cm.
(a) OB = 7 = 5 cm (b) Panjang lengkok AD = 3 × 1.4
B P O
1.4
OA = 3 = 4.2 cm (b) Cari nilai θ jika luas sektor OAB ialah (b) Cari jejari sektor POQ jika luas sektor ialah
OB 3 + 2 Perimeter kawasan berlorek Find the value of θ if the area of sector OAB is Find the radius of sector POQ if the area of the sector is
= 7 + 4.2 + 2 + 2 (i) 61.5 cm2. (ii) 67.5 cm2. (i) 42.525 cm2. (ii) 134.4 cm2.
OA = 3 × 5 = 3 cm = 15.2 cm
5
Maka, AB = 5 – 3 = 2 cm 1 × 82 × 1.05
(a) (i) Luas sektor OAB = 1 × 102 × 0.5 (a) (i) Luas sektor POQ =
2
2
3. Rajah di bawah menunjukkan satu sektor berpusat O. Jika ΔOAB ialah segi tiga sama sisi, cari = 25 cm2 = 33.6 cm2
The diagram shows a sector with centre O. If ΔOAB is an equilateral triangle, find
(ii) Luas sektor OAB = 1 × 102 × 1.1 1
(ii) Luas sektor POQ = × 182 × 1.05
2 2
A (a) ∠AOB, dalam radian. = 55 cm2 = 170.1 cm2
∠AOB, in radians.
12 cm (b) perimeter tembereng berlorek. (b) (i) 1 j 2θ = 61.5 1 × j 2 × 1.05 = 42.525
2 (b) (i)
the perimeter of the shaded segment. 2
(Berikan jawapan dalam sebutan π.) 1
× 102 × θ = 61.5 42.525 × 2
O B
2 j2 =
(Give the answers in terms of π.) 1.05
θ = 61.5 = 81
(a) OAB ialah segi tiga sama sisi. Maka, (b) AB = 12 cm 50 j = 9 cm
= 1.23 rad
∠AOB = 60° × π Panjang lengkok AB = 12 × π (ii)
1 2
j θ = 67.5 (ii) 1 × j2 × 1.05 = 134.4
180° 3 2 2
= 4π cm
= π rad j2 = 134.4 × 2
1
× 102 × θ = 67.5
3 Perimeter tembereng berlorek 2 1.05
= (12 + 4π) cm θ = 67.5 = 256
50 j = 16 cm
= 1.35 rad

111 112

28
C. Cari luas bagi setiap tembereng berlorek yang berikut. D. Selesaikan masalah yang berikut.
Find the area of each of the following shaded segments. 3.2 Solve the following problems. 3.3

CONTOH 1. Rajah di sebelah menunjukkan satu bulatan berpusat O. Diberi luas sektor OAB
1. O
13 cm ialah 72.11 cm2 dan luas tembereng berlorek ialah 32.49 cm2.
B
5 cm O The diagram shows a circle with centre O. Given the area of sector OAB is 72.11 cm2 and
A M B the area of the shaded segment is 32.49 cm2.
102° (a) Cari panjang OA.
10 cm M
A 14 cm B Find the length of OA.
80° (b) Hitung tinggi bagi ΔAOB, dengan AB ialah tapaknya.
Dalam ΔOAM, Calculate the height of ΔAOB, which has AB as its base.
O A
10 cm AM 2 = 132 – 52
= 144 (a) Luas sektor OAB = 72.11 cm2 (b) Luas ΔOAB
1 (OA2) 102° × π = 72.11 = Luas sektor OAB – Luas tembereng berlorek
Dalam ΔOAM,
AM kos 40° =
OM
10
AM = 12 cm
AB = 2 × 12 2  180°  = 72.11 – 32.49
sin 40° =
= 24 cm 72.11 × 180 × 2 = 39.62 cm2
10 OM = 10 kos 40° OA2 =
AM = 10 sin 40° 102 × π
= 7.660 cm kos ∠AOM = 5 1 (AB)(Tinggi) = 39.62
AB = 2 × 10 sin 40° 13 OA = 81.012 2
= 12.86 cm ∠AOM = 67.38° = 9.001 cm
∠AOB = 2 × 67.38° Tinggi = 2 × 39.62 = 5.66 cm
Luas tembereng berlorek 14
= 134.76°
= Luas sektor OAB – Luas ΔOAB
1 1 Luas tembereng berlorek 2. A Rajah di sebelah menunjukkan satu sektor AOB yang berpusat O. Diberi panjang
= j 2 θ – (AB)(OM) lengkok AB ialah 14.4 cm. Cari luas tembereng berlorek.
2 2 = 1 j 2θ – 1 (AB)(OM) 12 cm
The diagram shows a sector AOB with centre O. Given the length of arc AB is 14.4 cm.
2 2
= 1 (102)80 × π  – 1 (12.86)(7.660) M
= 1 (132)134.76° × π  – 1 (24)(5)
O Find the area of the shaded segment.
2 180 2
2 180° 2
= 69.81 – 49.25
= 138.74 cm2
= 20.56 cm 2
B

Katakan ∠AOB = θ AB = 2 × AM = 13.556 cm


2. A 3. 12 × θ = 14.4
A
M
B θ = 1.2 rad OM = 122 – 6.7782 = 9.902 cm

O 60° Dalam ΔOAM, AM = sin 0.6 rad Luas tembereng berlorek


M 15 cm 100° 12
AM = 12 sin 0.6 rad = 1 (122)(1.2) – 1 (13.556)(9.902)
13 cm
O 2 2
B
= 6.778 cm = 19.28 cm2
Dalam ΔOAM,
Dalam ΔOAM, AM
AM sin 50° = 3. Rajah di sebelah menunjukkan satu bulatan berpusat O. Diberi jejari bulatan itu
sin 30° = 15
13 AM = 15 sin 50° 30° ialah 15 cm. Cari
AM = 13 sin 30° AB = 2 × 15 sin 50° O The diagram shows a circle with centre O. Given the radius of the circle is 15 cm. Find
AB = 2 × 13 sin 30° = 22.98 cm
(a) luas sektor OAB. (b) luas tembereng berlorek.
= 13 cm the area of sector OAB. the area of the shaded segment.

OM kos 50° = OM A B
kos 30° = 15
13 OM = 15 kos 50°
OM = 13 kos 30° (a) ∠AOB = 2 × 30° (b) Dalam ΔOAB,
= 9.642 cm O
= 11.258 cm = 60° AM = 15 sin 30°
Luas tembereng berlorek AB = 2 × 15 sin 30° = 15 cm

30°
30°
Luas tembereng berlorek 1 π – 1 (22.98)(9.642) Luas sektor OAB
= (152)100° × OM = 15 kos 30° = 12.99 cm
180°  2
15 cm 15 cm
1 π – 1 (13)(11.258)
= (132)60° ×
180°  2
2 π
= 1 (152) 60° ×
180° 
2 Luas tembereng berlorek
= 85.56 cm2 2
= 15.31 cm2 = 117.8 – 1 (15)(12.99)
= 117.8 cm2 2 A M B

= 20.38 cm2

113 114

PRAKTIS FORMATIF Kertas 1 ANALISIS SOALAN SPM 5. Rajah di bawah menunjukkan sektor OQR dan sektor 7. Rajah di bawah menunjukkan sebuah sektor OAB
Subtopik 2013 2014 2015 2016 OPS dengan pusat O. KLON
SPM
dengan pusat O dan berjejari 10 cm.
Jawab semua soalan. 8.1 S. 17(a) – S. 18(a) – The diagram shows the sectors OQR and OPS with centre O. ’15 The diagram shows a sector OAB with centre O and a radius
8.2 S. 17(b) – – S. 19(a)
Q of 10 cm.
Answer all the questions. 8.3 – S. 20 S. 18(b) S. 19(b)
A
P
1. Rajah di bawah menunjukkan sektor POQ bagi satu
3. Rajah di bawah menunjukkan sektor KOL bagi satu O θ
bulatan berpusat O.
The diagram shows a sector POQ of a circle with centre O. bulatan berpusat O. S
The diagram shows a sector KOL of a circle with centre O. e
P R
B O
Diberi bahawa OP = 6 cm, nisbah OP : PQ = 3 : 2 dan Diberi OA = OB dan ∠OAB = 50°, cari
K L luas kawasan berlorek ialah 43.64 cm2. Cari Given that OA = OB and ∠OAB = 50°, find
It is given that OP = 6 cm, the ratio of OP : PQ = 3 : 2 and the
[Guna/Use π = 3.142]
θ J M area of the shaded region is 43.64 cm2. Find
O R Q (a) nilai θ, dalam radian.
1.62 rad
(a) panjang OQ, dalam cm. 8.1 the value of θ, in radians.
8.1 the length of OQ, in cm.
Diberi bahawa OR = 16 cm dan OP = 20 cm. Cari O (b) luas kawasan berlorek.
It is given that OR = 16 cm and OP = 20 cm. Find (b) nilai θ, dalam radian. 8.3 the area of the shaded region.
Diberi bahawa OJ = JK = OM = ML = 4 cm. Cari 8.3 the value of θ, in radian.
[Guna/Use π = 3.142] It is given that OJ = JK = OM = ML = 4 cm. Find [4]
[4]
(a) nilai θ, dalam radian. (a) panjang, dalam cm, lengkok KL.
8.1 the value of θ, in radian.
(a) OP : PQ = 3 : 2 (a) OAB ialah Δ sama kaki.
8.2 the length, in cm, of arc KL.
θ = 180° – 50° – 50°
(b) perimeter, dalam cm, kawasan berlorek. (b) luas, dalam cm2, kawasan berlorek. OQ = 5 × 6 cm = 10 cm
8.2 the perimeter, in cm, of the shaded region. 8.3 the area, in cm2, of the shaded region. 3 = 80°
[4] [4] (b) Luas kawasan berlorek = 80 × 3.142
(a) kos θ = 16 = 0.8 (a) Panjang lengkok KL = 8 × 1.62 = Luas sektor OQR – Luas sektor OPS 180
20 P = 1.396 rad
= 12.96 cm
θ = 0.6435 rad 43.64 = 1 (102)θ – 1 (62)θ (b) Luas kawasan berlorek
20 cm (b) Luas kawasan berlorek 2 2
(b) Panjang lengkok PQ 43.64 = 50θ – 18θ = Luas sektor OAB – Luas ΔOAB
= Luas sektor KOL – Luas ΔOJM
= 20(0.6435) θ 32θ = 43.64 = 1 (102)(1.396) – 1 (10)(10) sin 80°
= 12.87 cm O
16 cm R Q = 1 (82)(1.62) – 1 (42) sin 1.62 rad θ = 1.364 rad 2 2
2 2 = 20.56 cm2
Dalam ΔOPR, = 43.85 cm2 6. Rajah di bawah menunjukkan pandangan hadapan
KLON
PR = 202 – 162 = 12 cm2 SPM sebahagian lukisan mural berbentuk segi empat sama 8. Rajah di bawah menunjukkan sebuah bulatan dengan
’14
8.3
pada sebuah dinding koridor. PT adalah lengkok KLON pusat O dan jejari 12 cm.
Perimeter kawasan berlorek 4. Rajah di bawah menunjukkan sektor OAB bagi sebuah bulatan berpusat Q dan QT adalah lengkok bulatan SPM
’16 The diagram shows a circle with centre O and a radius of 12 cm.
= PR + RQ + Lengkok PQ KBAT
8.3 bulatan berpusat O dan sektor PQR bagi sebuah berpusat P. A
= 12 + 4 + 12.87 bulatan berpusat P. The diagram shows part of the front view of a square-shaped
= 28.87 cm The diagram shows a sector OAB of a circle with centre O and mural art on a wall of a corridor. PT is an arc of a circle with
ș B
a sector PQR of a circle with centre P. centre Q and QT is an arc of a circle with centre P.
2. Rajah di bawah menunjukkan sektor OPQ dengan 8m O
P P Q
KLON
SPM
pusat O.
’13 The diagram shows a sector OPQ with centre O.
A 8 cm B Diberi panjang lengkok minor AB ialah 20 cm.
T Given the length of the minor arc AB is 20 cm.
Q R
12 cm Q R
[Guna/Use π = 3.142]
S
150° 9 cm (a) Nyatakan nilai θ, dalam radian.
P
O
Kawasan berlorek merupakan bahagian yang perlu 8.2 State the value of θ, in radians.
O
dicat semula. Kira luas, dalam m2, kawasan itu. (b) Cari luas sektor major OAB, dalam cm2, betul
(a) Cari ∠POQ, dalam sebutan π radian. Diberi ∠AOB = 1.6 radian dan ∠QPR = 0.45 radian. The shaded region shows the part that needs to be repainted. 8.3
kepada empat angka bererti.
8.1 Find ∠POQ, in terms of π radians. Cari luas, dalam cm2, kawasan berlorek. Calculate the area, in m2, of that region. Find the area of the major sector OAB, in cm2, correct to
(b) Cari perimeter, dalam cm, sektor OPQ. Given ∠AOB = 1.6 radians and ∠QPR = 0.45 radian. Find the [4] four significant figures.
8.2Find the perimeter, in cm, of the sector OPQ. area, in cm2, of the shaded region. Luas segi empat sama PQRS = 8 × 8 = 64 m2 [4]
[Guna/Use π = 3.142] [4] [3] π (a) 12θ = 20
Luas sektor PQT = 1 × 8 × 8 × = 33.5 m2
Luas kawasan berlorek 2 3
(a) ∠POQ = 150° × π Luas tembereng TQ = 33.5 – Luas ΔPTQ θ = 20 = 1.667 rad
= Luas sektor OAB – Luas sektor PQR 12
180°
= 5 π rad = 1 (122)(1.6) – 1 (82)(0.45) = 33.5 – 1 × 8 × 8 × sin 60° (b) Luas sektor major OAB
6 2 2 2
5 = 100.8 cm2 = 5.80 m2 = 1 × 122 × (2π – 1.667)
(b) Perimeter sektor OPQ = 9 + 9 + 9 
6 
π 2
Luas kawasan yang perlu dicat semula = 64 – 33.5 – 5.80 = 332.4 cm2 (4 a.b.)
= 41.57 cm = 24.69 m2

115 116

29
PRAKTIS FORMATIF Kertas 2 ANALISIS SOALAN SPM 3. Rajah di bawah menunjukkan semibulatan AEB, 4. Rajah di bawah menunjukkan semibulatan OPTS
Subtopik 2013 2014 2015 2016
KLON
SPM
dengan pusat O dan sukuan bulatan DCE, dengan KLON
SPM
dengan pusat O dan berjejari 10 cm. QST ialah sektor
Jawab semua soalan. 8.1 S. 11(a) S. 11(a) – S. 7(a) ’13 pusat C. ’14 sebuah bulatan berpusat S dan R ialah titik tengah OP.
8.2 S. 11(b) S. 11(b) S. 6(a) S. 7(b) The diagram shows a semicircle AEB, with centre O and the The diagram shows a semicircle OPTS with centre O and
Answer all the questions. KBAT
8.3 S. 11(c) S. 11(c) S. 6(b) S. 7(c)
quadrant of a circle DCE, with centre C. a radius of 10 cm. QST is a sector of a circle with centre S
1. Dalam rajah di bawah, AOBC ialah satu semibulatan and R is the midpoint of OP.
2. Rajah di bawah menunjukkan sektor OPQ bagi satu
berpusat O dengan jejari 6 cm. BPD ialah sektor satu T
bulatan berpusat O dan berjejari 18 cm. Titik R E
bulatan berpusat P dengan jejari 9 cm.
In the diagram, AOBC is a semicircle with centre O and a radius terletak pada OP. 6c
The diagram shows a sector OPQ of a circle, centre O with m
of 6 cm. BPD is a sector of a circle with centre P and a radius 3 cm
a radius of 18 cm. Point R lies on OP. θ
of 9 cm.
D Q D A C O B

C Hitung P Q R O S
Calculate
Dengan menggunakan π = 3.142, hitung
[Guna/Use π = 3.142] By using π = 3.142, calculate

θ
θ O (a) nilai θ, dalam radian. (a) ∠TOR, dalam radian.
P R 8.1 the value of θ, in radians. [2]
A P O B 8.1 ∠TOR, in radians. [2]
(b) perimeter, dalam cm, seluruh rajah itu. (b) panjang, dalam cm, lengkok TQ.
Diberi bahawa OC berserenjang dengan AOB. Hitung Diberi bahawa OR = RQ = 12 cm. Hitung 8.2 the perimeter, in cm, of the whole diagram. [4]
It is given that OC is perpendicular to AOB. Calculate It is given that OR = RQ = 12 cm. Calculate 8.2 the length, in cm, of the arc TQ. [4]
(c) luas, dalam cm2, kawasan berlorek itu. (c) luas, dalam cm2, kawasan berlorek.
[Guna/Use π = 3.142] [Guna/Use π = 3.142] 8.3 the area, in cm2, of the shaded region. [4] 8.3 the area, in cm2, of the shaded region. [4]
(a) nilai θ, dalam radian. (a) nilai θ, dalam radian, betul kepada tiga tempat
8.1 the value of θ, in radians. [2] 8.1
perpuluhan. (a) ∠EOC = sin–1  3  (a) Dalam ΔTOR,
(b) perimeter, dalam cm, rantau berlorek. the value of θ, in radian, correct to three decimal places. 6
8.2
the perimeter, in cm, of the shaded region. [4] [2] = 30° kos ∠TOR = 5 = 1
10 2
(c) luas, dalam cm2, rantau berlorek. π
(b) perimeter, dalam cm, kawasan berlorek. = rad ∠TOR = 60°
8.3
the area, in cm2, of the shaded region. [4] 8.2
the perimeter, in cm, of the shaded region. [3] 6
= 60 × 3.142
(c) luas, dalam cm2, kawasan berlorek. θ = π – π = 2.618 rad 180
(a) OP = PB – OB = 9 – 6 = 3 cm 8.3
the area, in cm2, of the shaded region. [5] 6 = 1.047 rad
OC = 6 cm Q (b) Panjang OC = 6 kos 30°
tan θ = OC = 6 = 2
D (b) TOS ialah segi tiga sama kaki dan ∠TOS = 120°.
9 cm = 5.196 cm
OP 3 C Maka, ∠TSO = 30°
θ = 1.107 rad M Perimeter seluruh rajah = 0.5237 rad
9 cm = Lengkok BE + Lengkok DE + CD + OC + OB
(b) Panjang lengkok BC 6 cm Panjang lengkok TQ = 10 × 0.5237
π θ θ
O = 6(2.618) + 1 (2)(3.142)(3) + 3 + 5.196 + 6 = 5.237 cm
=6× A P O B P 6 cm R 12 cm 4
2 9 cm = 34.62 cm (c) Luas kawasan berlorek
= 9.426 cm Lukis garis lurus MR yang berserenjang dengan OQ.
(c) Luas tembereng EAC = Luas sektor QST – Luas ΔTRS
Panjang lengkok BD = 9 × 1.107 (a) Dalam ΔOMR,
= 9.963 cm = Luas sektor OAE – Luas segi tiga OCE Dalam ΔTOR, TR2 = 102 – 52
kos θ = 9 = 0.75
Dalam ΔOPC, PC = 6 2 + 3 2
12 = 1 (62) π  – 1 (5.196)(3) = 75
θ = 0.7227 rad 2 6 2 TR = 8.660 cm
= 6.708 cm = 1.632 cm2
= 0.723 rad
CD = PD – PC = 9 – 6.708 Luas ΔTRS = 1 × 8.660 × 15
(b) Perimeter kawasan berlorek Luas kawasan berlorek 2
= 2.292 cm = PR + RQ + Lengkok PQ = Luas sukuan bulatan DCE = 64.95 cm2
Perimeter rantau berlorek = 6 + 12 + 18(0.723) – Luas tembereng EAC
= 31.01 cm Dalam ΔTOS, TS2 = 75 + 152
= Lengkok BC + Lengkok BD + CD = 1 (3.142)(32) – 1.632 = 300
= 9.426 + 9.963 + 2.292 (c) Luas kawasan berlorek 4
= 21.681 cm = Luas sektor OPQ – Luas ΔOPR Luas sektor QST = 1 × TS2 × 0.5237
= 5.438 cm2 2
(c) Luas rantau berlorek = 1 (182)(0.723) – 1 (18) 122 – 92 = 1 × 300 × 0.5237
= Luas sektor BPD – Luas sukuan bulatan OBC 2 2 2
– Luas ΔOPC = 45.69 cm2 = 78.56 cm2
π
= 1 (92)(1.107) – 1 (62)   – 1 (3)(6) Luas kawasan berlorek = 78.56 – 64.95
2 2 2 2
= 7.556 cm2 = 13.61 cm2

117 118

5. Zahara ingin membuat topi berbentuk kon seperti 6. Rajah di bawah menunjukkan sektor AOB dengan
KLON
SPM
yang ditunjukkan dalam Rajah I. Tinggi kon itu KLON
SPM
pusat O dan sektor ABC dengan pusat B. FOKUS KBAT
’15 ialah 26 cm. ’16 The diagram shows a sector AOB with centre O and sector
Zahara wants to make a cap in the shape of a cone as shown ABC with centre B.
KBAT
in Diagram I. The height of the cone is 26 cm. A Kemahiran Kognitif: Mengaplikasi
O
Konteks: Luas Sektor, Panjang Lengkok

Lebar Dalam rajah di bawah, ABCD mewakili sebidang tanah yang berbentuk segi empat tepat. OPQR ialah sebuah sektor
Width berpusat O.
40° In the diagram, ABCD represents a piece of rectangular land. OPQR is a sector with centre O.
O B
C

18.8 cm Panjang Diberi bahawa OA = 18 cm dan ∠AOB = 40°. Cari A P B


I
Length It is given that OA = 18 cm and ∠AOB = 40°. Find
II [Guna/Use π = 3.142]
Rajah II menunjukkan bentangan kon yang berbentuk (a) panjang AB, dalam cm, betul kepada satu tempat
12 m O Q
sektor bagi sebuah bulatan dengan pusat O yang 8.1 perpuluhan.
dilukis di atas sekeping kad berbentuk segi empat the length of AB, in cm, correct to one decimal place.
tepat. [2]
Diagram II shows the net of the cone in the form of a sector (b) perimeter, dalam cm, sektor ABC. D C
8.2 the perimeter, in cm, of sector ABC. [3] R
of a circle with centre O drawn on a rectangular card.
20 m
(a) Hitung nilai minimum, dalam cm, bagi panjang (c) luas, dalam cm2, rantau berlorek.
8.2
dan lebar kad itu kepada integer terdekat. 8.3 the area, in cm2, of the shaded region. [5]
Lengkok PQR menyentuh tepi BC di Q. Serene ingin memagari kawasan yang berlorek dan menanam pokok bunga
Calculate the minimum value, in cm, of length and of
di situ. Hitung
width of the card to the nearest integer. [5] (a) ΔAOB ialah segi tiga sama kaki dengan OA = OB. Arc PQR is touching the edge BC at Q. Serene intends to fence off the shaded area and plant some flowers there. Calculate
(b) Seterusnya, cari luas, dalam cm2, kad yang tidak 1 AB = 18 sin 20°
8.3
digunakan. (a) luas, dalam m2, tanah yang akan ditanami dengan pokok bunga.
2
Hence, find the area, in cm , of the card unused. [3]
2
AB = 2 × 18 sin 20° the area, in m2, of the land to be planted with flowers. Info KBAT
= 12.313 cm (b) jumlah kos untuk pagar, jika kos untuk memagar ialah RM50 per m. BC ialah tangen kepada lengkok PQR. O dan
(a) Panjang lengkok AB the total cost for fencing, if the cost for fencing is RM50 per m.
= 12.3 cm Q masing-masing ialah titik tengah bagi AD
= Lilitan tapak kon dan BC.
= 2πj 26 cm (b) ∠ABC = (180° – 40°) ÷ 2 BC is the tangent to the arc PQR. O and Q
are the midpoints of AD and BC respectively.
= 2π  18.8  = 70° (a) OP = OQ = 20 m
2 = 1.222 radian
= 59.07 cm 9.4 cm tan θ = 6
Perimeter sektor ABC 20
Panjang sendeng kon = AB + BC + Lengkok AC θ = 0.2915 rad
C O A
= 26 2 + 9.42 θ = 12.3 + 12.3 + 12.3 × 1.222 A P B
AP = 202 – 62 = 19.079 m
= 27.65 cm = 39.63 cm
20 m
Maka, OA = 27.65 cm Luas kawasan berlorek 6m
B
(c) Luas rantau berlorek
Panjang lengkok AB = jθ = Luas sektor ABC + Luas tembereng yang dibatasi = 2(Luas ABQO – Luas ΔOAP – Luas sektor OPQ) 12 m O θ
Q
Maka, 27.65θ = 59.07 oleh lengkok AB dan perentas AB = 2 20(6) – 1 (6)(19.079) – 1 (202)(0.2915)
θ = 2.136 rad 2 2
= 1 (12.32)(1.222) + (Luas sektor AOB = 8.926 m2
∠BOC = π – 2.136 = 1.006 rad = 57.63° 2 D C
– Luas segi tiga AOB) (b) ∠POR = 2(0.2915) = 0.583 rad
R
kos 57.63° = OC 20 m
27.65 = 92.44 +  1 (182) 40° × π  PB = RC = 20 – 19.079 = 0.921 m
OC = 14.80 cm 2 180°
Maka, panjang minimum kad – 1 (12.3)(18 kos 20°) Perimeter kawasan berlorek = 20(0.583) + 2(0.921) + 12
2 = 25.502 m
= 27.65 + 14.80 = 92.44 + (113.11 – 104.02)
= 42 cm [kepada integer terdekat] Jumlah kos pagar = 25.502 × RM50
= 101.5 cm2 = RM1 275.10
Lebar minimum kad = 28 cm
[kepada integer terdekat]
(b) Luas kad yang tidak digunakan
= (42 × 28) – Luas sektor OAB
= 1 176 – 1 (27.652)(2.136)
2
= 359.5 cm2

119 120

30
BAB 2. y = 4x 2 + x 3. y = 5 – 4x 2

9
PEMBEZAAN y = 4x 2 + x ...... ➀ y = 5 – 4x2 ...... ➀
DIFFERENTIATION y + δ y = 4(x + δ x)2 + (x + δ x) y + δ y = 5 – 4(x + δ x)2
= 5 – 4[x2 + 2xδ x + (δ x)2]
= 4x 2 + 8xδ x + 4(δ x)2 + x + δ x ...... ➁ = 5 – 4x2 – 8xδ x – 4(δ x)2 ...... ➁
➁ – ➀: δ y = 8xδ x + 4(δ x)2 + δ x
9.1 Pembezaan dengan Menggunakan Prinsip Pertama δy ➁ – ➀: δ y = –8xδ x – 4(δ x)2
= 8x + 4δ x + 1 δy
A. Cari nilai bagi setiap yang berikut. δx = –8x – 4δ x
Find the value of each of the following. 1.1
dy δy δx
= had dy
CONTOH dx δx → 0 δ x = had (–8x – 4δ x)
1. had (2x – 7)
x→0
2. had
x→2
 2
x+6  = had (8x + 4δ x + 1) dx δx → 0
had 7x + 5 = had 7x + 5
 
x→∞ x x→∞ x x   =0–7 = 2
δx → 0
= 8x + 1
= –8x
2+6
= had 7 + 5
x→∞ x   = –7
= 2
=7+0 8
=7 = 1
4
9.2 Terbitan Pertama Fungsi Polinomial SPM K1 ‘13, ’14, ’16 K2 ‘13, ‘15

2 2
3. had  3x + 2x  4. had (x2 + 2x + 1) 5. had  x x– –x 3– 6  A. Cari dy atau f (x) bagi setiap fungsi yang berikut.
x→0 x x→4 x→3 dx
2 (x – 3)(x + 2)
= had  3x + 2x 
dy
= 16 + 8 + 1 = xhad
→3
Find
dx
or f (x) for each of the following functions. 2.1
x→0 x x x–3
= 25
= had (3 + 2x) = had (x + 2) CONTOH FAKTA UTAMA
x→0 x→3

=3 =3+2 (a) y = 2x4 (b) f(x) = – 34 = –3x–4 • y = k,


dy
=0
x dx
=5 dy
= 4(2x4 – 1) f (x) = –4(–3x–4 – 1) dy
dx • y = ax , n
dx
= nax n – 1
= 4(2x3) = 12x–5
12 dy
= 8x3 = 5 • y = a(bx + c)n ,
dx
= an(bx + c)n – 1(b)
B. Tentukan terbitan pertama bagi setiap yang berikut dengan menggunakan prinsip pertama. x
Determine the first derivative of each of the following by using the first principle. 1.4

CONTOH 1. y = 15x 2. y = –4x 2 3. y = –16x 3


1. y = 5x
y = 3x 2 y = 5x ...... ➀ dy dy dy
= 15x1 – 1 = 2(–4x 2 – 1) = 3(–16x 3 – 1)
y = 3x 2 ...... ➀ y + δ y = 5(x + δ x) dx dx dx
y + δy = 3(x + δx)2 = 5x + 5δ x ...... ➁ = 15x0 = 2(–4x) = 3(–16x 2)
= 3[x2 + 2xδx + (δx)2] ➁ – ➀: δ y = 5δ x = 15 = –8x = –48x 2
= 3x2 + 6xδx + 3(δx)2 ...... ➁ δy
=5
δx
➁ – ➀: δy = 6xδx + 3(δx)2
δy = 6x + 3δx Bahagikan kedua-dua dy δy
= had
δx
belah persamaan dx δx → 0 δ x 5 = –2
dengan δ x. 4. f (x) = 18 5. f (x) = 5x 6. f (x) = 2 3 = 2 x –3
dy = δy = had 5 x2 5x 5
had δx → 0
dx δx → 0 δx
=5 f (x) = 0 f (x) = –2(5x –2 – 1
) f (x) = –3  2 x –3 – 1
= had (6x + 3δx) 5
δx → 0
= –10x –3
= 6x = – 6 x –4
= – 103 5
δ x = Tokokan kecil dalam x
x = – 64
δ y = Tokokan kecil dalam y
5x

121 122

dy D. Bezakan setiap fungsi algebra yang berikut terhadap x.


B. Cari nilai atau f (x) untuk nilai x yang diberikan. Differentiate each of the following algebraic functions with respect to x. 2.3
dx
dy
Find the value of or f (x) for the given value of x. 2.2 CONTOH FAKTA UTAMA
dx
3 2
1 2 (b) 4x – 2x • y = f(x) ± g(x)
CONTOH 1. y = 2x3 ; x = 2 2. y = –3x2 ; x = 5 (a) 3x4 + x3 + x
2 x dy
= f (x) ± g(x)
d
 1

dx
y = 3x 2 ; x = 4 3x4 + x3 + x2 d 4x3 – 2x2
 
dy dy d
= 2(3x3 – 1) = 6x 2 = –3(2x2 – 1) = –6x dx 2 = (4x2 – 2x)
dy = 3(2x2 – 1 dx dx dx x dx d
• [f(x) ± g(x)]
 
) = 6x d d d 1 2 dx
dx = (3x4) + (x3) + x =
d (4x2 d
) – (2x)
Apabila x = 2, Apabila x = 5, dx dx dx 2 = f (x) ± g(x)
dx dx
Apabila x = 4, dy dy = –6(5) = –30 = 12x3 + 3x2 + x = 8x – 2
= 6(2)2 = 24
dy = 6(4) = 24 dx dx 3
dx 1. –2x3 + 22 + 6x 2. x – 2x +1 3. 33 – 4x
x x2 x
d d x 3 – 2x + 1 d 3
dx  
–2x 3 + 22 + 6x 
dx  dx x 3
3. f (x) = –x ; x = 6
3 8
4. f (x) = 2 ; x = –2
2
5. f(x) = 3 ; x = 1 – 4x
x x2
x 3x
d d d
x – 2 + 12 
dx 
f (x) = –3x 3 – 1 = –3x 2 8 = 8x –2 2 = 2 x –3 = (–2x 3 + 2x –2 + 6x) = = (3x –3 – 4x)
f (x) = f (x) = dx x x dx
x2 3x3 3 = –6x 2 – 43 + 6 d
f (6) = –3(6)2 = –108 x = (x – 2x –1 + x –2) = – 94 – 4
dx x
f (x) = –16x –3
= – 163 f (x) = 2 –3x –4 = – 24
x 3 x = 1 + 22 – 23
x x
f (–2) = – 16 3 = 2 f (1) = – 24 = –2
(–2) 1

dy
E. Cari bagi setiap fungsi yang berikut.
dx
C. Cari nilai f '(x) untuk nilai x yang diberikan. dy 2.3
2.2
Find for each of the following functions.
Find the value of f'(x) for the given value of x. dx

CONTOH 1. Diberi f (x) = –4x2, cari f (3). 2. Diberi f (x) = 5x3, cari f (–2). CONTOH 1. y = x2(5x + 3) 2. y = (x + 3)2
Diberi f (x) = x 4, cari f (2). Given f (x) = –4x 2, find f (3). Given f (x) = 5x 3, find f (–2). y = x2(x + 2) = 5x 3 + 3x 2 = x 2 + 6x + 9
Given f (x) = x 4, find f (2).
f (x) = –8x f (x) = 15x 2 dy dy
y = x2(x + 2) = 15x 2 + 6x = 2x + 6
f (x) = 4x 4–1
dx dx
= x3 + 2x2
= 4x 3 f (3) = –8(3) f (–2) = 15(–2)2
= –24 = 60 dy
f (2) = 4(23) = 3x2 + 4x
dx
= 4(8)
= 32

3. Diberi f (x) = 2 x3, cari f (4). 4. Diberi f (x) = 44 , cari f (2). 5. Diberi f (x) = 3 2 , cari f (1).
3 x 4x 4. y = 2x + 3
  5. y = x3 6x – 1
 
2
3. y = (x – 3)(5 – 2x)
Given f (x) =
2 3
x , find f (4). Given f (x) =
4
, find f (2). Given f(x) =
3
, find f (1). x x
3 x4 4x2 = 11x – 2x 2 – 15
= 6x 4 – x 2
dy = 4x 2 + 12 + 92
f (x) = 2 (3x3 – 1) = 2x 2 f (x) = 44 = 4x –4 f (x) = 3 2 = 3 x –2 = 11 – 4x x
dy
3 x 4x 4 dx dy = 24x 3 – 2x
= 8x – 183 dx
f (4) = 2(4)2 = 32 f (x) = –16x –5 = – 165 f (x) = – 3 x –3 = – 3 3 dx x
x 2 2x
f (2) = – 165 = – 1 f (1) = – 3 3 = – 3
2 2 2(1) 2

123 124

31
F. Bezakan fungsi algebra yang berikut dengan menggunakan petua hasil darab. 2
Differentiate the following algebraic functions by using the product rule. 2.4 1. f (x) = 3x – 1 2. f (x) = x + 3
2x + 6 x–1
CONTOH 1. y = (3x + 6)(x + 4)
2 u = 3x – 1 dan v = 2x + 6 u = x2 + 3 dan v=x–1
du = 3 dv = 2 du = 2x dv = 1
y = (x2 – 2)(x3 + 1) u = 3x 2 + 6 dan v=x+4 dx dx dx dx
du = 6x dv = 1
Katakan u = x2 – 2 dan v = x3 + 1 dx dx
du dv f (x) = (2x + 6)(3) – (3x2 – 1)(2) (x – 1)(2x) – (x2 + 3)(1)
= 2x = 3x2 dy (2x + 6) f (x) =
dx dx = u dv + v du (x – 1)2
dx dx dx = 6x + 18 – 6x2 + 2 2
dy dv du (2x + 6) = x – 2x –2 3
=u +v = (3x 2 + 6)(1) + (x + 4)(6x) (x – 1)
dx dx dx 20
= 3x 2 + 6 + 6x 2 + 24x = (x – 3)(x + 1)
= (x2 – 2)(3x2) + (x3 + 1)(2x) (2x + 6)2 =
= 9x 2 + 24x + 6 (x – 1)2
= 3x4 – 6x2 + 2x4 + 2x
= 5x4 – 6x2 + 2x

2. y = (1 – 2x3)(x + 2) 3. y = (x2 – x)(3x2 + 2x)


3. f (x) = 3x 4. f (x) = 3x – 4
u = 1 – 2x 3 dan v=x+2 u = x2 – x dan v = 3x 2 + 2x
2x2 + 2 5 – 2x
du = –6x 2 dv = 1 du = 2x – 1 dv = 6x + 2
dx dx dx dx u = 3x dan v = 2x 2 + 2 u = 3x – 4 dan v = 5 – 2x
du = 3 dv = 4x du = 3 dv = –2
dy dy
= (1 – 2x 3)(1) + (x + 2)(–6x 2) = (x 2 – x)(6x + 2) + (3x 2 + 2x)(2x – 1) dx dx dx dx
dx dx
= 1 – 2x 3 – 6x 3 – 12x 2 = 6x 3 – 6x 2 + 2x 2 – 2x + 6x 3 + 4x 2 – 3x 2 – 2x
f (x) =
(2x 2 + 2)(3) – 3x(4x) f (x) = (5 – 2x)(3) – (3x2 – 4)(–2)
= –8x 3 – 12x 2 + 1 = 12x 3 – 3x 2 – 4x (2x 2 + 2)2 (5 – 2x)

= 6x 2 + 6 – 12x 2 = 15 – 6x + 6x2 – 8
(2x 2 + 2)2 (5 – 2x)
6 – 6x 2 = 7
= (5 – 2x)2
(2x 2 + 2)2
G. Bezakan fungsi algebra yang berikut dengan menggunakan petua hasil bahagi. 6(1 – x 2)
Differentiate the following algebraic functions by using the quotient rule. 2.5 =
4(x 2 + 1)2
CONTOH = 3(1 – x 2)
2(x 2 + 1)2
x2 + 3 x+4
(a) f(x) = (b) f (x) =
x–2 x2 + 1
Katakan u = x2 + 3 dan v=x–2 Katakan u=x+4 dan v = x2 + 1 5. f (x) = x3 6. f (x) = 4x
du = 2x dv = 1 du = 1 dv = 2x x2 + 1 x3 + x
dx dx dx dx u = x3 dan v = x2 + 1 = 4
du = 3x2 dv = 2x x2 + 1
du – u dv du dv dx dx
v v –u u=4 dan v = x2 + 1
f (x) = dx dx f (x) = dx 2 dx
v2 v 2 2
) – x3(2x) du = 0 dv = 2x
f (x) = (x + 1)(3x dx dx
(x – 2)(2x) – (x2 + 3)(1) (x2 + 1)(1) – (x + 4)(2x) (x2 + 1)2
= =
(x – 2)2 (x2 + 1)2 4 2 4
= 3x +23x –2 2x 2
2 2 (x + 1) f (x) = (x + 1)(0) – 4(2x)
=
2x – 4x – x – 3 x2 + 1 – 2x2 – 8x (x2 + 1)2
= 4 2
(x – 2)2 (x2 + 1)2 = x 2 + 3x 2
(x + 1) = – 2 8x 2
x2 – 4x – 3 –x2 – 8x + 1 (x + 1)
= =
(x – 2)2 (x2 + 1)2

125 126

dy I. Cari (i) kecerunan, (ii) persamaan, bagi tangen dan normal kepada lengkung pada titik yang diberikan.
H. Cari bagi setiap fungsi yang berikut. Find (i) the gradients, (ii) the equations, of the tangent and the normal to the given curve at the given point. 2.7 ; 2.8 ; 2.9
dx
dy 2.6
Find for each of the following functions. Kecerunan tangen (m1) dan normal (m2) Persamaan tangen Persamaan normal
dx
Gradients of tangent (m1 ) and normal (m2 ) Equation of the tangent Equation of the normal
CONTOH FAKTA UTAMA
(a) y = (x3 + 2x)2 (b) y = (2x 3 – 6)5 • Petua rantai:
CONTOH
Chain rule: y = 3x2 – 4x + 1 ; (1, 0)
dy = 2(x 3 d (x 3 dy = 5(2x 3 d
+ 2x)2 – 1 + 2x) – 6)5 – 1 (2x 3 – 6) y = a(bx + c)n
dy = 6x – 4 y – 0 = 2(x – 1) 1
y – 0 = – (x – 1)
dx dx dx dx dy d 2
= an(bx + c)n – 1 (bx + c) dx y = 2x – 2
= 2(x 3 + 2x)(3x 2 + 2) = 5(2x 3 – 6)4(6x3 – 1) dx dx 1 1
y=– x+
= an(bx + c)n – 1(b) Pada x = 1, dy = 6(1) – 4 = 2 2 2
= 2x(x 2 + 2)(3x 2 + 2) = 5(2x 3 – 6)4(6x2) dx
1
= 30x 2(2x3 – 6)4 Maka, m1 = 2 dan m2 = – .
2
y = (4x – x2)2 ; (3, 9)
1. y = (4x – 5)3 2. y = (2x + 3)3
dy y – 9 = 1 (x – 3)
dy dy = 2(4x – x2)(4 – 2x) y – 9 = –12(x – 3) 12
= 3(4x – 5)2(4) = 3(2x + 3)2(2) dx
dx dx dy
y – 9 = –12x + 36 y–9= 1 x– 1
= 12(4x – 5)2 = 6(2x + 3)2 Pada x = 3, = 2[4(3) – 32][4 – 2(3)] = –12 y = –12x + 45 12 4
dx
y = 1 x + 35
Maka, m1 = – 12 dan m2 = 1 . 12 4
12

3 ; 1, 3
y=
(x2 + x)2 4  
3. y = (3x + x2)4 4. y = (x3 – 3x + 2)5 dy y – 3 = – 9 (x – 1) y– 3 = 4 (x – 1)
= –6(x + x)–3(2x + 1) = –6(2x
2 + 1) 4 4 4 9
dy dy dx (x2 + x)3
= 4(3x + x 2)3(3 + 2x) = 5(x 3 – 3x + 2)4(3x 2 – 3) dy y – 3 = –9x + 9 y– 3 = 4x– 4
dx dx Pada x = 1, = –6[2(1) + 1] =–9 4 4 4 4 9 9
= 4(3 + 2x)(3x + x 2)3 = 5(3x 2 – 3)(x 3 – 3x + 2)4 dx (12 + 1)3 4
y = –9x + 3 y = 4 x + 11
Maka, m1 = – 9 dan m2 = 4 . 4 9 36
4 9

y = (x2 + 1)(3x – 4) ; (2, 10)

5. y = (4x3 – 3x)3 6. y = (x3 – x)4 dy y – 10 = – 1 (x – 2)


= (x2 + 1)(3) + (3x – 4)(2x) 23
dx y – 10 = 23(x – 2)
dy dy y – 10 = – 1 x + 2
= 3(4x3 – 3x)2(12x2 – 3) = 4(x 3 – x)3(3x 2 – 1) Pada x = 2,
dy
= (22 + 1)(3) + [3(2) – 4][2(2)] y – 10 = 23x – 46
dx dx 23 23
= 9x2(4x2 – 3)2(4x2 – 1) = 4(3x 2 – 1)(x 3 – x)3 dx y = 23x – 36
= 23 y = – 1 x + 232
23 23
Maka, m1 = 23 and m2 = – 1 .
23

x ; 3, 3
y=
x2 + 1 10 
7. y = (x2 – 2x + 4)3 8. y = (x2 + 3x + 1)2
dy
= (x2
+ 1)(1) – x(2x) y – 3 = – 2 (x – 3) y– 3 = 25 (x – 3)
dy dy dx (x2 + 1)2 10 25 10 2
= 3(x 2 – 2x + 4)2(2x – 2) = 2(x2 + 3x + 1)(2x + 3)
dx dx Pada x = 3,
dy (32 + 1) – 3[2(3)]
= y– 3 =– 2 x+ 6 y– 3 = 25 x – 75
= 3(x 2 – 2x + 4)2(2)(x – 1) dx (32 + 1)2 10 25 25 10 2 2
= 6(x – 1)(x 2 – 2x + 4)2
=– 8 =– 2 y = – 2 x + 27 y = 25 x – 186
100 25 25 50 2 5
Maka, m1 = – 2 dan m2 = 25 .
25 2

127 128

32
J. Selesaikan masalah yang berikut. Cari titik pusingan bagi setiap lengkung yang berikut. Seterusnya, tentukan sama ada setiap titik pusingan itu
Solve the following problems. 2.7 ; 2.8 ; 2.9
ialah titik maksimum atau titik minimum.
1. Diberi kecerunan tangen kepada lengkung 2. Diberi y = mx + n ialah persamaan tangen kepada Find the turning point(s) of each of the following curves. Hence, determine whether each turning point is a maximum or
a minimum point. 3.1 ; 3.2
y = 3x2 + 6x + 4 ialah 12 apabila x = p, dengan lengkung y = 3x2 – 5x + 2 pada titik (2, 4). Cari
keadaan p ialah pemalar. Cari nilai p. nilai m dan nilai n. CONTOH
Given the gradient of the tangent to the curve Given y = mx + n is the equation of the tangent to the curve
y = 3x2 + 6x + 4 is 12 when x = p, where p is a constant. y = 3x2 – 5x + 2 at the point (2, 4). Find the values of 9
y= +x
Find the value of p. m and n. x
dy 9 dy
y = 3x2 + 6x + 4 y = 3x2 – 5x + 2 = – 2 + 1. Pada titik pusingan, = 0. Apabila x = 3, Apabila x = –3,
dx x dx
dy dy 9 9
= 6x + 6 = 6x – 5 y= +3 y= + (–3)
dx dx Maka, – 92 + 1 = 0 ⇒ 92 = 1 3 –3
dy x x =6 = –3 – 3
Kecerunan tangen ialah 12 apabila x = p. Apabila x = 2, = 6(2) – 5 = 7 x2 = 9
dx = –6
6p + 6 = 12
6p = 6 Persamaan tangen ialah x = 3 atau –3 Maka, titik pusingan ialah (3, 6) dan (–3, –6).
p=1 y – 4 = 7(x – 2)
y – 4 = 7x – 14 Pada titik (3, 6) Pada titik (–3, –6)
y = 7x – 10 Nilai x x=2 x=3 x=4 x = –4 x = –3 x = –1
Maka, m = 7 dan n = –10.
Nilai
dy – 5 0
7 7
0 –8
dx 4 16 16
3. Diberi lengkung y = (x2 – 4)2 dan dy = mx(x2 – 4), dengan keadaan m ialah pemalar. Cari
dx
dy Lakaran tangen
Given a curve y = (x2 – 4)2 and = mx(x2 – 4), where m is a constant. Find
dx
(a) nilai m. (b) persamaan tangen kepada lengkung itu pada titik yang koordinat-x ialah 1. Maka, (3, 6) ialah titik minimum. Maka, (–3, –6) ialah titik maksimum.
the value of m. the equation of the tangent to the curve at the point where the x-coordinate is 1.
1. y = 2x2 + 8x + 3 2. y = 8x – x2
(a) y = (x2 – 4)2 (b) Apabila x = 1,
dy dy dy dy
= 2(x2 – 4)(2x) = 4(1)(12 – 4) = –12 dan y = (12 – 4)2 = 9 = 4x + 8 = 8 – 2x
dx dx dx dx
= 4x(x2 – 4) dy dy
Persamaan tangen ialah Pada titik pusingan, = 0. Pada titik pusingan, = 0.
dx dx
Bandingkan mx(x2 – 4) dengan 4x(x2 – 4). y – 9 = –12(x – 1) Maka, 4x + 8 = 0 Maka, 8 – 2x = 0
y – 9 = –12x + 12 Nilai x –3 –2 –1 Nilai x 3 4 5
Maka, m = 4 x = –2 dy x=4 dy
y = –12x + 21 Apabila x = –2, Nilai –4 0 4 Apabila x = 4, Nilai 2 0 –2
dx dx
y = 2(–2)2 + 8(–2) + 3 y = 8(4) – 42
Lakaran Lakaran
= –5 = 16
tangen tangen
9.3 Nilai Maksimum dan Nilai Minimum SPM K1 ‘14, ‘15 K2 ‘13, ‘16
Maka, titik pusingan Maka, titik pusingan
ialah (–2, –5). Maka, (–2, –5) ialah ialah (4, 16). Maka, (4, 16) ialah titik
FAKTA UTAMA titik minimum. maksimum.

• Nilai maksimum atau nilai minimum diperoleh apabila dy = 0.


1 3
dx dy 3. y = x – x2 – 3x
=0 Titik 3
dy dx minimum
The maximum or the minimum value is obtained when
dx
= 0.
dy dy
dy
= x 2 – 2x – 3 Apabila x = 3, y = 1 (3)3 – 32 – 3(3) = –9
dy dy 0 0 dx 3
0 0 dx dx
• Apabila nilai dy berubah daripada nilai positif kepada sifar dan
dx
dx Titik dx
dy dy Maka, titik pusingan ialah –1, 5  dan (3, –9).
maksimum =0 Pada titik pusingan, = 0. 3
kemudian kepada nilai negatif, nilai maksimum diperoleh. dx dx
dy Maka, x 2 – 2x – 3 = 0 Nilai x –2 –1 0 2 3 4
When the value of changes from positive to zero and then to negative, a maximum value is obtained.
dx dy
(x + 1)(x – 3) = 0 Nilai 5 0 –3 –3 0 5
• Apabila nilai dy berubah daripada nilai negatif kepada sifar dan kemudian kepada nilai positif, nilai minimum diperoleh. x = –1 atau 3 dx
dx
Lakaran tangen
When the value of
dy
changes from negative to zero and then to positive, a minimum value is obtained. Apabila x = –1,
dx
• Titik maksimum atau titik minimum juga disebut titik pusingan. y = 1 (–1)3 – (–1)2 – 3(–1) = 5 Maka, –1, 5  ialah titik maksimum dan
A maximum or a minimum point is also called a turning point. 3 3 3
(3, –9) ialah titik minimum.

129 130

9.4 Kadar Perubahan SPM K1 ‘14, ‘16 9.5 Perubahan Kecil dan Penghampiran
Selesaikan masalah yang berikut. Selesaikan masalah yang berikut.
Solve the following problems. 4.1 Solve the following problems. 5.1 ; 5.2

CONTOH 1. Isi padu sebuah silinder bertambah pada kadar CONTOH 1. Diberi y = 4x2 + 5x – 4, cari perubahan kecil
Jejari satu bulatan berkurang pada kadar 0.2 cm s–1. 324 cm3 s–1. Tinggi, h cm, silinder itu adalah dua Diberi y = x2 + 3x – 8, cari perubahan kecil dalam y dalam y apabila x bertambah daripada 3 kepada
Cari kadar perubahan luas bulatan itu apabila jejarinya kali jejarinya, r cm. Cari kadar perubahan jejari apabila x bertambah daripada 2 kepada 2.01. Seterusnya, 3.1. Seterusnya, cari nilai hampir bagi y selepas
ialah 3 cm. apabila jejarinya ialah 6 cm. cari nilai hampir bagi y selepas perubahan itu berlaku perubahan itu berlaku pada x.
The radius of a circle decreases at a rate of 0.2 cm s–1. Find the The volume of a cylinder increases at a rate of 324 cm3 s–1. pada x. Given y = 4x2 + 5x – 4, find the approximate change in y
rate of change in the area of the circle when the radius is 3 cm. The height, h cm, of the cylinder is twice the radius, r cm. Given y = x2 + 3x – 8, find the approximate change in y when when x increases from 3 to 3.1. Hence, find the approximate
Find the rate of change in the radius when the radius is 6 cm. x increases from 2 to 2.01. Hence, find the approximate value value of y after the change has occurred in x.
dr = –0.2 of y after the change has occurred in x.
dv = 324 cm3 s–1 y = 4x 2 + 5x – 4
dt
dt y = x2 + 3x – 8 dy
A = πr2 = 8x + 5
v = πr 2h = πr 2(2r) = 2πr 3 dy = 2x + 3 dx
dA = 2πr
dv = 6πr 2 dx
dr δ x = 3.1 – 3
dr δx = 2.01 – 2 = 0.1
dr = –0.2, dv = dv × dr
Apabila r = 3 dan = 0.01 dy
dt dt dr dt Apabila x = 3, δ y ≈ × δx
dx
dA = dA × dr Apabila x = 2, δy ≈ dy × δx = (8x + 5)(0.1)
Petua rantai Apabila r = 6, dx
dt dr dt = (2x + 3)(0.01) = (8 × 3 + 5)(0.1)
= 2πr × (–0.2) 324 = 6π(6)2 × dr = (2 × 2 + 3)(0.01) = 2.9
dt
= 2π(3) × (–0.2) = 0.07
dr = 324 Nilai hampir bagi y = y + δ y
= –1.2π cm2 s–1 dt = 4(32) + 5(3) – 4 + 2.9
216π Nilai hampir bagi y = y + δy
= 3 cm s–1 = 22 + 3(2) – 8 + 0.07 = 49.9
2π = 2.07

2. Diberi y = x3, dengan menggunakan pembezaan, dy


2. Jejari satu bulatan bertambah pada kadar 0.5 cm s–1. 3. Ukuran sebuah kuboid ialah y cm × y cm × 2y cm. cari nilai hampir bagi 2.13. 3. Diberi y = 52 , cari nilai apabila x = 4. Seterusnya,
Cari kadar perubahan luas bulatan itu ketika luasnya x dx
Isi padu kuboid itu bertambah pada kadar Given y = x3, by using differentiation, find the approximate 5
ialah 25π cm2. 3.6 cm3 s–1 ketika isi padunya ialah 432 cm3. Cari value of 2.13. cari nilai hampir bagi .
The radius of a circle increases at a rate of 0.5 cm s –1. Find kadar perubahan y pada ketika itu. (4.01)2
5 dy
the rate of change in the area of the circle when the area The measurements of a cuboid is y cm × y cm × 2y cm. y = x3 Given y = , find the value of when x = 4. Hence,
is 25π cm2. x2 dx
The volume of the cuboid increases at a rate of 3.6 cm3 s–1 dy
when the volume is 432 cm3. Find the rate of change in y = 3x 2 find the approximate value of
5
.
dA = dA × dj dx (4.01)2
at that instant.
dt dj dt Andaikan x berubah daripada 2 kepada 2.1.
δ x = 2.1 – 2 y = 52 = 5x –2
A = πj 2 Isi padu kuboid, V = y × y × 2y x
= 2y3 = 0.1 dy
25π = πj 2 = –10x –3 = – 103
Maka, 2y3 = 432 dy dx x
j 2 = 25 Apabila x = 2, δ y ≈ × δx
y3 = 216 dx dy
j=5 Apabila x = 4, = – 103 = –0.1563
y=6 = 3x 2 × δ x dx 4
dA = 2πj dV = 6y2 = 3(2)2 × 0.1 Andaikan x berubah daripada 4 kepada 4.01.
dj dy = 1.2 δ x = 4.01 – 4
Apabila j = 5, dA = 2π(5) = 10π dy = 0.01
dj = 1 Maka, 2.13 = y + δ y
dy
dV 6y2 = 23 + 1.2 δy ≈ × δx
dj dx
Diberi = 0.5 = 9.2
dt dy dV dy = –0.1563 × 0.01
= × = 3.6 × 1 2
Maka, dA = 10π × 0.5 = 5π cm2 s–1 dt dt dV 6y = –0.001563
dt dy 1 5 = y + δy
Apabila y = 6, = 3.6 × Nilai hampir bagi
dt 6(62) (4.01)2
=
1 s–1 = 52 – 0.001563
60 4
= 0.3109

131 132

33
4. Jejari satu bulatan berubah daripada 3 cm kepada 5. Isi padu sebuah kubus bertambah daripada 64 cm3 B. Tentukan titik pusingan bagi setiap lengkung yang diberikan di bawah dan nyatakan sama ada setiap titik
3.2 cm. Cari perubahan kecil bagi lilitan bulatan itu. kepada 64.5 cm3. Cari perubahan kecil bagi panjang pusingan itu ialah titik maksimum atau titik minimum.
The radius of a circle changes from 3 cm to 3.2 cm. Find setiap tepi kubus itu. Determine the turning points of each curve given below and state whether each turning point is a maximum or a minimum
point. 6.2
the approximate change in the circumference of the circle. The volume of a cube increases from 64 cm3 to 64.5 cm3.
Find the approximate change in the length of each side of CONTOH
δ r = 3.2 – 3 FAKTA UTAMA
the cube.
= 0.2 x3
y= – 9x • Untuk titik minimum: d 2y
0
Katakan panjang setiap tepi kubus itu ialah x cm. 3
p = 2πr For a minimum point: dx2
V = x3 dy
dp = x2 – 9 • Untuk titik maksimum: d 2y
= 2π dV = 3x 2 dx For a maximum point:
0
dr dx2
dx dy
dp Pada titik pusingan, = 0.
δp ≈ × δr δ V = 64.5 – 64 = 0.5 dx d 2y
dr = 2x
= 2π × 0.2 Maka, x2 – 9 = 0 dx2
Apabila x 3 = 64 x2 = 9 d 2y
= 0.4π cm 3 Apabila x = 3, = 2(3) = 6  0
x = 64 = 4 x = ±3 dx2
δ V ≈ dV 33
Apabila x = 3, y= – 9(3) = –18
δ x dx 3 Maka, (3, –18) ialah titik minimum.
0.5 (–3)3 d 2y
≈ 3x 2 Apabila x = –3, y = – 9(–3) = 18 Apabila x = –3, = 2(–3) = – 6  0
δx 3 dx2
δ x = 0.52 Maka, (3, –18) dan (–3, 18) ialah titik pusingan. Maka, (–3, 18) ialah titik maksimum.
3x
Apabila x = 4, δ x = 0.5 2 = 0.01 cm
3×4 1. y = 6x3 – 6x2 + 8 2. y = 1 x3 – 2x2
3
9.6 Pembezaan Peringkat Kedua SPM K2 ’13
dy dy
= 18x 2 – 12x = x 2 – 4x
dx dx
d2y dy
A. Cari atau f (x) bagi setiap fungsi yang berikut. dy Pada titik pusingan, = 0.
dx2 Pada titik pusingan, = 0. dx
d2y
dx
Find or f ″(x) for each of the following functions. 6.1 18x 2 – 12x = 0 x 2 – 4x = 0
dx2
6x(3x – 2) = 0 x(x – 4) = 0
CONTOH FAKTA UTAMA x = 0 atau 2 x = 0 atau 4
(a) y = 4x3 – 1 (b) f (x) = (5x + 2)3
3
Apabila x = 0, y = 0.
x •
d 2y
dx2
=
d dy
dx dx   Apabila x = 0, y = 6(0)3 – 6(0)2 + 8
dy = 12x2 f (x) = 3(5x + 2) (5) 2
Apabila x = 4, y = 1 (4)3 – 2(4)2
+ 12 • f(x) =
d =8 3
dx x = 15(5x + 2)2 dx
[f(x)]

= –10 2
3 2
d2y
= 24x – 23 Apabila x = 2 , y = 6 2  – 6 2  + 8
f (x) = 30(5x + 2)(5) d 2y
 
dy 2
2 • ≠ dx 3 3 3 3
dx x = 150(5x + 2) dx2
1
=7 Maka, (0, 0) dan 4, –10 2  ialah titik pusingan.
1. y = 3x3 – 2x2 + 7 2. y = (3x – 5)3 3. f (x) = (3x + 5)4 9 3
dy dy f (x) = 4(3x + 5)3(3) Maka, (0, 8) dan  2 , 7 1  ialah titik pusingan. d 2y
= 2x – 4
= 9x 2 – 4x = 3(3x – 5)2(3) 3 9 dx 2
dx dx = 9(3x – 5)2 = 12(3x + 5)3 d 2y
= 36x – 12 d 2y
Apabila x = 0, = 2(0) – 4
2
d y d 2y f (x) = 36(3x + 5)2(3) dx 2 dx 2
= 18x – 4 = 18(3x – 5)(3) 2
= –4  0
dx 2 dx 2 = 108(3x + 5)2 Apabila x = 0, d y2 = 36(0) – 12
= 54(3x – 5) dx
= –12  0 (0, 0) ialah titik maksimum.
x2 + 5 d 2y
4. f (x) = 1 x5 – 4x3 + 1 x 2 5. f (x) = 6. y = x3 + 1 (0, 8) ialah titik maksimum. Apabila x = 4, = 2(4) – 4
5 2 x x dx 2
d 2y =40
f (x) = x – 12x + x
4 2
f (x) = x + 5
dy = 3x 2 – 1 Apabila x = 2 , 2 = 36  2  – 12
x dx x2 3 dx 3
f (x) = 4x 3 – 24x + 1
f (x) = 1 – 52 d2y = 6x – – 2 = 12  0 4, –10 23  ialah titik minimum.
x dx 2  x 3
f (x) = 103 = 6x + 23  23 , 7 19  ialah titik minimum.
x x

133 134

4. Kenny mempunyai sekeping zink berbentuk segi 5. Akibat daripada peningkatan kos sara hidup, Yusuf
PRAKTIS FORMATIF Kertas 1 ANALISIS SOALAN SPM
KLON empat tepat dengan perimeter 28 cm. Dia ingin KLON telah menanam beberapa jenis sayur untuk kegunaan
Subtopik 2013 2014 2015 2016 SPM SPM
9.1 – – – – ’15 menggunakan kepingan zink itu untuk membina ’14 sendiri di kawasan lapang berbentuk segi empat tepat
Jawab semua soalan. 9.2 S. 19 S. 18(a) – S. 7 9.3 9.3

Answer all the questions. 9.3 – S. 17 S. 23 –


sebuah silinder yang terbuka pada kedua-dua hujung. di sebelah rumahnya. Dia ingin memagar kawasan itu
KBAT Cari panjang dan lebar, dalam cm, kepingan zink KBAT yang berukuran 12x m dan (4 – x) m. Cari panjang,
9.4 – S. 18(b) – S. 6
1. Kecerunan tangen kepada lengkung y = x2(3 + px) 9.5 – – – – itu supaya isi padu silinder yang dibentuk ialah dalam m, pagar yang perlu dia beli apabila luas
9.2
pada x = –1 ialah –3. Cari nilai p. 9.6 – – – – maksimum. kawasan itu adalah maksimum.
The gradient of the tangent to the curve y = x2(3 + px) at Kenny has a rectangular piece of zinc with a perimeter of Due to the high living cost, Yusuf has planted several types of
x = –1 is –3. Find the value of p. 3. Rajah di bawah menunjukkan sebahagian daripada 28 cm. He wants to use that piece of zinc to build an open vegetables for his own consumption on an empty rectangular
[3] KLON cylinder at both ends. Find the length and width, in cm, of plot of land beside his house. He wants to fence the land which
SPM lengkung y = 2x + 6 dan satu garis lurus.
the piece of zinc that makes the volume of the cylinder is has a dimension of 12x m and (4 – x) m. Find the length, in m, of
y = x2(3 + px)
’16 x–2 maximum. the fence he has to buy when the area of the land is maximum.
9.2
2x + 6
= 3x2 + px3 The diagram shows part of a curve y = and [4] [4]
x–2
dy a straight line. Luas kawasan, L = 12x(4 – x)
= 6x + 3px2 Silinder yang dibina:
dx y = 48x – 12x2
dy 2πj cm dL = 48 – 24x
Apabila x = –1, = –3.
dx dx
Maka, 6(–1) + 3p(–1)2 = –3 h cm Kepingan zink h cm Apabila L maksimum, dL = 0.
dx
–6 + 3p = –3 x 48 – 24x = 0
P O
3p = 3 24x = 48
p=1 Panjang = 2πj cm
j cm x=2
Lebar = h cm
Perimeter = 28 cm Panjang pagar yang perlu dibeli
Diberi bahawa garis lurus itu selari dengan tangen = 2 × [12(2) + (4 – 2)]
kepada lengkung itu pada titik P. Cari persamaan 2πj × 2 + h × 2 = 28
4πj + 2h = 28 = 2 × 26
garis lurus itu jika pintasan-y ialah 3. = 52 m
2. Titik (1, –5) terletak pada lengkung y = 2x2 – 7x. Cari It is given that the straight line is parallel to the tangent of the 2πj + h = 14
KLON
SPM
The point (1, –5) lies on the curve y = 2x2 – 7x. Find curve at point P. Find the equation of the straight line if the h = 14 – 2πj
’13 (a) kecerunan tangen kepada lengkung itu di titik P. y-intercept is 3.
9.2
the gradient of the tangent to the curve at point P. [4] Isi padu kon, V = πj2h
(b) persamaan normal kepada lengkung itu di titik P. = πj2(14 – 2πj) dy
the equation of the normal to the curve at point P. Pada titik P, y = 0. 6. Diberi x = t2 – 3 dan = 12t3, cari
[4] = 14πj2 – 2π2j3 KLON
dt
Maka, 2x + 6 = 0 dV = 28πj – 6π2j2
SPM
’14 Given that x = t2 – 3 and
dy
= 12t3, find
(a) y = 2x2 – 7x x–2 dj
dt
dy 2x + 6 = 0 = 2πj(14 – 3πj) dx
= 4x – 7 (a) .
dx 2x = –6 9.2 dt
x = –3 dV = 0 untuk V maksimum.
dy dj dy
Pada (1, –5): = 4(1) – 7 = –3 2x + 6 (b) dalam sebutan x.
dx y= dx
x–2 Maka, 2πj(14 – 3πj) = 0 9.4

(b) Kecerunan normal pada titik P = 1 dy 14 – 3πj = 0 dy


in terms of x.
3 = 2(x – 2) – (2x + 6)
3πj = 14 dx
dx (x – 2)2 [4]
Persamaan normal pada titik P ialah
= 2x – 4 – 2x2 – 6 j = 14 (a) x = t2 – 3

y – (–5) = 1 (x – 1) (x – 2)
Panjang = 2πj dx = 2t
3
= – 10 2 dt
y + 5 = 1x – 1 (x – 2) = 2π  14 
3 3 3π dy dy
Apabila x = –3,
dy
=– 10 =–2 (b) = × dt
y = 1 x – 16 dx (–3 – 2)2 5 = 28 dx dt dx
3 3 3
Pintasan-y = 3 = 12t3 × 1
atau 3y – x + 16 = 0 1 2t
= 9 cm
3 = 6t2
Maka, persamaan garis lurus itu ialah y = – 2 x + 3.
5 = 6(x + 3)
Lebar = 14 – 2πj
= 14 – 28
3
2
= 4 cm
3

135 136

34
7. Sebuah kotak mempunyai tapak segi empat sama 10 PRAKTIS FORMATIF Kertas 2
9. Diberi bahawa y = 5 – . Cari perubahan kecil ANALISIS SOALAN SPM
KLON
SPM
bersisi x cm dan tinggi 10 cm. Jika sisi tapaknya 9.5 x Subtopik 2013 2014 2015 2016
’16 bertambah pada kadar tetap 0.2 cm s–1, cari kadar dalam x, dalam sebutan p, apabila nilai y berubah
Jawab semua soalan.
9.1 – – – –
9.4
perubahan luas permukaan, dalam cm2 s–1, ketika daripada 3 kepada 3 + p. Answer all the questions.
9.2
9.3
S. 5(a)
S. 5(b)


S. 2


S. 6
isi padu kotak itu ialah 160 cm3. It is given that y = 5 –
10
. Find the small change in x, in terms 9.4 – – – –
A box has a square base of sides x cm and a height of 10 cm. x 1. Rajah di bawah menunjukkan pandangan hadapan 9.5 – – – –
of p, when the value of y changes from 3 to 3 + p.
If the side of the base increases at a constant rate of KLON sebuah pintu. 9.6 S. 5(c) – – –
0.2 cm s–1, find the rate of change in the surface area, [3] SPM
’16 The diagram shows the front view of a door.
in cm2 s–1, when the volume of the box is 160 cm3. y = 5 – 10 = 5 – 10x–1 9.3

[3] x (b) dL = 8 – πx – 4x
dy dx
Isi padu kotak = 160 cm3 = 10x–2 = 102 Apabila luas permukaan hadapan pintu itu
dx x
10x2 = 160
x2 = 16 Apabila y = 3, ym maksimum, dL = 0.
dx
x=4 3 = 5 – 10 Maka, 8 – πx – 4x = 0
2 x x(π + 4) = 8
Luas permukaan, L = 2x + 4(10x) 10 = 2 2x m
= 2x2 + 40x x Lengkok bagi pintu itu ialah satu semibulatan. x= 8
dL = 4x + 40 π+4
x=5 Perimeter pintu itu ialah 8 meter.
dx = 8
The arc of the door is a semicircle. The perimeter of the door
dy 10 10 3.142 + 4
dan = 2 = 2 = 2 is 8 metres.
Diberi dx = 0.2 cm s–1. dx x 5 5 (a) Ungkapkan luas permukaan hadapan pintu itu = 1.120 m
dt
Apabila nilai y berubah daripada 3 kepada 3 + p, dalam sebutan x dan π. Luas maksimun permukaan hadapan pintu
Apabila x = 4 dan dx = 0.2, δ y = (3 + p) – 3 = p Express the front surface area of the door in terms of x
dt
and π. = 8(1.120) – 1 π(1.120)2 – 2(1.120)2
dL = dL × dx 2
δ y ≈ dy = 2 [3]
= 4.481 m2
dt dx dt δ x dx 5 (b) Dengan menggunakan π = 3.142, cari luas
= (4x + 40) × 0.2 p
= 2 maximum, dalam m2, permukaan hadapan pintu
= (4 × 4 + 40) × 0.2
δx 5 itu.
= 11.2 cm2 s–1 By using π = 3.142, find the maximum front surface area,
2. Diberi persamaan suatu lengkung ialah y = 3x(1 – 2x)4
δ x = 5p in m2, of the door.
KLON
SPM
dan lengkung itu melalui P(1, 3). Cari
2 ’15 It is given the equation of a curve is y = 3x(1 – 2x)4 and the
[3] 9.2
8. Diberi y = 3x2 – 2x + 3, cari curve passes through P(1, 3). Find
2
Given that y = 3x – 2x + 3, find (a) kecerunan lengkung itu pada titik P.
the gradient of the curve at point P.
dy 10. Diberi fungsi h(x) = kx3 + 3x2 + 4x, cari
xm
(a) nilai bagi apabila x = 2. [4]
9.2 dx 3
Given the function h(x) = kx + 3x + 4x, find 2
ym (b) persamaan garis normal kepada lengkung itu
dy (a) h (x).
the value of when x = 2. pada titik P.
dx 9.2
the equation of the normal to the curve at point P.
(b) perubahan hampir y, dalam sebutan m, apabila
9.5
x berubah daripada 2 kepada 2 + m, dengan
(b) nilai k jika h 
9.6
 12  = 24. 2x m
[3]
keadaan m ialah satu nilai kecil.
the approximate change in y, in terms of m, when x
the value of k if h   
1
2
= 24. (a) Diberi perimeter pintu = 8 m
πx + 2x + y + y = 8
(a) y = 3x(1 – 2x)4
[4] dy
changes from 2 to 2 + m, where m is a small value. 2y = 8 – πx – 2x …… ➀ = 3x(4)(1 – 2x)3(–2) + 3(1 – 2x)4
[4] dx
(a) h(x) = 3kx 2 + 6x + 4 = –24x(1 – 2x)3 + 3(1 – 2x)4
(a) y = 3x 2 – 2x + 3 Katakan luas permukaan hadapan pintu = L m2
dy
= 6x – 2 (b) h(x) = 6kx + 6 L = 1 πx2 + 2xy …… ➁ Pada P(1, 3), x = 1.
dx 2 dy
= –24(1)(1 – 2)3 + 3(1 – 2)4
Apabila x = 2,
dy
h
1
2  
= 24 Gantikan ➀ ke dalam ➁. dx
= 24 + 3
= 6(2) – 2 = 10 L = 1 πx2 + x(8 – πx – 2x) = 27
dx 6k  
1
2
+ 6 = 24 2 Maka, kecerunan lengkung pada titik P ialah 27.
3k = 18 = 1 πx2 + 8x – πx2 – 2x2
(b) Apabila x berubah daripada 2 kepada 2 + m, 2 1
k=6 (b) Kecerunan garis normal pada titik P ialah – .
x = 2 dan δ x = m. 27
= 8x – 1 πx2 – 2x2
dy 2 Persamaan garis normal pada titik P ialah
δy ≈ × δx 1
dx y–3=– (x – 1)
= 10 × m 27
= 10m 27y – 81 = –x + 1
x + 27y = 82

137 138

3. Lengkung y = x3 + x2 – 5x + 10 melalui titik A(2, 12) 4. Diberi persamaan suatu lengkung ialah:
KBAT dan mempunyai dua titik pusingan, P(1, 7) dan Q. KLON
SPM
Given the equation of a curve is: FOKUS KBAT
Cari y = x2(x – 3) + 3
’13

The curve y = x3 + x2 – 5x + 10 passes through the point 2


A(2, 12) and has two turning points, P(1, 7) and Q. Find Kemahiran Kognitif: Mengaplikasi
(a) kecerunan lengkung itu pada A. (a) Cari fungsi kecerunan bagi lengkung itu. Konteks: Pembezaan, Kadar Perubahan
9.2 Find the gradient function of the curve.
9.2 the gradient of the curve at A.
[3] [2] Rajah di bawah menunjukkan sebuah kelalang dasar bulat berjejari 8 cm. Sejenis cecair dituang ke dalam kelalang itu
(b) persamaan normal kepada lengkung itu pada A. (b) Cari kooordinat titik-titik pusingan. dengan kedalaman, h cm, cecair itu bertambah pada kadar 0.5 cm s–1.
9.3 Find the coordinates of the turning points. The diagram shows a round-bottomed flask with a radius of 8 cm. A type of liquid is poured into the flask such that the depth, h cm,
9.2 the equation of the normal to the curve at A.
[3] [3] of the liquid is increased at a rate of 0.5 cm s–1.
(c) koordinat bagi Q dan tentukan sama ada Q ialah (c) Seterusnya, tentukan sama ada setiap titik
9.6
titik maksimum atau titik minimum.
9.6
pusingan itu adalah maksimum atau minimum.
Hence, determine whether each of the turning points is
the coordinates of Q and determine whether Q is a
a maximum or a minimum.
maximum or a minimum point.
[3]
[4]
3
(a) y = x2(x – 3) +
(a) y = x3 + x2 – 5x + 10 2
dy 3
= 3x2 + 2x – 5 = x3 – 3x2 + h cm
dx 2
dy
Pada titik A(2, 12), x = 2. = 3x2 – 6x Cari
dx
Find
Kecerunan lengkung pada A,
dy (a) kadar perubahan luas permukaan cecair itu pada ketika kedalaman cecair ialah 6 cm.
dy (b) Pada titik pusingan, = 0.
= 3(2)2 + 2(2) – 5 = 11 dx the rate of change in the surface area of the liquid at the instant when the depth of the liquid is 6 cm.
dx 3x2 – 6x = 0 (b) perubahan kecil bagi luas permukaan, dalam sebutan π cm2, cecair itu apabila kedalaman cecair berubah daripada
(b) Persamaan normal kepada lengkung pada A ialah 3x(x – 2) = 0 6 cm kepada 6.2 cm.
1 x = 0 atau 2 the approximate change in the surface area, in terms of π cm2, of the liquid when the depth of the liquid changes from 6 cm to 6.2 cm.
y – 12 = – (x – 2) 3
11 Apabila x = 0, y =
11y – 132 = –x + 2 2 (a) Katakan jejari permukaan cecair = r cm
11y + x – 134 = 0 Apabila x = 2, y = 22(2 – 3) +
3 Info KBAT
2 r2 = 82 – (8 – h)2
dy = 64 – 64 + 16h – h2 Gunakan petua rantai untuk menentukan
(c) Pada titik pusingan, = 0. 5
dx =– = 16h – h2 kadar perubahan luas permukaan cecair.
Maka, 3x2 + 2x – 5 = 0 2 Use the chain rule to determine the rate of
Maka, titik pusingan ialah 0, 1
1 1 Luas permukaan cecair, A = πr2
2
(3x + 5)(x – 1) = 0 dan 2, –2  . change in the surface area of the liquid.
5 2 = π(16h – h2)
x = – atau 1 = 16πh – πh2
3 d 2y = 6x – 6
(c) dA
Pada titik P(1, 7), x = 1. dx 2 = 16π – 2πh
dh
5 1 d 2y
Pada titik 0, 1
2  dx 2 Diberi dh = 0.5 cm s–1
Pada titik Q, x = – . : = 6(0) – 6 = –6
3 dt
1
5 3
3
5 2
3
5
y = –  + –  – 5–  + 10
3
0, 1 2  ialah titik maksimum. Apabila h = 6,
dA dA dh
= ×
dt dh dt
13 = [16π – 2π(6)] × 0.5
= 16 1 d 2y 8 cm
Pada titik 2, –2
2  dx 2
27 : = 6(2) – 6 = 6 = 2π cm2 s–1 (8 – h) cm
2 13 r cm
Maka, koordinat bagi Q ialah –1
27 
, 16 . 1 (b) δh = 6.2 – 6 = 0.2 cm
d 2y = 6x + 2
3 2, –2 2
ialah titik minimum. h cm
dA
dx 2 δA ≈ × δh
dh
2
Pada Q –1 , 16
13 = [16π – 2π(6)] × 0.2
27 
,
3 = 0.8π cm2
d 2y = 6 – 5 + 2 = –8  0
dx 2  3 
2 13
Maka, Q –1
27 
, 16 ialah titik maksimum.
3

139 140

35

You might also like